MedSurg 3 Final

¡Supera tus tareas y exámenes ahora con Quizwiz!

9. A nurse was accidently stuck with a needle used on a patient who is infected with human immunodeficiency virus (HIV). After reporting the incident, what care should this nurse first receive? A. Personal protective equipment B. Combination antiretroviral therapy C. Counseling to report blood exposures D. A negative evaluation by the manager

B Postexposure prophylaxis with combination antiretroviral therapy can significantly decrease the risk of infection. Personal protective equipment should be available, although it may not have stopped this needle stick. The needle stick has been reported. The negative evaluation may or may not be needed but would not occur first.

10. A parent does not want their child to have any extra immunizations for diseases that no longer occur. What teaching about immunization should the nurse provide this mother? A. There is currently no need for those older vaccines. B. There is a reemergence of some of the infections, such as pertussis. C. There is no longer an immunization available for some of those diseases. D. The only way to protect your child is to have the federally required vaccines.

B Teaching the parent that some of the diseases are reemerging and the damage they can do to her child gives the mother the information to make an informed decision. The immunizations still exist and do protect individuals.

Physiological Integrity 33. When admitting a patient with a non-ST-segment-elevation myocardial infarction (NSTEMI) to the intensive care unit, which action should the nurse perform first? a. Obtain the blood pressure. b. Attach the cardiac monitor. c. Assess the peripheral pulses. d. Auscultate the breath sounds.

B Because dysrhythmias are the most common complication of myocardial infarction (MI), the first action should be to place the patient on a cardiac monitor. The other actions also are important and should be accomplished as quickly as possible. DIF: Cognitive Level: Apply (application) REF: 756 OBJ: Special Questions: Prioritization TOP: Nursing Process: Implementation MSC:

Physiological Integrity 39. To improve the physical activity level for a mildly obese 71-year-old patient, which action should the nurse plan to take? a. Stress that weight loss is a major benefit of increased exercise. b. Determine what kind of physical activities the patient usually enjoys. c. Tell the patient that older adults should exercise for no more than 20 minutes at a time. d. Teach the patient to include a short warm-up period at the beginning of physical activity.

B Because patients are more likely to continue physical activities that they already enjoy, the nurse will plan to ask the patient about preferred activities. The goal for older adults is 30 minutes of moderate activity on most days. Older adults should plan for a longer warm-up period. Benefits of exercises, such as improved activity tolerance, should be emphasized rather than aiming for significant weight loss in older mildly obese adults. DIF: Cognitive Level: Apply (application) REF: 761 TOP: Nursing Process: Planning MSC:

Psychosocial Integrity 20. When caring for a patient who is recovering from a sudden cardiac death (SCD) event and has no evidence of an acute myocardial infarction (AMI), the nurse will anticipate teaching the patient that a. sudden cardiac death events rarely reoccur. b. additional diagnostic testing will be required. c. long-term anticoagulation therapy will be needed. d. limited physical activity after discharge will be needed to prevent future events.

B Diagnostic testing (e.g., stress test, Holter monitor, electrophysiologic studies, cardiac catheterization) is used to determine the possible cause of the SCD and treatment options. SCD is likely to recur. Anticoagulation therapy will not have any effect on the incidence of SCD, and SCD can occur even when the patient is resting. DIF: Cognitive Level: Apply (application) REF: 762 TOP: Nursing Process: Planning MSC:

Physiological Integrity 13. When titrating IV nitroglycerin (Tridil) for a patient with a myocardial infarction (MI), which action will the nurse take to evaluate the effectiveness of the medication? a. Monitor heart rate. b. Ask about chest pain. c. Check blood pressure. d. Observe for dysrhythmias.

B The goal of IV nitroglycerin administration in MI is relief of chest pain by improving the balance between myocardial oxygen supply and demand. The nurse also will monitor heart rate and blood pressure (BP) and observe for dysrhythmias, but these parameters will not indicate whether the medication is effective. DIF: Cognitive Level: Apply (application) REF: 753 TOP: Nursing Process: Evaluation MSC:

Physiological Integrity 40. Which patient at the cardiovascular clinic requires the most immediate action by the nurse? a. Patient with type 2 diabetes whose current blood glucose level is 145 mg/dL b. Patient with stable angina whose chest pain has recently increased in frequency c. Patient with familial hypercholesterolemia and a total cholesterol of 465 mg/dL d. Patient with chronic hypertension whose blood pressure today is 172/98 mm Hg

B The history of more frequent chest pain suggests that the patient may have unstable angina, which is part of the acute coronary syndrome spectrum. This will require rapid implementation of actions such as cardiac catheterization and possible percutaneous coronary intervention. The data about the other patients suggest that their conditions are stable. DIF: Cognitive Level: Analyze (analysis) REF: 743 OBJ: Special Questions: Prioritization; Multiple Patients TOP: Nursing Process: Planning MSC:

Physiological Integrity 7. After the nurse teaches the patient about the use of carvedilol (Coreg) in preventing anginal episodes, which statement by a patient indicates that the teaching has been effective? a. "Carvedilol will help my heart muscle work harder." b. "It is important not to suddenly stop taking the carvedilol." c. "I can expect to feel short of breath when taking carvedilol." d. "Carvedilol will increase the blood flow to my heart muscle."

B Patients who have been taking -adrenergic blockers can develop intense and frequent angina if the medication is suddenly discontinued. Carvedilol (Coreg) decreases myocardial contractility. Shortness of breath that occurs when taking -adrenergic blockers for angina may be due to bronchospasm and should be reported to the health care provider. Carvedilol works by decreasing myocardial oxygen demand, not by increasing blood flow to the coronary arteries. DIF: Cognitive Level: Apply (application) REF: 745 TOP: Nursing Process: Evaluation MSC:

The nurse in the occupational health clinic prepares to administer the influenza vaccine by nasal spray to an employee. Which question should the nurse ask before administration of this vaccine? A. "Are you allergic to chicken?" B. "Could you be pregnant now?" C. "Did you ever have influenza?" D. "Have you ever had hepatitis B?"

B The live attenuated influenza vaccine (LAIV) is given by nasal spray and approved for healthy people age 2 to 49 years. The LAIV is given only to nonpregnant, healthy people. The inactivated vaccine is given by injection and is approved for use in people 6 months or older. The inactivated vaccine can be used in pregnancy, in people with chronic conditions, or in people who are immunosuppressed. Influenza vaccination is contraindicated if the person has a history of Guillain-Barré syndrome or a hypersensitivity to eggs.

Physiological Integrity 9. Diltiazem (Cardizem) is ordered for a patient with newly diagnosed Prinzmetal's (variant) angina. When teaching the patient, the nurse will include the information that diltiazem will a. reduce heart palpitations. b. decrease spasm of the coronary arteries. c. increase the force of the heart contractions. d. help prevent plaque from forming in the coronary arteries.

B Prinzmetal's angina is caused by coronary artery spasm. Calcium channel blockers (e.g., diltiazem, amlodipine [Norvasc]) are a first-line therapy for this type of angina. Lipid-lowering drugs help reduce atherosclerosis (i.e., plaque formation), and -adrenergic blockers decrease sympathetic stimulation of the heart (i.e., palpitations). Medications or activities that increase myocardial contractility will increase the incidence of angina by increasing oxygen demand. DIF: Cognitive Level: Apply (application) REF: 742 | 744 TOP: Nursing Process: Implementation MSC:

Which task can the registered nurse (RN) delegate to unlicensed assistive personnel (UAP) in the care of a stable patient who has a tracheostomy? A. Assessing the need for suctioning B. Suctioning the patient's oropharynx C. Assessing the patient's swallowing ability D. Maintaining appropriate cuff inflation pressure

B Providing the individual has been trained in correct technique, the UAP may suction the patient's oropharynx. Whereas assessing the need for suctioning should be performed by an RN or licensed practical nurse, swallowing assessment and the maintenance of cuff inflation pressure should be performed solely by the RN.

Physiological Integrity 23. A patient who is recovering from an acute myocardial infarction (AMI) asks the nurse about when sexual intercourse can be resumed. Which response by the nurse is best? a. "Most patients are able to enjoy intercourse without any complications." b. "Sexual activity uses about as much energy as climbing two flights of stairs." c. "The doctor will provide sexual guidelines when your heart is strong enough." d. "Holding and cuddling are good ways to maintain intimacy after a heart attack."

B Sexual activity places about as much physical stress on the cardiovascular system as most moderate-energy activities such as climbing two flights of stairs. The other responses do not directly address the patient's question or may not be accurate for this patient. DIF: Cognitive Level: Apply (application) REF: 761 TOP: Nursing Process: Implementation MSC:

The nurse is teaching a women's group about prevention of hypertension. What information should be included in the teaching for all the women (select all that apply.)? A. Lose weight. B. Limit nuts and seeds. C. Limit sodium and fat intake. D. Increase fruits and vegetables. E. Exercise 30 minutes most days.

C D E Primary prevention of hypertension is to make lifestyle modifications that prevent or delay the increase in BP. Along with exercise for 30 minutes on most days, the DASH eating plan is a healthy way to lower BP by limiting sodium and fat intake, increasing fruits and vegetables, and increasing nutrients that are associated with lowering BP. Nuts and seeds and dried beans are used for protein intake. Weight loss may or may not be necessary for the individual.

A 54-year-old patient admitted with diabetes mellitus, malnutrition, osteomyelitis, and alcohol abuse has a serum amylase level of 280 U/L and a serum lipase level of 310 U/L. To which of the following diagnoses does the nurse attribute these findings? A. Malnutrition B. Osteomyelitis C. Alcohol abuse D. Diabetes mellitus

C) The patient with alcohol abuse could develop pancreatitis as a complication, which would increase the serum amylase (normal 30-122 U/L) and serum lipase (normal 31-186 U/L) levels as shown.

When providing nutritional counseling for patients at risk for CAD, which foods would the nurse encourage patients to include in their diet (select all that apply)? A. Tofu B. Walnuts C. Tuna fish D. Whole milk E. Orange juice

A. Tofu B. Walnuts C. Tuna fish Tuna fish, tofu, and walnuts are all rich in omega-3 fatty acids, which have been shown to reduce the risks associated with CAD when consumed regularly.

A patient taking ibuprofen (Motrin) for treatment of OA has good pain relief but is experiencing increased dyspepsia and nausea with the drug's use. The nurse consults the patient's primary care provider about a. adding misoprostol (Cytotec) to the patient's drug regimen b. substituting naproxen (Naprosyn) for the ibuprofen (Motrin) c. administering the ibuprofen with antacids to decrease the GI irritation d. returning to the use of acetaminophen, but at a dose of 5 g/day instead of 4 g/day

A. adding misoprostol (Cytotec) to the patient's drug regimen

The nurse suggests that a patient recently diagnosed with rheumatoid arthritis (RA) plan to start each day with a. a warm bath followed by a short rest. b. a short routine of isometric exercises. c. active range-of-motion (ROM) exercises. d. stretching exercises to relieve joint stiffness.

ANS: A Taking a warm shower or bath is recommended to relieve joint stiffness, which is worse in the morning. Isometric exercises would place stress on joints and would not be recommended. Stretching and ROM should be done later in the day, when joint stiffness is decreased.

Which assessment information obtained by the nurse indicates that a patient with an exacerbation of rheumatoid arthritis (RA) is experiencing a side effect of prednisone (Deltasone)? a. The patient has joint pain and stiffness. b. The patient's blood glucose is 165 mg/dL. c. The patient has experienced a recent 5-pound weight loss. d. The patient's erythrocyte sedimentation rate (ESR) has increased.

ANS: B Corticosteroids have the potential to cause diabetes mellitus. The finding of an elevated blood glucose reflects this side effect of prednisone. Corticosteroids increase appetite and lead to weight gain. An elevated ESR and no improvement in symptoms would indicate that the prednisone was not effective but would not be side effects of the medication

A patient is being discharged after 2 weeks of IV antibiotic therapy for acute osteomyelitis in the left leg. Which information will be included in the discharge teaching? a. How to apply warm packs safely to the leg to reduce pain b. How to monitor and care for the long-term IV catheter site c. The need for daily aerobic exercise to help maintain muscle strength d. The reason for taking oral antibiotics for 7 to 10 days after discharge

ANS: B The patient will be on IV antibiotics for several months, and the patient will need to recognize signs of infection at the IV site and how to care for the catheter during daily activities such as bathing. IV antibiotics rather than oral antibiotics are used for acute osteomyelitis. Patients are instructed to avoid exercise and heat application because these will increase swelling and the risk for spreading infection.

A patient is hospitalized with new onset of Guillain-Barré syndrome. The most essential assessment for the nurse to complete is a. determining level of consciousness. b. checking strength of the extremities. c. observing respiratory rate and effort. d. monitoring the cardiac rate and rhythm.

ANS: C The most serious complication of Guillain-Barré syndrome is respiratory failure, and the nurse should monitor respiratory function continuously. The other assessments will also be included in nursing care, but they are not as important as respiratory assessment. DIF: Cognitive Level: Analyze (analysis)

Which menu choice by a patient with osteoporosis indicates that the nurse's teaching about appropriate diet has been effective? a. Pancakes with syrup and bacon b. Whole wheat toast and fresh fruit c. Egg-white omelet and a half grapefruit d. Oatmeal with skim milk and fruit yogurt

ANS: D Skim milk and yogurt are high in calcium. The other choices do not contain any high-calcium foods

A patient with newly diagnosed SLE asks the nurse how the disease will affect her life. The best response by the nurse is a. You can plan to have a near-normal life since SLE rarely causes death b. it is difficult to tell because to disease is so variable in its severity and progression c. life span is shortened somewhat in people with SLE, but the disease can be controlled with long-term use of corticosteroids d. most people with SLE have alternating periods of remissions and exacerbations with rapid progression to permanent organ damage

B. it is difficult to tell because the disease is so variable in its severity and progression

To preserve function and the ability to perform activities of daily living, the nurse teaches the patient with OA to a. avoid exercise that involves the affected joints b. plan and organize less stressful ways to perform tasks c. maintain normal activities during an acute episode to prevent loss of function d. use mild analgesics to control symptoms when performing tasks that cause pain

B. plan and organize less stressful ways to perform tasks

D

Before discharge, the nurse discusses activity levels with a 61-yr-old patient with chronic obstructive pulmonary disease (COPD) and pneumonia. Which exercise goal is most appropriate once the patient is fully recovered from this episode of illness? Slightly increase activity over the current level. Swim for 10 min/day, gradually increasing to 30 min/day. Limit exercise to activities of daily living to conserve energy. Walk for 20 min/day, keeping the pulse rate less than 130 beats/min.

5. The nurse is providing care for a patient who has been living with human immunodeficiency virus (HIV) for several years. Which assessment finding most clearly indicates an acute exacerbation of the disease? A. A new onset of polycythemia B. Presence of mononucleosis-like symptoms C. A sharp decrease in the patient's CD4+ count D. A sudden increase in the patient's WBC count

C A decrease in CD4+ count signals an exacerbation of the severity of HIV. Polycythemia is not characteristic of the course of HIV. A patient's WBC count is very unlikely to suddenly increase, with decreases being typical. Mononucleosis-like symptoms such as malaise, headache, and fatigue are typical of early HIV infection and seroconversion.

16. The nurse is monitoring the effectiveness of antiretroviral therapy (ART) for a patient with acquired immunodeficiency syndrome (AIDS). What laboratory study result indicates the medications are effective? A. Increased viral load B. Decreased neutrophil count C. Increased CD4+ T cell count D. Decreased white blood cell count

C Antiretroviral therapy is effective if there are decreased viral loads and increased CD4+ T cell counts.

After teaching a patient with chronic stable angina about nitroglycerin, the nurse recognizes the need for further teaching when the patient makes which statement? A. "I will replace my nitroglycerin supply every 6 months." B. "I can take up to five tablets every 3 minutes for relief of my chest pain." C. "I will take acetaminophen (Tylenol) to treat the headache caused by nitroglycerin." D. "I will take the nitroglycerin 10 minutes before planned activity that usually causes chest pain."

B. "I can take up to five tablets every 3 minutes for relief of my chest pain." The recommended dose of nitroglycerin is one tablet taken sublingually (SL) or one metered spray for symptoms of angina. If symptoms are unchanged or worse after 5 minutes, the patient should be instructed to activate the emergency medical services (EMS) system. If symptoms are improved, repeat the nitroglycerin every 5 minutes for a maximum of three doses and contact EMS if symptoms have not resolved completely.

During a health screening event which assessment finding would alert the nurse to the possible presence of osteoporosis in a white 61-year-old female? A. The presence of bowed legs B. A measurable loss of height C. Poor appetite and aversion to dairy products D. Development of unstable, wide-gait ambulation

B. A measurable loss of height A gradual but measurable loss of height and the development of kyphosis or "dowager's hump" are indicative of the presence of osteoporosis in which the rate of bone resorption is greater than bone deposition. Bowed legs may be caused by abnormal bone development or rickets but is not indicative of osteoporosis. Lack of calcium and Vitamin D intake may cause osteoporosis but are not indicative it is present. A wide gait is used to support balance and does not indicate osteoporosis.

The patient comes to the ED with severe, prolonged angina that is not immediately reversible. The nurse knows that if the patient once had angina related to a stable atherosclerotic plaque and the plaque ruptures, there may be occlusion of a coronary vessel and this type of pain. How will the nurse document this situation related to pathophysiology, presentation, diagnosis, prognosis, and interventions for this disorder? A. Unstable angina B. Acute coronary syndrome (ACS) C. ST-segment-elevation myocardial infarction (STEMI) D. Non-ST-segment-elevation myocardial infarction (NSTEMI)

B. Acute coronary syndrome (ACS) The pain with ACS is severe, prolonged, and not easy to relieve. ACS is associated with deterioration of a once-stable atherosclerotic plaque that ruptures, exposes the intima to blood, and stimulates platelet aggregation and local vasoconstriction with thrombus formation. The unstable lesion, if partially occlusive, will be manifest as unstable angina or NSTEMI. If there is total occlusion, it is manifest as a STEMI.

The nurse would assess a patient with complaints of chest pain for which clinical manifestations associated with a myocardial infarction (MI) (select all that apply)? A. Flushing B. Ashen skin C. Diaphoresis D. Nausea and vomiting E. S3 or S4 heart sounds

B. Ashen skin C. Diaphoresis D. Nausea and vomiting E. S3 or S4 heart sounds During the initial phase of an MI, catecholamines are released from the ischemic myocardial cells, causing increased sympathetic nervous system (SNS) stimulation. This results in the release of glycogen, diaphoresis, and vasoconstriction of peripheral blood vessels. The patient's skin may be ashen, cool, and clammy (not flushed) as a result of this response. Nausea and vomiting may result from reflex stimulation of the vomiting center by severe pain. Ventricular dysfunction resulting from the MI may lead to the presence of the abnormal S3 and S4 heart sounds.

Evolve Because the incidence of Lyme disease is very high in Wisconsin, the public health nurse is planning to provide community education to increase the number of people who seek health care promptly after a tick bite. What information should the nurse provide when teaching people who are at risk for a tick bite? A The best therapy for the acute illness is an IV antibiotic. B Check for an enlarging reddened area with a clear center. C Surveillance is necessary during the summer months only. D Antibiotics will prevent Lyme disease if taken for 10 days.

B. Check for an enlarging reddened area with a clear center.

The 24-year-old male patient who was successfully treated for Paget's disease has come to the clinic with a gradual onset of pain and swelling around the left knee. The patient is diagnosed with osteosarcoma without metastasis. The patient wants to know why he will be given chemotherapy before the surgery. What is the best rationale the nurse should tell the patient? A. The chemotherapy is being used to save your left leg. B. Chemotherapy is being used to decrease the tumor size. C. The chemotherapy will increase your 5-year survival rate. D. Chemotherapy will help decrease the pain before and after surgery.

B. Chemotherapy is being used to decrease the tumor size. Preoperative chemotherapy is used to decrease tumor size before surgery. The chemotherapy will not save his leg if the lesion is too big or there is neurovascular or muscle involvement. Adjunct chemotherapy after amputation or limb salvage has increased 5-year survival rate in people without metastasis. Chemotherapy is not used to decrease pain before or after surgery.

During treatment of the patient with an acute attack of gout, the nurse would expect to administer a. aspirin b. colchicine c. allopurinol (Zyloprim) d. probenecid (Benemid)

B. Colchicine

The nurse identifies a nursing diagnosis of pain related to muscle spasms for a 45-year-old patient who has low back pain from a herniated lumbar disc. What would be an appropriate nursing intervention to treat this problem? A. Provide gentle ROM to the lower extremities. B. Elevate the head of the bed 20 degrees and flex the knees. C. Place the bed in reverse Trendelenburg with the feet firmly against the footboard. D. Place a small pillow under the patient's upper back to gently flex the lumbar spine.

B. Elevate the head of the bed 20 degrees and flex the knees. The nurse should elevate the head of the bed 20 degrees and flex the knees to avoid extension of the spine and increasing the pain. The slight flexion provided by this position often is comfortable for a patient with a herniated lumbar disc. ROM to the lower extremities will be limited to prevent extremes of spinal movement. Reverse Trendelenburg and a pillow under the patient's upper back will more likely increase pain.

Which antilipemic medications should the nurse question for a patient with cirrhosis of the liver (select all that apply)? A. Niacin (Nicobid) B. Ezetimibe (Zetia) C. Gemfibrozil (Lopid) D. Atorvastatin (Lipitor) E. Cholestyramine (Questran)

B. Ezetimibe (Zetia) D. Atorvastatin (Lipitor) Ezetimibe (Zetia) should not be used by patients with liver impairment. Adverse effects of atorvastatin (Lipitor), a statin drug, include liver damage and myopathy. Liver enzymes must be monitored frequently and the medication stopped if these enzymes increase. Niacin's side effects subside with time, although decreased liver function may occur with high doses. Cholestyramine is safe for long-term use.

After teaching a patient with RA about the prescribed therapeutic regimen, the nurse determines that further instruction is needed when the patient says, a. it is important for me to perform my prescribed exercises every day b. I should perform most of my daily chores in the morning when my energy level is highest c. an ice pack to a joint for 10 minutes may help relieve pain and inflammation when I have an acute flare d. I can use assistive devices such as padded utensils, electric can openers, and elevated toilet seats to protect my joints

B. I should perform most of my daily chores in the morning when my energy level is highest

The nurse is caring for a patient admitted to the nursing unit with osteomyelitis of the tibia. Which symptom will the nurse most likely find on physical examination of the patient? A. Nausea and vomiting B. Localized pain and warmth C. Paresthesia in the affected extremity D. Generalized bone pain throughout the leg

B. Localized pain and warmth Osteomyelitis is an infection of bone and bone marrow that can occur with trauma, surgery, or spread from another part of the body. Because it is an infection, the patient will exhibit typical signs of inflammation and infection, including localized pain and warmth. Nausea and vomiting and paresthesia of the extremity are not expected to occur. Pain occurs, but it is localized, not generalized throughout the leg.

The nurse is examining the ECG of a patient who has just been admitted with a suspected MI. Which ECG change is most indicative of prolonged or complete coronary occlusion? A. Sinus tachycardia B. Pathologic Q wave C. Fibrillatory P waves D. Prolonged PR interval

B. Pathologic Q wave The presence of a pathologic Q wave, as often accompanies STEMI, is indicative of complete coronary occlusion. Sinus tachycardia, fibrillatory P waves (e.g., atrial fibrillation), or a prolonged PR interval (first-degree heart block) are not direct indicators of extensive occlusion.

A patient is seeking emergency care after choking on a piece of steak. The nursing assessment reveals a history of alcoholism, cigarette smoking, and hemoptysis. Which diagnostic study is most likely to be performed on this patient? A) Barium swallow B) Endoscopic biopsy C) Capsule endoscopy D) Endoscopic ultrasonography

B) Because of this patient's history of excessive alcohol intake, smoking, hemoptysis, and the current choking episode, cancer may be present. A biopsy is necessary to make a definitive diagnosis of carcinoma, so an endoscope will be used to obtain a biopsy and observe other abnormalities as well. A barium swallow may show narrowing of the esophagus, but it is more diagnostic for achalasia. An endoscopic ultrasonography may be used to stage esophageal cancer. Capsule endoscopy can show alterations in the esophagus but is more often used for small intestine problems. A barium swallow, capsule endoscopy, and endoscopic ultrasonography cannot provide a definitive diagnosis for cancer when it is suspected.

Inspection of an older patient's mouth reveals the presence of white, curd-like lesions on the patient's tongue. What is the most likely etiology for this abnormal assessment finding? A) Herpesvirus B) Candida albicans C) Vitamin deficiency D) Irritation from ill-fitting dentures

B) White, curd-like lesions surrounded by erythematous mucosa are associated with oral candidiasis. Herpesvirus causes benign vesicular lesions in the mouth. Vitamin deficiencies may cause a reddened, ulcerated, swollen tongue. Irritation from ill-fitting dentures will cause friable, edematous, painful, bleeding gingivae.

The nurse should recognize that the liver performs which functions (select all that apply)? A) Bile storage B) Detoxification C) Protein metabolism D) Steroid metabolism E) Red blood cell (RBC) destruction

B, C, D) The liver performs multiple major functions that aid in the maintenance of homeostasis. These include metabolism of proteins and steroids as well as detoxification of drugs and metabolic waste products. The Kupffer cells of the liver participate in the breakdown of old RBCs. The liver produces bile, but storage occurs in the gall bladder.

20. A patient is admitted to the emergency department (ED) with fever, swollen lymph glands, sore throat, headache, malaise, joint pain, and diarrhea. What nursing actions will help identify the need for further assessment of the cause of this patient's manifestations (select all that apply.)? A. Assessment of lung sounds B. Assessment of sexual behavior C. Assessment of living conditions D. Assessment of drug and syringe use E. Assessment of exposure to an ill person

B, D With these symptoms, assessing this patient's sexual behavior and possible exposure to shared drug equipment will identify if further assessment for the HIV virus should be made or the manifestations are from some other illness (e.g., lung sounds and living conditions may indicate further testing for TB).

The nurse is planning health promotion teaching for a 45-year-old patient with asthma, low back pain from herniated lumbar disc, and schizophrenia. What does the nurse determine would be the best exercise to include in an individualized exercise plan for the patient? A. Yoga B. Walking C. Calisthenics D. Weight lifting

B. Walking The patient would benefit from an aerobic exercise that takes into account the patient's health status and fits the patient's lifestyle. The best exercise is walking, which builds strength in the back and leg muscles without putting undue pressure or strain on the spine. Yoga, calisthenics, and weight lifting would all put pressure on or strain the spine.

A patient recovering from an acute exacerbation of RA tells the nurse she is too tired to bathe. The nurse should a. give the patient a bed bath to conserve her energy b. allow the patient a rest period before showering with the nurses' help c. tell the patient that she can skip bathing if she will walk in the hall later d. inform the patient that it is important for her to maintain self-care activities

B. allow the patient a rest period before showering with the nurses' help

A patient with gout is treated with drug therapy to prevent future attacks. The nurse teaches the patient that is is the most important to a. avoid all foods high in purine, such as organ meats b. have periodic determination of serum uric acid levels c. perform active ROM of all joints that have been affected by gout d. increase the dosage of medication with the onset of an acute attack

B. have periodic determination of serum uric acid levels

7. A pregnant woman who was tested and diagnosed with human immunodeficiency virus (HIV) infection is very upset. What should the nurse teach this patient about her baby's risk of being born with HIV infection? A. "The baby will probably be infected with HIV." B. "Only an abortion will keep your baby from having HIV." C. "Treatment with antiretroviral therapy will decrease the baby's chance of HIV infection." D. "The duration and frequency of contact with the organism will determine if the baby gets HIV infection."

C On average, 25% of infants born to women with untreated HIV will be born with HIV. The risk of transmission is reduced to less than 2% if the infected pregnant woman is treated with antiretroviral therapy. Duration and frequency of contact with the HIV organism is one variable that influences whether transmission of HIV occurs. Volume, virulence, and concentration of the organism as well as host immune status are variables related to transmission via blood, semen, vaginal secretions, or breast milk.

Physiological Integrity 15. Following an acute myocardial infarction (AMI), a patient ambulates in the hospital hallway. When the nurse is evaluating the patient's response to the activity, which assessment data would indicate that the exercise level should be decreased? a. Blood pressure (BP) changes from 118/60 to 126/68 mm Hg. b. Oxygen saturation drops from 99% to 95%. c. Heart rate increases from 66 to 92 beats/minute. d. Respiratory rate goes from 14 to 20 breaths/minute.

C A change in heart rate of more than 20 beats over the resting heart rate indicates that the patient should stop and rest. The increases in BP and respiratory rate, and the slight decrease in oxygen saturation, are normal responses to exercise. DIF: Cognitive Level: Apply (application) REF: 761 TOP: Nursing Process: Evaluation MSC:

6. Which type of bone tumor is a benign overgrowth of bone and cartilage and may transform into a malignant form? a. Endochroma b. Osteoclastoma c. Ewing's sarcoma d. Osteochondroma

6. d. Osteochondroma is a benign overgrowth of bone and cartilage near the end of the bone at the growth plate, especially in long bones, pelvis, or scapula. It may transform to a malignant form. Osteoclastoma is a benign bone tumor with a high rate of recurrence, but does not become malignant. Endochroma is benign but is an intramedullary cartilage tumor found in a cavity of a single hand or foot bone. Ewing's sarcoma develops in the medullary cavity of long bones, especially the femur, humerus, pelvis, and tibia.

7. Screening for HIV infection generally involves a. detecting CD8+ cytotoxic T cells in saliva. b. laboratory analysis of saliva to detect CD4+ T cells. c. analysis of lymph tissues for the presence of HIV RNA. d. laboratory analysis of blood to detect HIV antigen or antibody.

7. Correct answer: d Rationale: The most useful screening tests for HIV detect HIV-specific antibodies and/or antigen.

The nurse is caring for a group of patients on a medical unit. After receiving report, which patient should the nurse see first? A 42-yr-old patient with multiple sclerosis who was admitted with sepsis A 72-yr-old patient with Parkinson's disease who has aspiration pneumonia A 38-yr-old patient with myasthenia gravis who declined prescribed medications A 45-yr-old patient with amyotrophic lateral sclerosis who refuses enteral feedings

A 38-yr-old patient with myasthenia gravis who declined prescribed medications Patients with myasthenia gravis who discontinue pyridostigmine (Mestinon) will experience myasthenic crisis. Myasthenia crisis results in severe muscle weakness and can lead to a respiratory arrest.

Which assessment information will the nurse obtain to evaluate the effectiveness of the prescribed calcitonin (Cibacalcin) and ibandronate (Boniva) for a patient with Paget's disease? a. Pain level b. Oral intake c. Daily weight d. Grip strength

ANS: A Bone pain is one of the common early manifestations of Paget's disease, and the nurse should assess the pain level to determine whether the treatment is effective. The other information will also be collected by the nurse, but will not be used in evaluating the effectiveness of the therapy.

The nurse determines that additional instruction is needed when a patient diagnosed with scleroderma says which of the following? a. "Paraffin baths can be used to help my hands." b. "I should lie down for an hour after each meal." c. "Lotions will help if I rub them in for a long time." d. "I should perform range-of-motion exercises daily."

ANS: B Because of the esophageal scarring, patients should sit up for 2 hours after eating. The other patient statements are correct and indicate that the teaching has been effective.

Which action will the nurse take when caring for a patient who develops tetanus from injectable substance use? a. Avoid use of sedatives. b. Provide a quiet environment. c. Provide range-of-motion exercises daily. d. Check pupil reaction to light every 4 hours.

ANS: B In patients with tetanus, painful seizures can be precipitated by jarring, loud noises, or bright lights, so the nurse will minimize noise and avoid shining light into the patient's eyes. Range-of-motion exercises may also stimulate the patient and cause seizures. Although the patient has a history of injectable drug use, sedative medications will be needed to decrease spasms. DIF: Cognitive Level: Apply (application)

After change-of-shift report on the neurology unit, which patient will the nurse assess first? a. Patient with Bell's palsy who has herpes vesicles in front of the ear b. Patient with botulism who is drooling and experiencing difficulty swallowing c. Patient with neurosyphilis who has tabes dorsalis and decreased deep tendon reflexes d. Patient with an abscess caused by injectable drug use who needs tetanus immune globulin

ANS: B The patient's diagnosis and difficulty swallowing indicate the nurse should rapidly assess for respiratory distress. The information about the other patients is consistent with their diagnoses and does not indicate any immediate need for assessment or intervention. DIF: Cognitive Level: Analyze (analysis)

When administering alendronate (Fosamax) to a patient with osteoporosis, the nurse will a. ask about any leg cramps or hot flashes. b. assist the patient to sit up at the bedside. c. be sure that the patient has recently eaten. d. administer the ordered calcium carbonate.

ANS: B To avoid esophageal erosions, the patient taking bisphosphonates should be upright for at least 30 minutes after taking the medication. Fosamax should be taken on an empty stomach, not after taking other medications or eating. Leg cramps and hot flashes are not side effects of bisphosphonates

A

Although a diagnosis of cystic fibrosis (CF) is most often made before age 2 years, an 18-yr-old patient at the student health center with a history of frequent lung and sinus infections has clinical manifestations consistent with undiagnosed CF. Which information would be accurate for the nurse to include when teaching the patient about a scheduled sweat chloride test? "Sweat chloride greater than 60 mmol/L is consistent with a diagnosis of CF." "The test measures the amount of sodium chloride in your postexercise sweat." "If sweating occurs after an oral dose of pilocarpine, the test result for CP is positive." "If the sweat chloride test result is positive on two occasions, genetic testing will be necessary."

13. The nurse is teaching a group of young adults who live in a dormitory about the prevention of antibiotic-resistant infections. What should be included in the teaching plan? A. Save leftover antibiotics for future uses. B. Hand washing can prevent many infections. C. Antibiotics are indicated for preventing most colds. D. Stop taking prescribed antibiotics when symptoms improve.

B Hand washing is the single most important action to prevent infections. Antibiotics are used to treat bacterial infections, not colds and flu caused by viruses. Patients should complete the entire prescription of antibiotics to prevent the development of resistant bacteria. Antibiotics should not be taken to prevent infections unless they are given prophylactically before undergoing certain surgeries and dental work.

A patient who was burned in a workplace accident has completed the acute phase of treatment and the plan of care has been altered to prioritize rehabilitation. What nursing action should be prioritized during this phase of treatment? A) Monitoring fluid and electrolyte imbalances B) Providing education to the patient and family C) Treating infection D) Promoting thermoregulation

Ans: B Feedback: Patient and family education is a priority during rehabilitation. There should be no fluid and electrolyte imbalances in the rehabilitation phase. The presence of impaired thermoregulation or infection would suggest that the patient is still in the acute phase of burn recovery.

A patient with a history of chronic hypertension is being evaluated in the emergency department for a blood pressure of 200/140 mm Hg. Which patient assessment question is the priority? A. Is the patient pregnant? B. Does the patient need to urinate? C. Does the patient have a headache or confusion? D. Is the patient taking antiseizure medications as prescribed?

C. The nurse's priority assessments include neurologic deficits, retinal damage, heart failure, pulmonary edema, and renal failure. The headache or confusion could be seen with hypertensive encephalopathy from increased cerebral capillary permeability leading to cerebral edema. In addition, headache or confusion could represent signs and symptoms of a hemorrhagic stroke. Pregnancy can lead to secondary hypertension. Needing to urinate and taking antiseizure medication do not support a hypertensive emergency.

The health care provider orders lactulose for a patient with hepatic encephalopathy. The nurse will monitor for effectiveness of this medication for this patient by assessing which of the following? A. Relief of constipation B. Relief of abdominal pain C. Decreased liver enzymes D. Decreased ammonia levels

D) Hepatic encephalopathy is a complication of liver disease and is associated with elevated serum ammonia levels. Lactulose traps ammonia in the intestinal tract. Its laxative effect then expels the ammonia from the colon, resulting in decreased serum ammonia levels and correction of hepatic encephalopathy.

Safe and Effective Care Environment 37. A patient who has chest pain is admitted to the emergency department (ED) and all of the following are ordered. Which one should the nurse arrange to be completed first? a. Chest x-ray b. Troponin level c. Electrocardiogram (ECG) d. Insertion of a peripheral IV

C The priority for the patient is to determine whether an acute myocardial infarction (AMI) is occurring so that reperfusion therapy can begin as quickly as possible. ECG changes occur very rapidly after coronary artery occlusion, and an ECG should be obtained as soon as possible. Troponin levels will increase after about 3 hours. Data from the chest x-ray may impact the patient's care but are not helpful in determining whether the patient is experiencing a myocardial infarction (MI). Peripheral access will be needed but not before the ECG. DIF: Cognitive Level: Apply (application) REF: 749 OBJ: Special Questions: Prioritization TOP: Nursing Process: Implementation MSC:

Safe and Effective Care Environment 41. A patient with diabetes mellitus and chronic stable angina has a new order for captopril (Capoten). The nurse should teach the patient that the primary purpose of captopril is to a. lower heart rate. b. control blood glucose levels. c. prevent changes in heart muscle. d. reduce the frequency of chest pain.

C The purpose for angiotensin-converting enzyme (ACE) inhibitors in patients with chronic stable angina who are at high risk for a cardiac event is to decrease ventricular remodeling. ACE inhibitors do not directly impact angina frequency, blood glucose, or heart rate. DIF: Cognitive Level: Apply (application) REF: 745 TOP: Nursing Process: Planning MSC:

A nurse is assessing the recent health history of a 63-year-old patient with osteoarthritis (OA). The nurse determines that the patient is trying to manage the condition appropriately when the patient describes which activity pattern? A) Bed rest with bathroom privileges B) Daily high-impact aerobic exercise C) Regular exercise program of walking D) Frequent rest periods with minimal exercise

C) A regular low-impact exercise, such as walking, is important in helping to maintain joint mobility in the patient with osteoarthritis. A balance of rest and activity is needed. High-impact aerobic exercises would cause stress to affected joints and further damage.

A patient with cholelithiasis needs to have the gallbladder removed. Which patient assessment is a contraindication for a cholecystectomy? A) Low-grade fever of 100° F and dehydration B) Abscess in the right upper quadrant of the abdomen C) Activated partial thromboplastin time (aPTT) of 54 seconds D) Multiple obstructions in the cystic and common bile duct

C) An aPTT of 54 seconds is above normal and indicates insufficient clotting ability. If the patient had surgery, significant bleeding complications postoperatively are very likely. Fluids can be given to eliminate the dehydration; the abscess can be assessed, and the obstructions in the cystic and common bile duct would be relieved with the cholecystectomy.

The nurse is caring for a patient who has osteoarthritis (OA) of the knees. The nurse teaches the patient that the most beneficial measure to protect the joints is to do what? A) Use a wheelchair to avoid walking as much as possible. B) Sit in chairs that cause the hips to be lower than the knees. C) Eat a well-balanced diet to maintain a healthy body weight. D) Use a walker for ambulation to relieve the pressure on the hips

C) Because maintaining an appropriate load on the joints is essential to the preservation of articular cartilage integrity, the patient should maintain an optimal overall body weight or lose weight if overweight. Walking is encouraged. The chairs that would be best for this patient have a higher seat and armrests to facilitate sitting and rising from the chair. Relieving pressure on the hips is not important for OA of the knees.

When assessing a patient's abdomen, what would be most appropriate for the nurse to do? A) Palpate the abdomen before auscultation. B) Percuss the abdomen before auscultation. C) Auscultate the abdomen before palpation. D) Perform deep palpation before light palpation.

C) During examination of the abdomen, auscultation is done before percussion and palpation because these latter procedures may alter the bowel sounds.

A female patient has a sliding hiatal hernia. What nursing interventions will prevent the symptoms of heartburn and dyspepsia that she is experiencing? A) Keep the patient NPO. B) Put the bed in the Trendelenberg position. C) Have the patient eat 4 to 6 smaller meals each day. D) Give various antacids to determine which one works for the patient.

C) Eating smaller meals during the day will decrease the gastric pressure and the symptoms of hiatal hernia. Keeping the patient NPO or in a Trendelenberg position are not safe or realistic for a long period of time for any patient. Varying antacids will only be done with the care provider's prescription, so this is not a nursing intervention.

The health care team is assessing a male patient for acute pancreatitis after he presented to the emergency department with severe abdominal pain. Which laboratory value is the best diagnostic indicator of acute pancreatitis? A) Gastric pH B) Blood glucose C) Serum amylase D) Serum potassium

C) Elevated serum amylase levels indicate early pancreatic dysfunction and are used to diagnose acute pancreatitis. Serum lipase levels stay elevated longer than serum amylase in acute pancreatitis. Blood glucose, gastric pH, and potassium levels are not direct indicators of acute pancreatic dysfunction.

The nurse determines that a pnt has experienced the beneficial effects of medication therapy with famotidine (Pepcid) when which of the following symptoms is relieved? A) Nausea B) Belching C) Epigastric pain D) Difficulty swallowing

C) Epigastric pain Famotidine is an H2-receptor antagonist that inhibits parietal cell output of HCl acid and minimizes damage to gastric mucosa related to hyperacidity, thus relieving epigastric pain.

The condition of a patient who has cirrhosis of the liver has deteriorated. Which diagnostic study would help determine if the patient has developed liver cancer? A) Serum α-fetoprotein level B) Ventilation/perfusion scan C) Hepatic structure ultrasound D) Abdominal girth measurement

C) Hepatic structure ultrasound, CT, and MRI are used to screen and diagnose liver cancer. Serum α-fetoprotein level may be elevated with liver cancer or other liver problems. Ventilation/perfusion scans do not diagnose liver cancer. Abdominal girth measurement would not differentiate between cirrhosis and liver cancer

A patient who is scheduled for surgery with general anesthesia in 1 hour is observed with a moist, but empty water glass in his hand. Which assessment finding may indicate that the patient drank a glass of water? A) Flat abdomen without movement upon inspection B) Tenderness at left upper quadrant upon palpation C) Easily heard, loud gurgling in the right upper quadrant D) High-pitched, hollow sounds in the left upper quadrant

C) If the patient drank water on an empty stomach, gurgling can be assessed without a stethoscope or assessed with auscultation. High-pitched, hollow sounds are tympanic and indicate an empty cavity. A flat abdomen and tenderness do not indicate that the patient drank a glass of water.

The patient receiving chemotherapy rings the call bell and reports an onset of nausea. The nurse should prepare a prn dose of which of the following medications? A) Morphine sulfate B) Zolpidem (Ambien) C) Ondansetron (Zofran) D) Dexamethasone (Decadron)

C) Ondansetron (Zofran) (Ondansetron is a 5-HT3 receptor antagonist antiemetic that is especially effective in reducing cancer chemotherapy-induced nausea and vomiting.)

When caring for the patient with heart failure, the nurse knows that which gastrointestinal process is most dependent on cardiac output and may affect the patient's nutritional status? A) Ingestion B) Digestion C) Absorption D) Elimination

C) Substances that interface with the absorptive surfaces of the GI tract (primarily in the small intestine) diffuse across the intestinal membranes into intestinal capillaries and are then carried to other parts of the body for use in energy production. The cardiac output provides the blood flow for this absorption of nutrients to occur.

Physiological Integrity 35. The nurse obtains the following data when assessing a patient who experienced an ST-segment-elevation myocardial infarction (STEMI) 2 days previously. Which information is most important to report to the health care provider? a. The troponin level is elevated. b. The patient denies ever having a heart attack. c. Bilateral crackles are auscultated in the mid-lower lobes. d. The patient has occasional premature atrial contractions (PACs).

C The crackles indicate that the patient may be developing heart failure, a possible complication of myocardial infarction (MI). The health care provider may need to order medications such as diuretics or angiotensin-converting enzyme (ACE) inhibitors for the patient. Elevation in troponin level at this time is expected. PACs are not life-threatening dysrhythmias. Denial is a common response in the immediate period after the MI. DIF: Cognitive Level: Apply (application) REF: 748 OBJ: Special Questions: Prioritization TOP: Nursing Process: Assessment MSC:

Physiological Integrity 17. A patient is recovering from a myocardial infarction (MI) and develops chest pain on day 3 that increases when taking a deep breath and is relieved by leaning forward. Which action should the nurse take next? a. Assess the feet for pedal edema. b. Palpate the radial pulses bilaterally. c. Auscultate for a pericardial friction rub. d. Check the heart monitor for dysrhythmias.

C The patient's symptoms are consistent with the development of pericarditis, a possible complication of MI. The other assessments listed are not consistent with the description of the patient's symptoms. DIF: Cognitive Level: Apply (application) REF: 749 TOP: Nursing Process: Implementation MSC:

The nurse is caring for a patient with a tracheostomy. What is the priority nursing assessment for this patient? A. Electrolyte levels and daily weights B. Assessment of speech and swallowing C. Respiratory rate and oxygen saturation D. Pain assessment and assessment of mobility

C The priority assessment in the care of a patient with a tracheostomy focuses on airway and breathing. These assessments supersede the nurse's assessments that may also be necessary, such as nutritional status, speech, pain, and swallowing ability.

The basic pathophysiologic process of rheumatoid arthritis (RA) is a. destruction of joint cartilage and bones by an autoimmune process b. initiated by a viral infection that destroys the synovial membranes of joints c. the presence of HLA-DR4 antigen that causes inflammatory responses throughout the body d. an immune response that activates complement and produces inflammation of joints and other organ systems

D. an immune response that activates complement and produces inflammation of joints and other organ systems

A pt reports having dry mouth and asks for some liquid to drink. The nurse reasons that this symptom can most likely be attributed to a common adverse effect of which of the following medications? A) Digoxin (Lanoxin) B) Cefotetan (Cefotan) C) Famotidine (Pepcid) D) Promethazine (Phenergan)

D) Promethazine (Phenergan) A common adverse effect of promethazine, an antihistamine antiemetic agent, is dry mouth; another is blurred vision.

A female patient complains of a throbbing headache. The nurse learns the patient has experienced photophobia and headaches previously. Which diagnosis does the nurse suspect? Cluster headache Migraine headache Polycythemia vera Hemorrhagic stroke

Migraine headache Although a headache may occur with any of these options, a *migraine headache* is the only one that has a *throbbing headache with an aura* (the photophobia).

A patient has been receiving scheduled doses of *phenytoin (Dilantin)* and begins to experience *diplopia*. Which additional findings would the nurse expect? An aura or focal seizure Nystagmus or confusion Abdominal pain or cramping Irregular pulse or palpitations

Nystagmus or confusion *Diplopia* is a sign of *phenytoin toxicity*. The nurse should assess for other signs of toxicity, which include *neurologic changes, such as nystagmus, ataxia, confusion, dizziness, or slurred speech*.

C

The nurse determines that the patient is not experiencing adverse effects of albuterol (Proventil) after noting which patient vital sign? Temperature of 98.4°F Oxygen saturation 96% Pulse rate of 72 beats/min Respiratory rate of 18/ breaths/min

C

The nurse evaluates that nursing interventions to promote airway clearance in a patient admitted with chronic obstructive pulmonary disease (COPD) are successful based on which finding? Absence of dyspnea Improved mental status Effective and productive coughing PaO2 within normal range for the patient

C

The nurse is assigned to care for a patient in the emergency department admitted with an exacerbation of asthma. The patient has received a â-adrenergic bronchodilator and supplemental oxygen. If the patient's condition does not improve, the nurse should anticipate what as the most likely next step in treatment? IV fluids Biofeedback therapy Systemic corticosteroids Pulmonary function testing

C

During an assessment of a 45-yr-old patient with asthma, the nurse notes wheezing and dyspnea. The nurse interprets that these symptoms are related to what pathophysiologic change? Laryngospasm Pulmonary edema Narrowing of the airway Overdistention of the alveoli

B

When planning teaching for the patient with chronic obstructive pulmonary disease (COPD), the nurse understands that what causes the manifestations of the disease? An overproduction of the antiprotease a1-antitrypsin Hyperinflation of alveoli and destruction of alveolar walls Hypertrophy and hyperplasia of goblet cells in the bronchi Collapse and hypoventilation of the terminal respiratory unit

C

When teaching the patient with chronic obstructive pulmonary disease (COPD) about smoking cessation, what information should be included related to the effects of smoking on the lungs and the increased incidence of pulmonary infections? Smoking causes a hoarse voice. Cough will become nonproductive. Decreased alveolar macrophage function Sense of smell is decreased with smoking.

C

When teaching the patient with cystic fibrosis about the diet and medications, what is the priority information to be included in the discussion? Fat soluble vitamins and dietary salt should be avoided. Insulin may be needed with a diabetic diet if diabetes mellitus develops. Pancreatic enzymes and adequate fat, calories, protein, and vitamins are needed. Distal intestinal obstruction syndrome (DIOS) can be treated with increased water.

D

Which statement made by the patient with chronic obstructive pulmonary disease (COPD) indicates a need for further teaching regarding the use of an ipratropium inhaler? "I can rinse my mouth following the two puffs to get rid of the bad taste." "I should wait at least 1 to 2 minutes between each puff of the inhaler." "Because this medication is not fast acting, I cannot use it in an emergency if my breathing gets worse." "If my breathing gets worse, I should keep taking extra puffs of the inhaler until I can breathe more easily."

The nurse performs a detailed assessment of the abdomen of a patient with a possible bowel obstruction, knowing that manifestations of a obstruction in the large intestine are (select all that apply): a. persistent abdominal pain b. marked abdominal distention c. diarrhea that is loose or liquid d. colicky, severe, intermittent pain e. profuse committing that relieves abdominal pain

a & b Rationale: With lower intestinal obstructions, abdominal distention is markedly increased and pain is persistent. Onset of a large intestine obstruction is gradual, vomiting is rare, and there is usually absolute constipation, not diarrhea.

When a 35-year-old female patient is admitted to the emergency department with acute abdominal pain, which possible diagnosis should you consider that may be the cause of her pain (select all that apply)? a. Gastroenteritis b. Ectopic pregnancy c. GI bleeding d. Irritable bowel syndrome e. Inflammatory bowel disease

a, b, c, d & e Rationale: All these conditions could cause acute abdominal pain.

A pt who has an inhalation injury is receiving albuterol (Ventolin) for bronchospasm. What is the most important adverse effect of this medication for the nurse to manage? a. gi distress b. tachycardia c. restlessness d. hypokalemia

b. albuterol stimulates beta adrenergic receptors in the lungs to cause bronchodilation and is nonselective and also causes receptors in the heart to increase heart rate.

A patient is to undergo skin grafting with the use of cultured epithelial autografts full-thickness burns. The nurse explains to the patient that this treatment involves a) Shaving a split-thickness layer of the patient's skin to cover the burn wound. b) Using epidermal growth factor to cultivate cadaver skin for temporary wound coverage. c) Growing small specimens of the patient's skin into sheets to use as permanent skin coverage. d) Exposing animal skin to growth factors to decrease antigenicity so it can be used for permanent wound coverage.

c Rationale: Cultured epithelial autograft (CEA) is a method of obtaining permanent skin from a person with limited available skin for harvesting. CEA is grown from biopsy specimens obtained from the patient's own unburned skin.

18. The client with RA has nontender movable nodules in subcutaneous tissue over the elbows and shoulders. Which statement is the best explanation for the nodules? a. The nodules indicate a rapidly progressive destruction of the affected tissue b. The nodules are small amounts of synovial fluid that have become crystallized c. The nodules are lymph nodes that have proliferated to try to fight the disease d. The nodules present a favorable prognosis and mean the client is better

a. The nodules indicate a rapidly progressive destruction of the affected tissue

A patient who has *amyotrophic lateral sclerosis (ALS)* is hospitalized with *pneumonia*. Which nursing action will be included in the plan of care? a. Observe for agitation and paranoia. b. Assist with active range of motion (ROM). c. Give muscle relaxants as needed to reduce spasms. d. Use simple words and phrases to explain procedures.

b. Assist with active range of motion (ROM). *ALS* causes progressive *muscle weakness*, but assisting the patient to perform active *ROM* will help *maintain strength* as long as possible.

The nurse is caring for a client who sustained superficial partial thickness burns on the anterior lower legs and anterior thorax. Which of the following does the nurse expect to note during the resuscitation/emergent phase of the burn injury? a. decreased heart rate b. increased urinary output c. increased blood pressure d. elevated hematocrit levels

d. during the resuscitation/emergent phase, the hematocrit level increases to above normal because of hemoconcentration from the large fluid shift.

The patient is having an esophagoenterostomy with anastomosis of a segment of the colon to replace the resected portion. What initial postoperative care should the nurse expect when this patient returns to the nursing unit? A) Turn, deep breathe, cough, and use spirometer every 4 hours. B) Maintain an upright position for at least 2 hours after eating. C) NG will have bloody drainage, and it should not be repositioned. D) Keep in a supine position to prevent movement of the anastomosis.

C) The patient will have bloody drainage from the NG tube for 8 to 12 hours, and it should not be repositioned or reinserted without contacting the surgeon. Turning and deep breathing will be done every 2 hours, and the spirometer will be used more often than every 4 hours. Coughing would put too much pressure in the area and should not be done. Because the patient will have the NG tube, the patient will not be eating yet. The patient should be kept in a semi-Fowler's or Fowler's position, not supine, to prevent reflux and aspiration of secretions.

Following bowel resection, a patient has a nasogastric (NG) tube to suction, but complains of nausea and abdominal distention. The nurse irrigates the tube as necessary as ordered, but the irrigating fluid does not return. What should be the priority action by the nurse? A) Notify the physician. B) Auscultate for bowel sounds. C) Reposition the tube and check for placement. D) Remove the tube and replace it with a new one.

C) The tube may be resting against the stomach wall. The first action by the nurse (since this is intestinal surgery and not gastric surgery) is to reposition the tube and check it again for placement. The physician does not need to be notified unless the tube function cannot be restored by the nurse. The patient does not have bowel sounds, which is why the NG tube is in place. The NG tube would not be removed and replaced unless it was no longer in the stomach or the obstruction of the tube could not be relieved.

The patient with chronic gastritis is being put on a combination of medications to eradicate H. pylori. Which drugs does the nurse know will probably be used for this patient? A) Antibiotic(s), antacid, and corticosteroid B) Antibiotic(s), aspirin, and antiulcer/protectant C) Antibiotic(s), proton pump inhibitor, and bismuth D) Antibiotic(s) and nonsteroidal antiinflammatory drugs (NSAIDs)

C) To eradicate H. pylori, a combination of antibiotics, a proton pump inhibitor, and possibly bismuth (for quadruple therapy) will be used. Corticosteroids, aspirin, and NSAIDs are drugs that can cause gastritis and do not affect H. pylori.

The patient developed gout while hospitalized for a heart attack. When doing discharge teaching for this patient who takes aspirin for its antiplatelet effect, what should the nurse include about preventing future attacks of gout? A) Limit fluid intake. B) Administration of probenecid (Benemid) C) Administration of allopurinol (Zyloprim) D) Administration of nonsteroidal antiinflammatory drugs (NSAIDs)

C) To prevent future attacks of gout, the urate-lowering drug allopurinol may be administered. Increased fluid will be encouraged to prevent precipitation of uric acid in the renal tubules. This patient will not be able to take the uricosuric drug probenecid because the aspirin the patient must take will inactivate its effect, resulting in urate retention. NSAIDs for pain management will not be used, related to the aspirin, because of the potential for increased side effects.

Which clinical manifestations of inflammatory bowel disease are common to both patients with ulcerative colitis (UC) and Crohn's disease (select all that apply)? A) Restricted to rectum B) Strictures are common. C) Bloody, diarrhea stools D) Cramping abdominal pain E) Lesions penetrate intestine.

C, D) Clinical manifestations of UC and Crohn's disease include bloody diarrhea, cramping abdominal pain, and nutritional disorders. Intestinal lesions associated with UC are usually restricted to the rectum before moving into the colon. Lesions that penetrate the intestine or cause strictures are characteristic of Crohn's disease.

19. A heterosexual patient is concerned that they may contract human immunodeficiency virus (HIV) from a bisexual partner. What should the nurse include when teaching about preexposure prophylaxis (select all that apply.)? A. Take fluconazole (Diflucan). B. Take amphotericin B (Fungizone). C. Use condoms for risk-reducing sexual relations. D. Take emtricitabine and tenofovir (Truvada) regularly. E. Have regular HIV testing for herself and her husband.

C, D, E Using male or female condoms, having monthly HIV testing for the patient and partner, and taking emtricitabine and tenofovir regularly have shown to decrease the infection of heterosexual women having sex with a partner who participates in high-risk behavior. Fluconazole and amphotericin B are taken for Candida albicans, Coccidioides immitis, and Cryptococcus neoformans, which are all opportunistic diseases associate with HIV infection.

When teaching how lisinopril (Zestril) will help lower the patient's blood pressure, which mechanism of action should the nurse explain? A. Blocks β-adrenergic effects B. Relaxes arterial and venous smooth muscle C. Inhibits conversion of angiotensin I to angiotensin II D. Reduces sympathetic outflow from central nervous system

C. Lisinopril is an angiotensin-converting enzyme inhibitor that inhibits the conversion of angiotensin I to angiotensin II, which reduces angiotensin II-mediated vasoconstriction and sodium and water retention. β blockers result in vasodilation and decreased heart rate. Direct vasodilators relax arterial and venous smooth muscle. Central acting α-adrenergic antagonists reduce sympathetic outflow from the central nervous system to produce vasodilation and decreased systemic vascular resistance and blood pressure.

The nurse is caring for a patient admitted with chronic obstructive pulmonary disease (COPD), angina, and hypertension. Before administering the prescribed daily dose of atenolol 100 mg PO, the nurse assesses the patient carefully. Which adverse effect is this patient at risk for given the patient's health history? A. Hypocapnia B. Tachycardia C. Bronchospasm D. Nausea and vomiting

C. Atenolol is a cardioselective β1-adrenergic blocker that reduces blood pressure and could affect the β2-receptors in the lungs with larger doses or with drug accumulation. Although the risk of bronchospasm is less with cardioselective β-blockers than nonselective β-blockers, atenolol should be used cautiously in patients with COPD.

The nurse has reviewed proper body mechanics with a patient with a history of low back pain caused by a herniated lumbar disc. Which statement made by the patient indicates a need for further teaching? A. "I should sleep on my side or back with my hips and knees bent." B. "I should exercise at least 15 minutes every morning and evening." C. "I should pick up items by leaning forward without bending my knees." D. "I should try to keep one foot on a stool whenever I have to stand for a period of time."

C. "I should pick up items by leaning forward without bending my knees." The patient should avoid leaning forward without bending the knees. Bending the knees helps to prevent lower back strain and is part of proper body mechanics when lifting. Sleeping on the side or back with hips and knees bent and standing with a foot on a stool will decrease lower back strain. Back strengthening exercises are done twice a day once symptoms subside.

When caring for a patient with an electrical burn injury, the nurse should question a health care provider's order for A. Mannitol 75 gm IV. B. Urine for myoglobulin. C. Lactated Ringer's at 25 ml/hr. D. Sodium bicarbonate 24 mEq every 4 hours.

c. Electrical injury puts the patient at risk for myoglobinuria, which can lead to acute renal tubular necrosis (ATN). Treatment consists of infusing lactated Ringer's at a rate sufficient to maintain urinary output at 75 to 100 ml/hr. Mannitol can also be used to maintain urine output. Sodium bicarbonate may be given to alkalinize the urine. The urine would also be monitored for the presence of myoglobin. An infusion rate of 25 ml/hr is not sufficient to maintain adequate urine output in prevention and treatment of ATN.

While the nurse is transporting a patient on a stretcher to the radiology department, the patient begins having a tonic-clonic seizure. Which action should the nurse take? a. Insert an oral airway during the seizure to maintain a patent airway. b. Restrain the patient's arms and legs to prevent injury during the seizure. c. Time and observe and record the details of the seizure and postictal state. d. Avoid touching the patient to prevent further nervous system stimulation.

c. Time and observe and record the details of the seizure and postictal state. Because the diagnosis and treatment of seizures frequently are based on the description of the seizure, recording the length and details of the seizure is important.

1. The client diagnosed with OA is a resident in a long term care facility. The resident is refusing to bathe because she is hurting. Which instruction should the nurse give the unlicensed nursing assistant? a. Allow the client to stay in bed until the pain becomes bearable b. Tell the assistant to give the client a bed bath this morning c. Try to encourage the client to get up and go to the shower d. Notify the family that the client is refusing to be bathed

c. Try to encourage the client to get up and go to the shower

A child has just been admitted to the pediatric burn unit. Currently, the child is being evaluated for burns to his chest and upper legs. He complains of thirst and asks for a drink. What is the most appropriate nursing action? a. give a small glass of clear liquid b. give a small glass of a full liquid c. keep the child NPO d. order a pediatric meal tray with extra liquids

c. Until a complete assessment and treatment plan are initiated, the child should be kept NPO. A complication of major burns is paralytic ileus, so until that has been ruled out, oral fluids should not be provided.

The nurse manager is observing a new nursing graduate caring for a burn client in protective isolation. The nurse manager intervenes if the new nursing graduate planned to implement which incorrect component of protective isolation technique? a. using sterile sheets and linens b. performing strict hand washing technique c. wearing gloves and gown only when giving direct care to the client d. wearing protective garb, including a mask, gloves, cap, shoe covers, gowns, and plastic apron

c. ppe should be worn whenever entering the client's room

Fluid and electrolyte shifts that occur during the early emergent phase include a. adherence of albumin to vascular walls b. movement of potassium into the vascular space c. sequestering of sodium and water in the interstitial fluid. d. hemolysis of red blood cells from large volumes of rapidly administered fluid.

c. sequestering of sodium and water in the interstitial fluid.

The patient is having a gastroduodenostomy (Billroth I operation) for stomach cancer. What long-term complication is occurring when the patient reports generalized weakness, sweating, palpitations, and dizziness 15 to 30 minutes after eating? A) Malnutrition B) Bile reflux gastritis C) Dumping syndrome D) Postprandial hypoglycemia

C. After a Billroth I operation, dumping syndrome may occur 15 to 30 minutes after eating because of the hypertonic fluid going to the intestine and additional fluid being drawn into the bowel. Malnutrition may occur but does not cause these symptoms. Bile reflux gastritis cannot happen when the stomach has been removed. Postprandial hypoglycemia occurs with similar symptoms, but 2 hours after eating.

The nurse prepares to administer IV ibandronate (Boniva) to a 67-year-old woman with osteoporosis. What is a priority laboratory assessment to make before the administration of ibandronate? A. Serum calcium B. Serum creatinine C. Serum phosphate D. Serum alkaline phosphatase

C. Serum phosphate Ibandronate is a bisphosphonate that is administered IV every 3 months and is administered slowly over 15 to 30 seconds to prevent renal damage. Ibandronate should not be used by patients taking other nephrotoxic drugs or by those with severe renal impairment (defined as serum creatinine above 2.3 mg/dL or creatinine clearance less than 30 mL/min).

4. The nurse teaches the staff ensuring that standard precautions should be used when providing care for which type of patient? A. All patients regardless of diagnosis B. Pediatric and gerontologic patients C. Patients who are immunocompromised D. Patients with a history of infectious diseases

A Standard precautions are designed for all care of all patients in hospitals and health care facilities.

The nurse is caring for patients in a primary care clinic. Which individual is most at risk to develop osteomyelitis caused by Staphylococcus aureus? A. 22-year-old female with gonorrhea who is an IV drug user B. 48-year-old male with muscular dystrophy and acute bronchitis C. 32-year-old male with type 1 diabetes mellitus and a stage IV pressure ulcer D. 68-year-old female with hypertension who had a knee arthroplasty 3 years ago

C. 32-year-old male with type 1 diabetes mellitus and a stage IV pressure ulcer Osteomyelitis caused by Staphylococcus aureus is usually associated with a pressure ulcer or vascular insufficiency related to diabetes mellitus. Osteomyelitis caused by Staphylococcus epidermidis is usually associated with indwelling prosthetic devices such as joint replacements. Osteomyelitis caused by Neisseria gonorrhoeae is usually associated with gonorrhea. Osteomyelitis caused by Pseudomonas is usually associated with IV drug use. Muscular dystrophy is not associated with osteomyelitis.

The nurse assesses the right femoral artery puncture site as soon as the patient arrives after having a stent inserted into a coronary artery. The insertion site is not bleeding or discolored. What should the nurse do next to ensure the femoral artery is intact? A. Palpate the insertion site for induration. B. Assess peripheral pulses in the right leg. C. Inspect the patient's right side and back. D. Compare the color of the left and right legs.

C. Inspect the patient's right side and back. The best method to determine that the right femoral artery is intact after inspection of the insertion site is to logroll the patient to inspect the right side and back for retroperitoneal bleeding. The artery can be leaking and blood is drawn into the tissues by gravity. The peripheral pulses, color, and sensation of the right leg will be assessed per agency protocol.

The nurse determines that dietary teaching for a 75-year-old patient with osteoporosis has been successful when the patient selects which highest-calcium meal? A. Chicken stir-fry with 1 cup each onions and green peas, and 1 cup of steamed rice B. Ham and Swiss cheese sandwich on whole wheat bread, steamed broccoli, and an apple C. A sardine (3 oz) sandwich on whole wheat bread, 1 cup of fruit yogurt, and 1 cup of skim milk D. A two-egg omelet with 2 oz of American cheese, one slice of whole wheat toast, and a half grapefruit

C. A sardine (3 oz) sandwich on whole wheat bread, 1 cup of fruit yogurt, and 1 cup of skim milk The highest calcium content is present in the lunch containing milk and milk products (yogurt) and small fish with bones (sardines). Chicken, onions, green peas, rice, ham, whole wheat bread, broccoli, apple, eggs, and grapefruit each have less than 75 mg of calcium per 100 g of food. Swiss cheese and American cheese have more calcium, but not as much as the sardines, yogurt, and milk.

A patient had an open reduction repair of a bilateral nasal fracture. The nurse plans to implement an intervention that focuses on both nursing and medical goals for this patient. Which intervention should the nurse implement? A. Apply an external splint to the nose. B. Insert plastic nasal implant surgically. C. Humidify the air for mouth breathing. D. Maintain surgical packing in the nose.

D A goal that is common to nursing and medical management of a patient after rhinoplasty is to prevent the formation of a septal hematoma and potential infections resulting from a septal hematoma. Therefore, the nurse helps to keep the nasal packing in the nose. The packing applies direct pressure to oozing blood vessels to stop postoperative bleeding. A medical goal includes realigning the fracture with an external or internal splint. The nurse helps maintain the airway by humidifying inspired air because the nose is unable to do so following surgery because it is swollen and packed with gauze.

Physiological Integrity 6. Which statement made by a patient with coronary artery disease after the nurse has completed teaching about therapeutic lifestyle changes (TLC) diet indicates that further teaching is needed? a. "I will switch from whole milk to 1% milk." b. "I like salmon and I will plan to eat it more often." c. "I can have a glass of wine with dinner if I want one." d. "I will miss being able to eat peanut butter sandwiches."

D Although only 30% of the daily calories should come from fats, most of the fat in the TLC diet should come from monosaturated fats such as are found in nuts, olive oil, and canola oil. The patient can include peanut butter sandwiches as part of the TLC diet. The other patient comments indicate a good understanding of the TLC diet. DIF: Cognitive Level: Apply (application) REF: 738 TOP: Nursing Process: Evaluation MSC:

1. When developing a teaching plan for a 61-year-old man with the following risk factors for coronary artery disease (CAD), the nurse should focus on the a. family history of coronary artery disease. b. increased risk associated with the patient's gender. c. increased risk of cardiovascular disease as people age. d. elevation of the patient's low-density lipoprotein (LDL) level.

D Because family history, gender, and age are nonmodifiable risk factors, the nurse should focus on the patient's LDL level. Decreases in LDL will help reduce the patient's risk for developing CAD. DIF: Cognitive Level: Apply (application) REF: 736 TOP: Nursing Process: Planning MSC:

Physiological Integrity 11. Nadolol (Corgard) is prescribed for a patient with chronic stable angina and left ventricular dysfunction. To determine whether the drug is effective, the nurse will monitor for a. decreased blood pressure and heart rate. b. fewer complaints of having cold hands and feet. c. improvement in the strength of the distal pulses. d. the ability to do daily activities without chest pain.

D Because the medication is ordered to improve the patient's angina, effectiveness is indicated if the patient is able to accomplish daily activities without chest pain. Blood pressure and heart rate may decrease, but these data do not indicate that the goal of decreased angina has been met. The noncardioselective -adrenergic blockers can cause peripheral vasoconstriction, so the nurse would not expect an improvement in distal pulse quality or skin temperature. DIF: Cognitive Level: Apply (application) REF: 753 TOP: Nursing Process: Evaluation MSC:

The nurse teaches the patient with RA that one of the most effective methods of aerobic exercise is a. ballet dancing b. casual walking c. aquatic exercises d. low-impact aerobic exercises

C. aquatic exercises

During an acute exacerbation, a patient with SLE is treated with corticosteroids. The nurse would expect the steroids to begin to be tapered when serum laboratory results indicate a. increased RBCs b. decreased ESR c. decreased anti-DNA d. increased complement

C. decreased anti-DNA

A 70-year old patient is being evaluated for symptoms of RA. The nurse recognizes that a major problem in the management of RA in the older adult is that a. RA is usually more severe in older adults b. older patients are not as likely to comply with treatment regimens c. drug interactions and toxicity are more likely to occur with multidrug therapy d. laboratory and other diagnostic tests are not effective in identifying RA in older adults

C. drug interactions and toxicity are more likely to occur with multidrug therapy

A patient with OA uses NSAIDs to decrease pain and inflammation. The nurse teaches the patient that common side effects of these drugs include a. allergic reactions, fever, and oral lesions b. fluid retention, hypertension, and bruising c. skin rashes, gastric irritation, and headache d. prolonged bleeding time, blood dyscrasias, and hepatic damage

C. skin rashes, gastric irritation, and headacche

During the physical assessment of the patient with moderate RA, the nurse would expect to find a. hepatomegaly b. Heberden's nodes c. spindle-shaped fingers d. crepitus on joint movement

C. spindle-shaped fingers

The nurse is admitting a patient who is scheduled for knee arthroscopy related to osteoarthritis (OA). Which of the following findings would the nurse expect to be present on examination of the patient's knees? A. Ulnar drift B. Pain with joint movement C. Reddened, swollen affected joints D. Stiffness that increases with movement

Correct answer: B. Pain with joint movement Rationale: OA is characterized predominantly by joint pain upon movement and is a classic feature of the disease.

A female patient's complex symptomatology over the past year has culminated in a diagnosis of systemic lupus erythematosus (SLE). Which of the patient's following statements demonstrates the need for further teaching about the disease? A. "I'll try my best to stay out of the sun this summer." B. "I know that I probably have a high chance of getting arthritis." C. "I'm hoping that surgery will be an option for me in the future." D. "I understand that I'm going to be vulnerable to getting infections."

Correct answer: C. "I'm hoping that surgery will be an option for me in the future." Rationale: SLE carries an increased risk of infection, sun damage, and arthritis. Surgery is not a key treatment modality for SLE.

The nurse is assessing the recent health history of a 63-year-old patient with osteoarthritis (OA). The nurse determines that the patient is trying to manage the condition appropriately when the patient describes which of the following activity patterns? A. Bed rest with bathroom privileges B. Daily high-impact aerobic exercise C.A regular exercise program of walking D. Frequent rest periods with minimal exercise

Correct answer: C. A regular exercise program of walking Rationale: A regular low-impact exercise, such as walking, is important in helping to maintain joint mobility in the patient with osteoarthritis.

Which of the following patient statements most clearly suggests a need to assess the patient for ankylosing spondylitis (AS)? A. "My right elbow has become red and swollen over the last few days." B. "I wake up stiff every morning and my knees just don't want to bend." C. "My husband tells me that my posture has become so stooped this winter." D. "My lower back pain seems to be getting worse all the time and nothing seems to help."

Correct answer: D. "My lower back pain seems to be getting worse all the time and nothing seems to help." Rationale: AS primarily affects the axial skeleton. Based on this, symptoms of inflammatory spine pain are often the first clues to a diagnosis of AS. Knee or elbow involvement is not consistent with the typical course of AS. Back pain is likely to precede the development of kyphosis.

A

The nurse identifies the nursing diagnosis of activity intolerance for a patient with asthma. In patients with asthma, the nurse assesses for which etiologic factor for this nursing diagnosis? Work of breathing Fear of suffocation Effects of medications Anxiety and restlessness

C

The nurse is assigned to care for a patient who has anxiety and an exacerbation of asthma. What is the primary reason for the nurse to carefully inspect the chest wall of this patient? Allow time to calm the patient. Observe for signs of diaphoresis. Evaluate the use of intercostal muscles. Monitor the patient for bilateral chest expansion.

D

The nurse is assisting a patient to learn self-administration of beclomethasone, two puffs inhaled every 6 hours. What should the nurse explain as the best way to prevent oral infection while taking this medication? Chew a hard candy before the first puff of medication. Rinse the mouth with water before each puff of medication. Ask for a breath mint after the second puff of medication. Rinse the mouth with water after the second puff of medication.

C

The nurse is caring for a 48-yr-old male patient admitted for exacerbation of chronic obstructive pulmonary disease. The patient develops severe dyspnea at rest, with a change in respiratory rate from 26 breaths/min to 44 breaths/min. Which action by the nurse would be the most appropriate? Have the patient perform huff coughing. Perform chest physiotherapy for 5 minutes. Teach the patient to use pursed-lip breathing. Instruct the patient in diaphragmatic breathing.

B

The nurse, who has administered a first dose of oral prednisone to a patient with asthma, writes on the care plan to begin monitoring for which patient parameters? Apical pulse Daily weight Bowel sounds Deep tendon reflexes

A

The patient has an order for each of the following inhalers. Which one should the nurse offer to the patient at the onset of an asthma attack? Albuterol Ipratropium bromide Salmeterol (Serevent) Beclomethasone (Qvar)

A patient who is admitted to a burn unit is hypovolemic. A new nurse asks an experienced nurse about the patient's condition. Which response if made by the experienced nurse is most appropriate? a) "Blood loss from burned tissue is the most likely cause of hypovolemia." b) "Third spacing of fluid into fluid-filled vesicles is usually the cause of hypovolemia." c) "The usual cause of hypovolemia is vaporation of fluid from denuded body surfaces." d) "Increased capillary permeability causes fluid shifts out of blood vessels and results in hypovolemia."

d Rationale: Hypovolemic shock is caused by a massive shift of fluids out of the blood vessels as a result of increased capillary permeability. Water, sodium, and plasma proteins move into interstitial spaces and other surrounding tissue.

2. In evaluating an asthmatic patient's knowledge of self-care, the nurse recognizes that additional instruction is needed when the patient says, a. "I use my corticosteroid inhaler when I feel short of breath." b. "I get a flu shot every year and see my HCP if I have an upper respiratory tract infection." c. "I use my inhaler before I visit my aunt who has a cat, but I only visit for a few minutes because of my allergies." d. "I walk 30 minutes every day but sometimes I have to use my bronchodilator inhaler before walking to prevent me from getting short of breath."

a. "I use my corticosteroid inhaler when I feel short of breath."

When evaluating the patient's understanding about the care of the ileostomy, what statement by the patient indicates the patient needs more teaching? a. "I will be able to regulate when I have stools." b. "I will be able to wear the pouch until it leaks." c. "Dried fruit and popcorn must be chewed very well." d. "The drainage from my stoma can damage my skin."

a. "I will be able to regulate when I have stools." The ileostomy is in the ileum and drains liquid stool frequently, unlike the colostomy which has more formed stool the further distal the ostomy is in the colon. The ileostomy pouch is usually worn 4-7 days or until it leaks. It must be changed immediately if it leaks because the drainage is very irritating to the skin. To avoid obstruction, popcorn, dried fruit, coconut, mushrooms, olives, stringy vegetables, food with skin, and meats with casings must be chewed extremely well before swallowing because of the narrow diameter of the ileostomy lumen.

A woman who has multiple sclerosis (MS) asks the nurse about risks associated with pregnancy. Which response by the nurse is accurate? a. "MS symptoms may be worse after the pregnancy." b. "Women with MS frequently have premature labor." c. "MS is associated with an increased risk for congenital defects." d. "Symptoms of MS are likely to become worse during pregnancy."

a. "MS symptoms may be worse after the pregnancy." During the postpartum period, women with MS are at greater risk for exacerbation of symptoms. There is no increased risk for congenital defects in infants born of mothers with MS. Symptoms of MS may improve during pregnancy. Onset of labor is not affected by MS.

A client's burn is infected and mafenide (Sulfamylon) is prescribed. The nurse's knowledge about this medication would indicate that which organism is involved? a. pseudomonas aeruginosa b. tubercle bacillus c. Methicillin resistant staphylococcus aureus (MRSA) d. Candida albicans

a. Mafenide is useful in treatment of partial and full thickness burns to prevent septicemia caused by organisms suche as pseudomonas aeruginosa.

The nurse is caring for a 68-year-old patient admitted with abdominal pain, nausea, and vomiting. The patient has an abdominal mass, and a bowel obstruction is suspected. The nurse auscultating the abdomen listens for which type of bowel sounds that are consistent with the patient's clinical picture? a. Low-pitched and rumbling above the area of obstruction b. High-pitched and hypoactive below the area of obstruction c. Low-pitched and hyperactive below the area of obstruction d. High-pitched and hyperactive above the area of obstruction

d. High-pitched and hyperactive above the area of obstruction Early in intestinal obstruction, the patient's bowel sounds are hyperactive and high-pitched, sometimes referred to as "tinkling" above the level of the obstruction. This occurs because peristaltic action increases to "push past" the area of obstruction. As the obstruction becomes complete, bowel sounds decrease and finally become absent.

A patient is being treated with *carbidopa/levodopa (Sinemet)* for *Parkinson's disease*. Which information indicates a *need for change* in the medication or dosage? a. Shuffling gait c. Cogwheel rigidity of limbs b. Tremor at rest d. Uncontrolled head movement

d. Uncontrolled head movement *Dyskinesia* is an *adverse effect* of the *Sinemet*, indicating a need for a change in medication or decrease in dose. The other findings are typical with Parkinson's disease.

A nursing intervention that is most appropriate to decrease post-operative edema and pain after an inguinal herniorrhaphy is: a. applying a truss to the hernia site b. allowing the patient to stand to void c. supporting the incision during coughing d. applying a scrotal support with ice bag

d. applying a scrotal support with ice bag Rationale: Scrotal edema is a painful complication after an inguinal hernia repair. Scrotal support with application of an ice bag may help relieve pain and edema.

A male pt suffered full thickness burns to the chest and back and the nurse notes the pressure alarm on his mechanical ventilator is sounding every 5 minutes. What is the most relevant assessment to prevent respiratory complications in this pt? a. pH b. PaCO2 c. Breath sounds d. chest expansion

d. assessing the pt's chest expansion is initially the most important because a sever burn that includes the anterior and posterior thorax can restrict chest expansion from eschar or scar tissue.

A 61-year-old patient with suspected bowel obstruction had a nasogastric tube inserted at 4:00 AM. The nurse shares in the morning report that the day shift staff should check the tube for patency at what times? a. 7:00 AM, 10:00 AM, and 1:00 PM b. 8:00 AM, 12:00 PM, and 4:00 PM c. 9:00 AM and 3:00 PM d. 9:00 AM, 12:00 PM, and 3:00 PM

b. 8:00 AM, 12:00 PM, and 4:00 PM A nasogastric tube should be checked for patency routinely at 4-hour intervals. Thus if the tube were inserted at 4:00 AM, it would be due to be checked at 8:00 AM, 12:00 PM, and 4:00 PM.

19. The client diagnosed with RA who has been prescribed Plaquenil, shows marked improvement. Which instruction regarding the use of this medication should the nurse teach? a. Explain that the less medication loses its efficacy after a few months b. Continue to have regular eye exams while taking the medication c. Have yearly MRIs to follow the progress d. Discuss that the drug is taken for 3 weeks and then stopped for a week

b. Continue to have regular eye exams while taking the medication

6. Which client goal would be most appropriate for a client diagnosed with OA? a. Perform passive range-of-motion exercises b. Maintain optimal functional ability c. Client will walk three miles a day d. Client will join a health club

b. Maintain optimal functional ability

5. During a follow-up visit to a patient with acute osteomyelitis treated with IV antibiotics, the home health nurse is told by the patient's wife that she can hardly get the patient to eat because his mouth is so sore. In assessing the patient's mouth, what should the nurse expect to find? a. A dry, cracked tongue with a central furrow b. White, curdlike membranous lesions of the mucosa c. Ulcers of the mouth and lips surrounded by a reddened base d. Single or clustered vesicles on the tongue and buccal mucosa

b. One of the most common adverse effects of prolonged and high-dose antibiotic therapy is overgrowth of Candida albicans in the oral cavity and genitourinary tract. These infections are manifested by whitish-yellow, curdlike lesions of the mucosa. A dry, cracked, furrowed tongue is characteristic of severe dehydration and vesicles are characteristic of herpes simplex infections. Mouth and lip ulcers are characteristic of aphthous somatitis, or canker sores.

The home health registered nurse (RN) is planning care for a patient with a seizure disorder related to a recent head injury. Which nursing action can be delegated to a licensed practical/vocational nurse (LPN/LVN)? a. Make referrals to appropriate community agencies. b. Place medications in the home medication organizer. c. Teach the patient and family how to manage seizures. d. Assess for use of medications that may precipitate seizures.

b. Place medications in the home medication organizer.

A hospitalized patient with a history of cluster headache awakens during the night with a severe stabbing headache. Which action should the nurse take first? a. Put a moist hot pack on the patient's neck. b. Start the prescribed PRN O2 at 6 L/min. c. Give the ordered PRN acetaminophen (Tylenol). d. Notify the patient's health care provider immediately.

b. Start the prescribed PRN O2 at 6 L/min. *Acute treatment* for *cluster headache* is administration of *100% O2 at 6 to 8 L/min*. If the patient obtains relief with the O2, there is no immediate need to notify the health care provider. *Cluster headaches last only 60 to 90 minutes*, so oral pain medications have minimal effect. Hot packs are helpful for tension headaches but are not as likely to reduce pain associated with a cluster headache.

Lewis In assessing the joints of a patient with osteoarthritis, the nurse understands that Heberden's nodes a. are often red, swollen, and tender. b. indicate osteophyte formation at the DIP joints. c. are the result of pannus formation at the PIP joints. d. occur from deterioration of cartilage by proteolytic enzymes.

b. indicate osteophyte formation at the DIP joints.

The nurse should expect to apply which type of ordered antiseptic to a client with a burn wound, once the area has been cleansed with sterile saline? a. copper containing b. silver containing c. biguanide d. acetic acid

b. silver sulfadiazine is a metallic type of antiseptic that is widely used on burns. The silver in the solution is toxic to bacteria, and prevents them from reproducing.

The nurse plans care for a male pt who suffered thermal burns to the entire posterior aspect of his body when he fell on an outdoor grill. Which pt need is likely to be the primary problem of this pt in the emergent phase? a. maintain tissue oxygenation b. halt progression of the burn c. maintain intravascular volume d. prevent invasion of pathogens

b. the first priority is halting the severity of the burn, to limit the depth of the burn and quick action must be a priority.

The nurse expects the assessment of a patient who is experiencing a cluster headache to include a. nuchal rigidity. c. projectile vomiting. b. unilateral ptosis. d. throbbing, bilateral facial pain.

b. unilateral ptosis. Unilateral eye edema, tearing, and ptosis are characteristic of cluster headaches. Nuchal rigidity suggests meningeal irritation, such as occurs with meningitis.

You are teaching a patient with osteopenia. What is important to include in the teaching plan? a.Lose weight. b.Stop smoking. c.Eat a high-protein diet. d.Start swimming for exercise

b.Stop smoking. Patients with osteopenia should be instructed to quit smoking in order to decrease loss of bone mass.

Knowing the most common causes of household fires, which of the following prevention strategies would the nurse focus on when teaching about fire safety? a. set how water temp at 140 F b. use only hardwired smoke detectors c. encourage regular home fire exit drills d. never permit older adults to cook unattended.

c.

14. Which client problem is priority for a client diagnosed with RA? a. Activity intolerance b. Fluid and Electrolyte balance c. Alteration in comfort d. Excessive nutritional intake

c. Alteration in comfort

A

A male patient with chronic obstructive pulmonary disease (COPD) becomes dyspneic at rest. His baseline blood gas results are PaO2 70 mm Hg, PaCO2 52 mm Hg, and pH 7.34. What updated patient assessment requires the nurse's priority intervention? Arterial pH 7.26 PaCO2 50 mm Hg Patient in tripod position Increased sputum expectoration

The pathophysiology of systemic lupus erthematosus (SLE) is characterized by a. destruction of nucleic acids and other self-proteins by autoantibodies b. overproduction of collagen that disrupts the functioning of internal organs c. formation of abnormal IgG that attaches to cellular antigens, activating complement d. increased activity of T-suppressor cells with B-cell hypoactivity, resulting in an immunodeficiency

A. destruction of nucleic acids and other self-proteins by autoantibodies

Teaching that the nurse will plan for the patient with SLE includes a. ways to avoid exposure to sunlight b. increasing dietary protein and carbohydrate intake c. the necessity of genetic counseling before planning a family d. the use of no pharmacologic pain interventions instead of analgesics

A. ways to avoid exposure to sunlight

A patient with Bell's palsy refuses to eat while others are present because of embarrassment about drooling. The best response by the nurse is to a. respect the patient's feelings and arrange for privacy at mealtimes. b. teach the patient to chew food on the unaffected side of the mouth. c. offer the patient liquid nutritional supplements at frequent intervals. d. discuss the patient's concerns with visitors who arrive at mealtimes.

ANS: A The patient's desire for privacy should be respected to encourage adequate nutrition and reduce patient embarrassment. Liquid supplements may help maintain nutrition but will reduce the patient's enjoyment of the taste of food. It would be inappropriate for the nurse to discuss the patient's embarrassment with visitors unless the patient wishes to share this information. Chewing on the unaffected side of the mouth will enhance nutrition and enjoyment of food but will not decrease the drooling. DIF: Cognitive Level: Analyze (analysis)

A 63-year-old patient hospitalized with polymyositis has joint pain, an erythematosus facial rash, eyelid edema, and a weak, hoarse voice. The priority nursing diagnosis for the patient is a. risk for aspiration related to dysphagia. b. disturbed visual perception related to swelling. c. acute pain related to generalized inflammation. d. risk for impaired skin integrity related to scratching.

ANS: A The patient's vocal weakness and hoarseness indicate weakness of the pharyngeal muscles and a high risk for aspiration. The other nursing diagnoses also are appropriate but are not as high a priority as the maintenance of the patient's airway

The nurse is caring for a patient who has been experiencing stroke symptoms for 60 minutes. Which action can the nurse delegate to a licensed practical/vocational nurse (LPN/LVN)? a. Assess the patient's gag and cough reflexes. b. Determine when the stroke symptoms began. c. Administer the prescribed short-acting insulin. d. Infuse the prescribed IV metoprolol (Lopressor).

ANS: C Administration of subcutaneous medications is included in LPN/LVN education and scope of practice. The other actions require more education and scope of practice and should be done by the registered nurse (RN). DIF: Cognitive Level: Apply (application) REF: 1359 OBJ: Special Questions: Delegation TOP: Nursing Process: Planning MSC: NCLEX: Safe and Effective Care Environment

A patient who had a C7 spinal cord injury 1 week ago has a weak cough effort and crackles. The initial intervention by the nurse should be to a. suction the patient's nasopharynx. b. notify the patient's health care provider. c. push upward on the epigastric area as the patient coughs. d. encourage incentive spirometry every 2 hours during the day.

ANS: C Because the cough effort is poor, the initial action should be to use assisted coughing techniques to improve the patient's ability to mobilize secretions. The use of the spirometer may improve respiratory status, but the patient's ability to take deep breaths is limited by the loss of intercostal muscle function. Suctioning may be needed if the patient is unable to expel secretions by coughing but should not be the nurse's first action. The health care provider should be notified if airway clearance interventions are not effective or additional collaborative interventions are needed. DIF: Cognitive Level: Analyze (analysis)

A patient whose tracheostomy was inserted 30 minutes ago is recovering in the postanesthesia recovery unit when the tracheostomy tube is expelled by coughing. What is the priority action by the nurse? A. Suction the tracheostomy opening. B. Maintain the airway with a sterile hemostat. C. Use an Ambu bag and mask to ventilate the patient. D. Insert the tracheostomy tube obturator into the stoma.

B As long as the patient is not in acute respiratory distress after dislodging the tracheostomy tube, the nurse should use a sterile hemostat to maintain an open airway until a sterile tracheostomy tube can be reinserted into the tracheal opening. The tracheostomy is an open surgical wound that has not had time to mature into a stoma. If the patient is in respiratory distress, the nurse will use an Ambu bag and mask to ventilate the patient temporarily.

A patient who is given a bisacodyl (Dulcolax) suppository asks the nurse how long it will take to work. The nurse replies that the patient will probably need to use the bedpan or commode within which time frame after administration? A) 2-5 minutes B) 15-60 minutes C) 2-4 hours D) 6-8 hours

B) Bisacodyl suppositories usually are effective within 15 to 60 minutes of administration, so the nurse should plan accordingly to assist the patient to use the bedpan or commode.

The nurse is caring for a patient treated with intravenous fluid therapy for severe vomiting. As the pt recovers and begins to tolerate oral intake, the N understands that which of the following food choices would be most appropriate? A) Ice tea B) Dry toast C) Warm broth D) Plain hamburger

B) Dry toast (Dry toast or crackers may alleviate the feeling of nausea and prevent further vomiting. Extremely hot or cold liquids and fatty foods are generally not well tolerated.)

Following administration of a dose of metoclopramide (Reglan) to the patient, the nurse determines that the medication has been effective when what is noted? A) Decreased blood pressure B) Absence of muscle tremors C) Relief of nausea and vomiting D) No further episodes of diarrhea

C) Metoclopramide is classified as a prokinetic and antiemetic medication. If it is effective, the patient's nausea and vomiting should resolve. Metoclopramide does not affect blood pressure, muscle tremors, or diarrhea.

Postoperative care of a patient undergoing coronary artery bypass graft (CABG) surgery includes monitoring for what common complication? A. Dehydration B. Paralytic ileus C. Atrial dysrhythmias D. Acute respiratory distress syndrome

C. Atrial dysrhythmias Postoperative dysrhythmias, specifically atrial dysrhythmias, are common in the first 3 days following CABG surgery. Although the other complications could occur, they are not common complications.

Assessment data in the patient with osteoarthritis commonly include a. gradual weight loss b. elevated WBC count c. joint pain that worsens with use d. straw-colored synovial fluid

Correct answer: c Rationale: Osteoarthritis pain ranges from mild discomfort to significant disability. Joint pain is the predominant symptom, and the pain generally worsens with joint use.

For which problem is percutaneous coronary intervention (PCI) most clearly indicated? A. Chronic stable angina B. Left-sided heart failure C. Coronary artery disease D. Acute myocardial infarction

D. Acute myocardial infarction PCI is indicated to restore coronary perfusion in cases of myocardial infarction. Chronic stable angina and CAD are normally treated with more conservative measures initially. PCI is not relevant to the pathophysiology of heart failure, such as left-sided heart failure.

A left-handed patient with left-sided hemiplegia has difficulty feeding himself. Which intervention should the nurse include in the plan of care? a. Provide a wide variety of food choices. b. Provide oral care before and after meals. c. Assist the patient to eat with the right hand. d. Teach the patient the "chin-tuck" technique.

ANS: C Because the patient has difficulty feeding himself, the appropriate interventions will focus on teaching the patient to use the right hand for self-feeding. The other interventions are appropriate for patients with other etiologies for the imbalanced nutrition. DIF: Cognitive Level: Apply (application) REF: 1361 TOP: Nursing Process: Planning MSC: NCLEX: Physiological Integrity

Which nursing diagnosis is a priority in the care of a patient with myasthenia gravis (MG)? Acute confusion Bowel incontinence Activity intolerance Disturbed sleep pattern

Activity intolerance

A patient has experienced burns to his upper thighs and knees. Following the application of new wound dressings, the nurse should perform what nursing action? A) Instruct the patient to keep the wound site in a dependent position. B) Administer PRN analgesia as ordered. C) Assess the patient's peripheral pulses distal to the dressing. D) Assist with passive range of motion exercises to ìsetî the new dressing.

Ans: C Feedback: Dressings can impede circulation if they are wrapped too tightly. The peripheral pulses must be checked frequently and burned extremities elevated. Dependent positioning does not need to be maintained. PRN analgesics should be administered prior to the dressing change. ROM exercises do not normally follow a dressing change.

A 20-year-old patient with a history of muscular dystrophy is hospitalized with a respiratory tract infection. Which nursing action will be included in the plan of care? a. Assist the patient with ambulation. b. Logroll the patient every 1 to 2 hours. c. Discuss the need for genetic testing with the patient. d. Teach the patient about the muscle biopsy procedure.

ANS: A Since the goal for the patient with muscular dystrophy is to keep the patient active for as long as possible, assisting the patient to ambulate will be part of the care plan. The patient will not require logrolling. Muscle biopsies are necessary to confirm the diagnosis but are not necessary for a patient who already has a diagnosis. There is no need for genetic testing since the patient already knows the diagnosis

When teaching about clopidogrel (Plavix), the nurse will tell the patient with cerebral atherosclerosis a. to monitor and record the blood pressure daily. b. to call the health care provider if stools are tarry. c. that clopidogrel will dissolve clots in the cerebral arteries. d. that clopidogrel will reduce cerebral artery plaque formation.

ANS: B Clopidogrel inhibits platelet function and increases the risk for gastrointestinal bleeding, so patients should be advised to notify the health care provider about any signs of bleeding. The medication does not lower blood pressure, decrease plaque formation, or dissolve clots. DIF: Cognitive Level: Apply (application) REF: 1353 TOP: Nursing Process: Implementation MSC: NCLEX: Physiological Integrity

1. The nurse is caring for a patient newly diagnosed with human immunodeficiency virus (HIV). What does the nurse explain to the patient the criteria for diagnosis is based on? A. Presence of HIV antibodies B. CD4+ T cell count below 200/µL C. Presence of oral hairy leukoplakia D. White blood cell count below 5000/µL

B Diagnostic criteria for AIDS include a CD4+ T-cell count below 200/µL or the development of specified opportunistic infections, cancers, wasting syndrome, or dementia. The other options may be found in patients with HIV disease but do not define the advancement of HIV infection to AIDS.

A nurse is caring for a patient with burns who is in the later stages of the acute phase of recovery. The plan of nursing care should include which of the following nursing actions? A) Maintenance of bed rest to aid healing B) Choosing appropriate splints and functional devices C) Administration of beta adrenergic blockers D) Prevention of venous thromboembolism

Ans: D Feedback: Prevention of deep vein thrombosis (DVT) is an important factor in care. Early mobilization of the patient is important. The nurse monitors the splints and functional devices, but these are selected by occupational and physical therapists. The hemodynamic changes accompanying burns do not normally require the use of beta blockers.

Physiological Integrity 10. The nurse will suspect that the patient with stable angina is experiencing a side effect of the prescribed metoprolol (Lopressor) if the a. patient is restless and agitated. b. blood pressure is 90/54 mm Hg. c. patient complains about feeling anxious. d. cardiac monitor shows a heart rate of 61 beats/minute.

B Patients taking -adrenergic blockers should be monitored for hypotension and bradycardia. Because this class of medication inhibits the sympathetic nervous system, restlessness, agitation, hypertension, and anxiety will not be side effects. DIF: Cognitive Level: Apply (application) REF: 745 TOP: Nursing Process: Evaluation MSC:

Physiological Integrity 28. Which assessment finding by the nurse caring for a patient who has had coronary artery bypass grafting using a right radial artery graft is most important to communicate to the health care provider? a. Complaints of incisional chest pain b. Pallor and weakness of the right hand c. Fine crackles heard at both lung bases d. Redness on both sides of the sternal incision

B The changes in the right hand indicate compromised blood flow, which requires immediate evaluation and actions such as prescribed calcium channel blockers or surgery. The other changes are expected and/or require nursing interventions. DIF: Cognitive Level: Apply (application) REF: 752 OBJ: Special Questions: Prioritization TOP: Nursing Process: Assessment MSC:

The nurse teaches a patient about the use of budesonide intranasal spray for seasonal allergic rhinitis. The nurse determines that medication teaching is successful if the patient makes which statement? A. "My liver function will be checked with blood tests every 2 to 3 months." B. "The medication will decrease the congestion within 3 to 5 minutes after use." C. "I may develop a serious infection because the medication reduces my immunity." D. "I will use the medication every day of the season whether I have symptoms or not."

D Budesonide should be started 2 weeks before pollen season starts and used on a regular basis, not as needed. The spray acts to decrease inflammation and the effect is not immediate as with decongestant sprays. At recommended doses, budesonide has only local effects and will not result in immunosuppression or a systemic infection. Zafirlukast (Accolate) is a leukotriene receptor antagonist and may alter liver function tests (LFTs). LFTs must be monitored periodically in the patient taking zafirlukast.

A patient with a history of tonsillitis reports difficulty breathing. Which patient assessment data warrants emergency interventions by the nurse? A. Bilateral erythema of especially large tonsils B. Temperature 102.2°F, diaphoresis, and chills C. Contraction of neck muscles during inspiration D. β-hemolytic streptococcus in the throat culture

C Contraction of neck muscles during inspiration indicates that the patient is using accessory muscles for breathing and is in serious respiratory distress. The reddened and enlarged tonsils indicate pharyngitis. The increased temperature, diaphoresis, and chills indicate an infection, which could be β-hemolytic streptococcus or fungal infection, but not an emergency situation for the patient.

A nurse is working with a 73-year-old patient with osteoarthritis (OA). In assessing the patient's understanding of this disorder, the nurse concludes teaching has been effective when the patient uses which description of the condition? A) Joint destruction caused by an autoimmune process B) Degeneration of articular cartilage in synovial joints C) Overproduction of synovial fluid resulting in joint destruction D) Breakdown of tissue in non-weight-bearing joints by enzymes

B) OA is a degeneration of the articular cartilage in diarthrodial (synovial) joints from damage to the cartilage. The condition has also been referred to as degenerative joint disease. OA is not an autoimmune disease. There is no overproduction of synovial fluid causing destruction or breakdown of tissue by enzymes.

Physiological Integrity 5. After the nurse has finished teaching a patient about the use of sublingual nitroglycerin (Nitrostat), which patient statement indicates that the teaching has been effective? a. "I can expect some nausea as a side effect of nitroglycerin." b. "I should only take the nitroglycerin if I start to have chest pain." c. "I will call an ambulance if I still have pain after taking 3 nitroglycerin 5 minutes apart." d. "Nitroglycerin helps prevent a clot from forming and blocking blood flow to my heart."

C The emergency medical services (EMS) system should be activated when chest pain or other symptoms are not completely relieved after 3 sublingual nitroglycerin tablets taken 5 minutes apart. Nitroglycerin can be taken to prevent chest pain or other symptoms from developing (e.g., before intercourse). Gastric upset (e.g., nausea) is not an expected side effect of nitroglycerin. Nitroglycerin does not impact the underlying pathophysiology of coronary artery atherosclerosis. DIF: Cognitive Level: Apply (application) REF: 743 TOP: Nursing Process: Evaluation MSC:

The nurse is providing teaching to a patient recovering from an MI. How should resumption of sexual activity be discussed? A. Delegated to the primary care provider B. Discussed along with other physical activities C. Avoided because it is embarrassing to the patient D. Accomplished by providing the patient with written material

B. Discussed along with other physical activities Although some nurses may not feel comfortable discussing sexual activity with patients, it is a necessary component of patient teaching. It is helpful to consider sex as a physical activity and to discuss or explore feelings in this area when other physical activities are discussed. Although providing the patient with written material is appropriate, it should not replace a verbal dialogue that can address the individual patient's questions and concerns.

The patient has decided to use the voice rehabilitation that offers the best speech quality even though it must be cleaned regularly. The nurse knows that this is what kind of voice rehabilitation? A. Electromyography B. Intraoral electrolarynx C. Neck type electrolarynx D. Transesophageal puncture

D The transesophageal puncture provides a fistula between the esophagus and trachea with a one-way valved prosthesis to prevent aspiration from the esophagus to the trachea. Air moves from the lungs and vibrates against the esophagus, and words are formed with the tongue and lips as the air moves out the mouth. The electromyography and both electrolarynx methods produce low-pitched mechanical sounds.

Physiological Integrity 38. After receiving change-of-shift report about the following four patients, which patient should the nurse assess first? a. 39-year-old with pericarditis who is complaining of sharp, stabbing chest pain b. 56-year-old with variant angina who is to receive a dose of nifedipine (Procardia) c. 65-year-old who had a myocardial infarction (MI) 4 days ago and is anxious about the planned discharge d. 59-year-old with unstable angina who has just returned to the unit after having a percutaneous coronary intervention (PCI)

D This patient is at risk for bleeding from the arterial access site for the PCI, so the nurse should assess the patient's blood pressure, pulse, and the access site immediately. The other patients should also be assessed as quickly as possible, but assessment of this patient has the highest priority. DIF: Cognitive Level: Analyze (analysis) REF: 751 OBJ: Special Questions: Prioritization; Multiple Patients TOP: Nursing Process: Assessment MSC:

A patient reports feeling numbness and tingling of the left arm before experiencing a seizure. The nurse determines that this history is consistent with what type of seizure? a. Focal c. Absence b. Atonic d. Myoclonic

a. Focal The initial symptoms of a focal seizure involve clinical manifestations that are localized to a particular part of the body or brain.

During the emergent phase of burn injury, the nurse assesses for the presence of hypovolemia. In burn patients, hypovolemia occurs primarily as a result of a. Blood loss from injured tissue. b. Third spacing of fluid into fluid-filled vesicles. c. Evaporation of fluid from denuded body surfaces. d. Capillary permeability with fluid shift to the interstitium.

d Rationale: Hypovolemic shock is caused by a massive shift of fluids out of the blood vessels as a result of increased capillary permeability. Water, sodium, and plasma proteins move into interstitial spaces and other surrounding tissue.

A 48-yr-old man was just diagnosed with Huntington's disease. His 20-yr-old son is upset about his father's diagnosis. What is the nurse's best response? Provide emotional and psychologic support. Encourage him to get diagnostic genetic testing. Explain that cognitive deterioration will be treated with counseling. Instruct that chorea and psychiatric disorders can be treated with haloperidol (Haldol).

Provide emotional and psychologic support. The patient's son will first need emotional and psychologic support. He should be taught about diagnostic genetic testing but should decide for himself with a genetic counselor if and when he wants this done

A male patient with a diagnosis of Parkinson's disease (PD) is admitted to a long-term care facility. Which action should the health care team take to promote adequate nutrition for this patient? Provide multivitamins with each meal. Provide a diet that is low in complex carbohydrates and high in protein. Provide small, frequent meals throughout the day that are easy to chew and swallow. Provide the patient with a minced or pureed diet that is high in potassium and low in sodium.

Provide small, frequent meals throughout the day that are easy to chew and swallow.

Physiological Integrity 4. Which information given by a patient admitted with chronic stable angina will help the nurse confirm this diagnosis? a. The patient states that the pain "wakes me up at night." b. The patient rates the pain at a level 3 to 5 (0 to 10 scale). c. The patient states that the pain has increased in frequency over the last week. d. The patient states that the pain "goes away" with one sublingual nitroglycerin tablet.

D Chronic stable angina is typically relieved by rest or nitroglycerin administration. The level of pain is not a consistent indicator of the type of angina. Pain occurring at rest or with increased frequency is typical of unstable angina. DIF: Cognitive Level: Understand (comprehension) REF: 742 TOP: Nursing Process: Assessment MSC:

Physiological Integrity 36. A patient had a non-ST-segment-elevation myocardial infarction (NSTEMI) 3 days ago. Which nursing intervention included in the plan of care is most appropriate for the registered nurse (RN) to delegate to an experienced licensed practical/vocational nurse (LPN/LVN)? a. Evaluation of the patient's response to walking in the hallway b. Completion of the referral form for a home health nurse follow-up c. Education of the patient about the pathophysiology of heart disease d. Reinforcement of teaching about the purpose of prescribed medications

D LPN/LVN education and scope of practice include reinforcing education that has previously been done by the RN. Evaluating the patient response to exercise after a NSTEMI requires more education and should be done by the RN. Teaching and discharge planning/ documentation are higher level skills that require RN education and scope of practice. DIF: Cognitive Level: Apply (application) REF: 15-16 OBJ: Special Questions: Delegation TOP: Nursing Process: Planning MSC:

Health Promotion and Maintenance 43. After reviewing a patient's history, vital signs, physical assessment, and laboratory data, which information shown in the accompanying figure is most important for the nurse to communicate to the health care provider? a. Q waves on ECG b. Elevated troponin levels c. Fever and hyperglycemia d. Tachypnea and crackles in lungs

D Pulmonary congestion and tachypnea suggest that the patient may be developing heart failure, a complication of myocardial infarction (MI). Mild fever and hyperglycemia are common after MI because of the inflammatory process that occurs with tissue necrosis. Troponin levels will be elevated for several days after MI. Q waves often develop with ST-segment-elevation MI. DIF: Cognitive Level: Analyze (analysis) REF: 737 OBJ: Special Questions: Prioritization TOP: Nursing Process: Assessment MSC:

Physiological Integrity 31. When caring for a patient with acute coronary syndrome who has returned to the coronary care unit after having angioplasty with stent placement, the nurse obtains the following assessment data. Which data indicate the need for immediate action by the nurse? a. Heart rate 102 beats/min b. Pedal pulses 1+ bilaterally c. Blood pressure 103/54 mm Hg d. Chest pain level 7 on a 0 to 10 point scale

D The patient's chest pain indicates that restenosis of the coronary artery may be occurring and requires immediate actions, such as administration of oxygen and nitroglycerin, by the nurse. The other information indicates a need for ongoing assessments by the nurse. DIF: Cognitive Level: Apply (application) REF: 746 OBJ: Special Questions: Prioritization TOP: Nursing Process: Assessment MSC:

The patient seeks relief from the symptoms of an upper respiratory infection (URI) lasting for 5 days. Which patient assessment should the nurse use to help determine if the URI has developed into acute sinusitis? A. Coughing B. Fever, chills C. Dust allergy D. Maxillary pain

D The nurse should assess the patient for sinus pain or pressure as a clinical indicator of acute sinusitis. Coughing and fever are nonspecific clinical indicators of a URI. A history of an allergy that is likely to affect the upper respiratory tract is supportive of the sinusitis diagnosis but is not specific for sinusitis.

Physiological Integrity 8. A patient who has had chest pain for several hours is admitted with a diagnosis of rule out acute myocardial infarction (AMI). Which laboratory test should the nurse monitor to help determine whether the patient has had an AMI? a. Myoglobin b. Homocysteine c. C-reactive protein d. Cardiac-specific troponin

D Troponin levels increase about 4 to 6 hours after the onset of myocardial infarction (MI) and are highly specific indicators for MI. Myoglobin is released within 2 hours of MI, but it lacks specificity and its use is limited. The other laboratory data are useful in determining the patient's risk for developing coronary artery disease (CAD) but are not helpful in determining whether an acute MI is in progress. DIF: Cognitive Level: Understand (comprehension) REF: 749 TOP: Nursing Process: Assessment MSC:

When teaching the patient about the diet for diverticular disease, which foods should the nurse recommend? A) White bread, cheese, and green beans B) Fresh tomatoes, pears, and corn flakes C) Oranges, baked potatoes, and raw carrots D) Dried beans, All Bran (100%) cereal, and raspberries

D) A high fiber diet is recommended for diverticular disease. Dried beans, All Bran (100%) cereal, and raspberries all have higher amounts of fiber than white bread, cheese, green beans, fresh tomatoes, pears, corn flakes, oranges, baked potatoes, and raw carrots.

Which patient statement most clearly suggests a need to assess the patient for ankylosing spondylitis (AS)? A) "My right elbow has become red and swollen over the last few days." B) "I wake up stiff every morning, and my knees just don't want to bend." C) "My husband tells me that my posture has become so stooped this winter." D) "My lower back pain seems to be getting worse all the time, and nothing seems to help."

D) AS primarily affects the axial skeleton. Based on this, symptoms of inflammatory spine pain are often the first clues to a diagnosis of AS. Knee or elbow involvement is not consistent with the typical course of AS. Back pain is likely to precede the development of kyphosis.

When planning care for a patient with cirrhosis, the nurse will give highest priority to which of the following nursing diagnoses? A. Imbalanced nutrition: less than body requirements B. Impaired skin integrity related to edema, ascites, and pruritus C. Excess fluid volume related to portal hypertension and hyperaldosteronism D. Ineffective breathing pattern related to pressure on diaphragm and reduced lung volume

D) Although all of these nursing diagnoses are appropriate and important in the care of a patient with cirrhosis, airway and breathing are always the highest priorities.

The nurse is performing a focused abdominal assessment of a patient who has been recently admitted. In order to palpate the patient's liver, where should the nurse palpate the patient's abdomen? A) Left lower quadrant B) Left upper quadrant C) Right lower quadrant D) Right upper quadrant

D) Although the left lobe of the liver is located in the left upper quadrant of the abdomen, the bulk of the liver is located in the right upper quadrant.

The nurse is preparing to administer a dose of bisacodyl (Dulcolax). In explaining the medication to the patient, the nurse would state that it acts in which of the following ways? A) Increases bulk in the stool B) Lubricates the intestinal tract to soften feces C) Increases fluid retention in the intestinal tract D) Increases peristalsis by stimulating nerves in the colon wall

D) Bisacodyl is a stimulant laxative that aids in producing a bowel movement by irritating the colon wall and stimulating enteric nerves. It is available in oral and suppository forms. Fiber and bulk forming drugs increase bulk in the stool; water and stool softeners soften feces, and saline and osmotic solutions cause fluid retention in the intestinal tract.

The nurse is preparing to administer a scheduled dose of docusate sodium (Colace) when the patient complains of an episode of loose stool and does not want to take the medication. Which of the following is the appropriate action by the nurse? A) Write an incident report about this untoward event. B) Attempt to have the family convince the patient to take the ordered dose. C) Withhold the medication at this time and try to administer it later in the day. D) Chart the dose as not given on the medical record and explain in the nursing progress notes.

D) Bisacodyl is a stimulant laxative that aids in producing a bowel movement by irritating the colon wall and stimulating enteric nerves. It is available in oral and suppository forms. Fiber and bulk forming drugs increase bulk in the stool; water and stool softeners soften feces, and saline and osmotic solutions cause fluid retention in the intestinal tract.

The patient with a history of lung cancer and hepatitis C has developed liver failure and is considering liver transplantation. After the comprehensive evaluation, the nurse knows that which factor discovered may be a contraindication for liver transplantation? A) Has completed a college education B) Has been able to stop smoking cigarettes C) Has well-controlled type 1 diabetes mellitus D) The chest x-ray showed another lung cancer lesion.

D) Contraindications for liver transplant include severe extrahepatic disease, advanced hepatocellular carcinoma or other cancer, ongoing drug and/or alcohol abuse, and the inability to comprehend or comply with the rigorous post-transplant course.

What should the nurse instruct the patient to do to best enhance the effectiveness of a daily dose of docusate sodium (Colace)? A) Take a dose of mineral oil at the same time. B) Add extra salt to food on at least one meal tray. C) Ensure dietary intake of 10 g of fiber each day. D) Take each dose with a full glass of water or other liquid.

D) Docusate lowers the surface tension of stool, permitting water and fats to penetrate and soften the stool for easier passage. The patient should take the dose with a full glass of water and should increase overall fluid intake, if able, to enhance effectiveness of the medication. Dietary fiber intake should be a minimum of 20 g daily to prevent constipation. Mineral oil and extra salt are not recommended.

The nurse is caring for a 68-year-old patient admitted with abdominal pain, nausea, and vomiting. The patient has an abdominal mass, and a bowel obstruction is suspected. The nurse auscultating the abdomen listens for which type of bowel sounds that are consistent with the patient's clinical picture? A) Low-pitched and rumbling above the area of obstruction B) High-pitched and hypoactive below the area of obstruction C) Low-pitched and hyperactive below the area of obstruction D) High-pitched and hyperactive above the area of obstruction

D) Early in intestinal obstruction, the patient's bowel sounds are hyperactive and high-pitched, sometimes referred to as "tinkling" above the level of the obstruction. This occurs because peristaltic action increases to "push past" the area of obstruction. As the obstruction becomes complete, bowel sounds decrease and finally become absent.

Which menu choice by a patient with osteoporosis indicates that the nurse's teaching about appropriate diet has been effective? a. Pancakes with syrup and bacon b. Whole wheat toast and fruit jelly c. Two-egg omelet and a half grapefruit d. Oatmeal with skim milk and fruit yogurt

ANS: D Skim milk and yogurt are high in calcium. The other choices do not contain any high calcium foods.

19. A patient is started on alendronate (Fosamax) once weekly for the treatment of osteoporosis. The nurse determines that further instruction about the drug is needed when what is said by the patient? a. "I should take the drug with a meal to prevent stomach irritation." b. "This drug will prevent further bone loss and increase my bone density." c. "I need to sit or stand upright for at least 30 minutes after taking the drug." d. "I will still need to take my calcium supplements while taking this new drug."

19. a. The bisphosphonates, such as alendronate, must be taken at least 30 minutes before food or other medications to promote their absorption. As well, because they are very irritating to the stomach and esophagus, the patient must remain upright for at least 30 minutes after taking the medication to prevent reflux into the esophagus. These drugs will prevent further bone loss and increase bone density but calcium and vitamin D supplementation is still needed for bone formation.

The patient with fibromyalgia is suffering with pain at 12 of the 18 identification sites, including the neck and upper back and the knees. The patient also reports nonrefreshing sleep, depression, and being anxious when dealing with multiple tasks. The nurse should teach this patient about what treatments (select all that apply)? A) Low-impact aerobic exercise B) Relaxation strategy (biofeedback) C) Antiseizure drug pregabalin (Lyrica) D) Morphine sulfate extended-release tablets E) Serotonin reuptake inhibitor (e.g., sertraline [Zoloft])

A, B, C, E) Because the treatment of fibromyalgia is symptomatic, this patient will be prescribed something for pain, such as pregabalin, and a serotonin reuptake inhibitor for depression. Low- impact aerobic exercise will prevent muscle atrophy without increasing pain at the knees. Relaxation can help decrease the patient's stress and anxiety. Long-acting opioids are generally avoided unless pain cannot be relieved by other medications.

Despite a high dosage, a male patient who is taking nifedipine (Procardia XL) for antihypertensive therapy continues to have blood pressures over 140/90 mm Hg. What should the nurse do next? A. Assess his adherence to therapy. B. Ask him to make an exercise plan. C. Instruct him to use the DASH diet. D. Request a prescription for a thiazide diuretic.

A. A long-acting calcium-channel blocker such as nifedipine causes vascular smooth muscle relaxation, resulting in decreased systemic vascular resistance and arterial blood pressure and related side effects. The patient data the nurse has about this patient is very limited, so the nurse needs to begin by assessing adherence to therapy.

When reinforcing health teaching about the management of osteoarthritis (OA), the nurse determines that the patient needs additional instruction after making which of the following statements? A. "I should take the Celebrex as prescribed to help control the pain." B. "I should try to stay standing all day to keep my joints from becoming stiff." C. "I can use a cane if I find it helpful in relieving the pressure on my back and hip." D. "A warm shower in the morning will help relieve the stiffness I have when I get up."

Correct answer: B. "I should try to stay standing all day to keep my joints from becoming stiff." Rationale: It is important to maintain a balance between rest and activity to prevent overstressing the joints with OA.

The nurse is working with a 73-year-old patient with osteoarthritis (OA). In assessing the patient's understanding of this disorder, the nurse concludes teaching has been effective when the patient describes the condition as which of the following? A. Joint destruction caused by an autoimmune process B. Degeneration of articular cartilage in synovial joints C. Overproduction of synovial fluid resulting in joint destruction D. Breakdown of tissue in non-weight-bearing joints by enzymes

Correct answer: B. Degeneration of articular cartilage in synovial joints Rationale: OA is a degeneration or breakdown of the articular cartilage in synovial joints. The condition has also been referred to as degenerative joint disease.

A 24-year-old patient with a 12-year history of Becker muscular dystrophy is hospitalized with heart failure. What is an appropriate nursing intervention for this patient? a. Feed and bathe the patient to avoid exhausting the muscle. b. Reposition frequently to avoid skin and respiratory complications. c. Provide hand weights for the patient to exercise the upper extremities. d. Use orthopedic braces to promote ambulation and prevent muscle wasting.

8. b. Promotion of muscle activity is important in any patient with muscular dystrophy but when the disease has progressed to cardiomyopathy or respiratory failure, activity must be balanced with oxygen supply. At this stage of the disease, care should be taken to prevent skin or respiratory complications. The patient should be encouraged to perform as much self-care and exercise as energy allows but this will be limited.

When caring for a patient who is 3 hours postoperative laryngectomy, what is the nurse's highest priority assessment? A. Patient comfort B. Airway potency C. Incisional drainage D. Blood pressure and heart rate

B Remember the ABCs with prioritization. Airway patency is always the highest priority and is essential for a patient undergoing surgery surrounding the upper respiratory system. Comfort, drainage, and vital signs follow the ABCs in priority.

In caring for a patient admitted with poorly controlled hypertension, which laboratory test result should the nurse understand as indicating the presence of target organ damage? A. Serum uric acid of 3.8 mg/dL B. Serum creatinine of 2.6 mg/dL C. Serum potassium of 3.5 mEq/L D. Blood urea nitrogen of 15 mg/dL

B. The normal serum creatinine level is 0.6 to 1.3 mg/dL. This elevated level indicates target organ damage to the kidneys. The other laboratory results are within normal limits.

When providing dietary instruction to a patient with hypertension, the nurse would advise the patient to restrict intake of which meat? A. Broiled fish B. Roasted duck Correct C. Roasted turkey D. Baked chicken breast

B. Roasted duck is high in fat, which should be avoided by the patient with hypertension. Weight loss may slow the progress of atherosclerosis and overall cardiovascular disease risk. The other meats are lower in fat and are therefore acceptable in the diet.

The nurse supervises an unlicensed assistant personnel (UAP) who is taking the blood pressure of 58-yr-old obese female patient admitted with heart failure. Which action by the UAP will require the nurse to intervene? A. Waiting 2 minutes after position changes to take orthostatic pressures B. Deflating the blood pressure cuff at a rate of 8 to 10 mm Hg per second C. Taking the blood pressure with the patient's arm at the level of the heart D. Taking a forearm blood pressure because the largest cuff will not fit the patient's upper arm

B. The cuff should be deflated at a rate of 2 to 3 mm Hg per second. The arm should be supported at the level of the heart for accurate blood pressure measurements. Using a cuff that is too small causes a falsely high reading and too large causes a falsely low reading. If the maximum size blood pressure cuff does not fit the upper arm, the forearm may be used. Orthostatic blood pressures should be taken within 1 to 2 minutes of repositioning the patient.

A patient is being discharged from the emergency department after being treated for epistaxis. In teaching first aid measures in the event the epistaxis would recur, what measures should the nurse suggest (select all that apply.)? A. Tilt patient's head backwards. B. Apply ice compresses to the nose. C. Tilt head forward while lying down. D. Pinch the entire soft lower portion of the nose. E. Partially insert a small gauze pad into the bleeding nostril.

BDE First aid measures to control epistaxis include placing the patient in a sitting position, leaning forward. Pinching the soft lower portion of the nose or inserting a small gauze pad into the bleeding nostril should stop the bleeding within 15 minutes. Tilting the head back or forward does not stop the bleeding but rather allows the blood to enter the nasopharynx, which could result in aspiration or nausea or vomiting from swallowing blood. Lying down also will not decrease the bleeding.

The patient has been diagnosed with head and neck cancer. Along with the treatment for the cancer, what other treatment should the nurse expect to teach the patient about? A. Nasal packing B. Epistaxis balloon C. Gastrostomy tube D. Peripheral skin care

C Because 50% of patients with head and neck cancer are malnourished before treatment begins, many patients need enteral feeding via a gastrostomy tube because the effects of treatment make it difficult to take in enough nutrients orally, whether surgery, chemotherapy, or radiation is used. Nasal packing could be used with epistaxis or with nasal or sinus problems. Peripheral skin care would not be expected because it is not related to head and neck cancer.

Physiological Integrity 12. Heparin is ordered for a patient with a non-ST-segment-elevation myocardial infarction (NSTEMI). What is the purpose of the heparin? a. Heparin enhances platelet aggregation. b. Heparin decreases coronary artery plaque size. c. Heparin prevents the development of new clots in the coronary arteries. d. Heparin dissolves clots that are blocking blood flow in the coronary arteries.

C Heparin helps prevent the conversion of fibrinogen to fibrin and decreases coronary artery thrombosis. It does not change coronary artery plaque, dissolve already formed clots, or enhance platelet aggregation. DIF: Cognitive Level: Understand (comprehension) REF: 746 TOP: Nursing Process: Implementation MSC:

Health Promotion and Maintenance 2. Which nursing intervention will be most effective when assisting the patient with coronary artery disease (CAD) to make appropriate dietary changes? a. Give the patient a list of low-sodium, low-cholesterol foods that should be included in the diet. b. Emphasize the increased risk for heart problems unless the patient makes the dietary changes. c. Help the patient modify favorite high-fat recipes by using monosaturated oils when possible. d. Inform the patient that a diet containing no saturated fat and minimal salt will be necessary.

C Lifestyle changes are more likely to be successful when consideration is given to the patient's values and preferences. The highest percentage of calories from fat should come from monosaturated fats. Although low-sodium and low-cholesterol foods are appropriate, providing the patient with a list alone is not likely to be successful in making dietary changes. Completely removing saturated fat from the diet is not a realistic expectation. Up to 7% of calories in the therapeutic lifestyle changes (TLC) diet can come from saturated fat. Telling the patient about the increased risk without assisting further with strategies for dietary change is unlikely to be successful. DIF: Cognitive Level: Apply (application) REF: 736-737 TOP: Nursing Process: Implementation MSC:

The nurse teaches a 28-yr-old man newly diagnosed with hypertension about lifestyle modifications to reduce his blood pressure. Which patient statement requires reinforcement of teaching? A. "I will avoid adding salt to my food during or after cooking." B."If I lose weight, I might not need to continue taking medications." C. "I can lower my blood pressure by switching to smokeless tobacco." D. "Diet changes can be as effective as taking blood pressure medications.

C Nicotine contained in tobacco products (smoking and chew) cause vasoconstriction and increase blood pressure. Persons with hypertension should restrict sodium to 1500 mg/day by avoiding foods high in sodium and not adding salt in preparation of food or at meals. Weight loss can decrease blood pressure between 5 to 20 mm Hg. Following dietary recommendations (e.g., the DASH diet) lowers blood pressure, and these decreases compare with those achieved with blood pressure-lowering medication.

Physiological Integrity 16. During the administration of the thrombolytic agent to a patient with an acute myocardial infarction (AMI), the nurse should stop the drug infusion if the patient experiences a. bleeding from the gums. b. increase in blood pressure. c. a decrease in level of consciousness. d. a nonsustained episode of ventricular tachycardia.

C The change in level of consciousness indicates that the patient may be experiencing intracranial bleeding, a possible complication of thrombolytic therapy. Some bleeding of the gums is an expected side effect of the therapy but not an indication to stop infusion of the thrombolytic medication. A decrease in blood pressure could indicate internal bleeding. A nonsustained episode of ventricular tachycardia is a common reperfusion dysrhythmia and may indicate that the therapy is effective. DIF: Cognitive Level: Apply (application) REF: 752 TOP: Nursing Process: Evaluation MSC:

Safe and Effective Care Environment 30. Which electrocardiographic (ECG) change is most important for the nurse to report to the health care provider when caring for a patient with chest pain? a. Inverted P wave b. Sinus tachycardia c. ST-segment elevation d. First-degree atrioventricular block

C The patient is likely to be experiencing an ST-segment-elevation myocardial infarction (STEMI). Immediate therapy with percutaneous coronary intervention (PCI) or thrombolytic medication is indicated to minimize myocardial damage. The other ECG changes may also suggest a need for therapy, but not as rapidly. DIF: Cognitive Level: Apply (application) REF: 747-748 OBJ: Special Questions: Prioritization TOP: Nursing Process: Assessment MSC:

Physiological Integrity 42. After reviewing information shown in the accompanying figure from the medical records of a 43-year-old, which risk factor modification for coronary artery disease should the nurse include in patient teaching? a. Importance of daily physical activity b. Effect of weight loss on blood pressure c. Dietary changes to improve lipid levels d. Ongoing cardiac risk associated with history of tobacco use

C The patient has an elevated low-density lipoprotein (LDL) cholesterol and low high-density lipoprotein (HDL) cholesterol, which will increase the risk of coronary artery disease. Although the blood pressure is in the prehypertensive range, the patient's waist circumference and body mass index (BMI) indicate an appropriate body weight. The risk for coronary artery disease a year after quitting smoking is the same as a nonsmoker. The patient's occupation indicates that daily activity is at the levels suggested by national guidelines. DIF: Cognitive Level: Analyze (analysis) REF: 737 TOP: Nursing Process: Planning MSC:

Physiological Integrity 18. In preparation for discharge, the nurse teaches a patient with chronic stable angina how to use the prescribed short-acting and long-acting nitrates. Which patient statement indicates that the teaching has been effective? a. "I will check my pulse rate before I take any nitroglycerin tablets." b. "I will put the nitroglycerin patch on as soon as I get any chest pain." c. "I will stop what I am doing and sit down before I put the nitroglycerin under my tongue." d. "I will be sure to remove the nitroglycerin patch before taking any sublingual nitroglycerin."

C The patient should sit down before taking the nitroglycerin to decrease cardiac workload and prevent orthostatic hypotension. Transdermal nitrates are used prophylactically rather than to treat acute pain and can be used concurrently with sublingual nitroglycerin. Although the nurse should check blood pressure before giving nitroglycerin, patients do not need to check the pulse rate before taking nitrates. DIF: Cognitive Level: Apply (application) REF: 743 TOP: Nursing Process: Evaluation MSC:

A female patient's complex symptomatology over the past year has led to a diagnosis of systemic lupus erythematosus (SLE). Which statement demonstrates the patient's need for further teaching about the disease? A) "I'll try my best to stay out of the sun this summer." B) "I know that I probably have a high chance of getting arthritis." C) "I'm hoping that surgery will be an option for me in the future." D) "I understand that I'm going to be vulnerable to getting infections."

C) Surgery is not a key treatment modality for SLE, so this indicates a need for further teaching. SLE carries an increased risk of infection, sun damage, and arthritis.

The nurse is caring for a patient who has osteoarthritis (OA) of the knees. The nurse teaches the patient that the most beneficial measure to protect the joints is to do which of the following? A. Use a wheelchair to avoid walking as much as possible. B. Eat a well-balanced diet to maintain a healthy body weight. C. Incorrect Use a walker for ambulation to relieve the pressure on her hips. D. Sit in chairs that do not cause her hips to be lower than her knees.

Correct answer: B. Eat a well-balanced diet to maintain a healthy body weight. Rationale: Because maintaining an appropriate load on the joints is essential to the preservation of articular cartilage integrity, the patient should maintain an optimal overall body weight or lose weight if overweight.

Lewis A patient with rheumatoid arthritis is experiencing articular involvement of the joints. The nurse recognizes that these characteristic changes include (select all that apply) a. bamboo-shaped fingers. b. metatarsal head dislocation in feet. c. noninflammatory pain in large joints. d. asymmetric involvement of small joints. e. morning stiffness lasting 60 minutes or more.

b, e

When caring for a client with extensive burns, the nurse anticipates that pain medication will be administered via which route? a. oral b. IV c. IM d. Subq

b.

A patient is admitted for joint replacement surgery and has a permanent tracheostomy. Which task is appropriate for the nurse to delegate to unlicensed assistive personnel (UAP)? A. Suction the tracheostomy. B. Check stoma site for skin breakdown. C. Complete tracheostomy care using sterile technique. D. Provide oral care with a toothbrush and tonsil suction tube.

D Oral care (for a stable patient with a tracheostomy) can be delegated to UAP. A registered nurse would be responsible for assessments (e.g., checking the stoma for skin breakdown) and tracheostomy suctioning and care.

Physiological Integrity 24. A patient with hyperlipidemia has a new order for colesevelam (Welchol). Which nursing action is most appropriate when giving the medication? a. Have the patient take this medication with an aspirin. b. Administer the medication at the patient's usual bedtime. c. Have the patient take the colesevelam with a sip of water. d. Give the patient's other medications 2 hours after the colesevelam.

D The bile acid sequestrants interfere with the absorption of many other drugs, and giving other medications at the same time should be avoided. Taking an aspirin concurrently with the colesevelam may increase the incidence of gastrointestinal side effects such as heartburn. An increased fluid intake is encouraged for patients taking the bile acid sequestrants to reduce the risk for constipation. For maximum effect, colesevelam should be administered with meals. DIF: Cognitive Level: Apply (application) REF: 739 TOP: Nursing Process: Implementation MSC:

The nurse is caring for a postoperative patient with a colostomy. The nurse is preparing to administer a dose of famotidine (Pepcid) when the patient asks why the medication was ordered since the patient does not have a history of heartburn or gastroesophageal reflux disease (GERD). What response by the nurse would be the most appropriate? A) "This will prevent air from accumulating in the stomach, causing gas pains." B) "This will prevent the heartburn that occurs as a side effect of general anesthesia." C) "The stress of surgery is likely to cause stomach bleeding if you do not receive it." D) "This will reduce the amount of HCl in the stomach until the nasogastric tube is removed and you can eat a regular diet again."

D) Famotidine is an H2-receptor antagonist that inhibits gastric HCl secretion and thus minimizes damage to gastric mucosa while the patient is not eating a regular diet after surgery. Famotidine does not prevent air from accumulating in the stomach or stop the stomach from bleeding. Heartburn is not a side effect of general anesthesia.

The family of a patient newly diagnosed with hepatitis A asks the nurse what they can do to prevent becoming ill themselves. Which of the following responses by the nurse is most appropriate? A. "The hepatitis vaccine will provide immunity from this exposure and future exposures." B. "I am afraid there is nothing you can do since the patient was infectious before admission." C. "You will need to be tested first to make sure you don't have the virus before we can treat you." D. "An injection of immunoglobulin will need to be given to prevent or minimize the effects from this exposure."

D) Immunoglobulin provides temporary (1-2 months) passive immunity and is effective for preventing hepatitis A if given within 2 weeks after exposure. It may not prevent infection in all persons, but it will at least modify the illness to a subclinical infection. The hepatitis vaccine is only used for preexposure prophylaxis

The nurse would question the use of which cathartic agent in a patient with renal insufficiency? A) Bisacodyl (Dulcolax) B) Lubiprostone (Amitiza) C) Cascara sagrada (Senekot) D) Magnesium hydroxide (Milk of Magnesia)

D) Milk of Magnesia may cause hypermagnesemia in patients with renal insufficiency. The nurse should question this order with the health care provider. Bisacodyl, lubiprostone, and cascara sagrada are safe to use in patients with renal insufficiency as long as the patient is not currently dehydrated.

A patient who has hepatitis B surface antigen (HBsAg) in the serum is being discharged with pain medication after knee surgery. Which medication order should the nurse question because it is most likely to cause hepatic complications? A) Tramadol (Ultram) B) Hydromorphone (Dilaudid) C) Oxycodone with aspirin (Percodan) D) Hydrocodone with acetaminophen (Vicodin)

D) The analgesic with acetaminophen should be questioned because this patient is a chronic carrier of hepatitis B and is likely to have impaired liver function. Acetaminophen is not suitable for this patient because it is converted to a toxic metabolite in the liver after absorption, increasing the risk of hepatocellular damage.

The patient with suspected pancreatic cancer is having many diagnostic studies done. Which one can be used to establish the diagnosis of pancreatic adenocarcinoma and for monitoring the response to treatment? A) Spiral CT scan B) A PET/CT scan C) Abdominal ultrasound D) Cancer-associated antigen 19-9

D) The cancer-associated antigen 19-9 (CA 19-9) is the tumor marker used for the diagnosis of pancreatic adenocarcinoma and for monitoring the response to treatment. Although a spiral CT scan may be the initial study done and provides information on metastasis and vascular involvement, this test and the PET/CT scan or abdominal ultrasound do not provide additional information.

When teaching the patient with acute hepatitis C (HCV), the patient demonstrates understanding when the patient makes which statement? A) "I will use care when kissing my wife to prevent giving it to her." B) "I will need to take adofevir (Hepsera) to prevent chronic HCV." C) "Now that I have had HCV, I will have immunity and not get it again." D) "I will need to be checked for chronic HCV and other liver problems."

D) The majority of patients who acquire HCV usually develop chronic infection, which may lead to cirrhosis or liver cancer. HCV is not transmitted via saliva, but percutaneously and via high-risk sexual activity exposure. The treatment for acute viral hepatitis focuses on resting the body and adequate nutrition for liver regeneration. Adofevir (Hepsera) is taken for severe hepatitis B (HBV) with liver failure. Chronic HCV is treated with pegylated interferon with ribavirin. Immunity with HCV does not occur as it does with HAV and HBV, so the patient may be reinfected with another type of HCV.

The nurse asks a 68-year-old patient scheduled for colectomy to sign the operative permit as directed in the physician's preoperative orders. The patient states that the physician has not really explained very well what is involved in the surgical procedure. What is the most appropriate action by the nurse? A) Ask family members whether they have discussed the surgical procedure with the physician. B) Have the patient sign the form and state the physician will visit to explain the procedure before surgery. C) Explain the planned surgical procedure as well as possible and have the patient sign the consent form. D) Delay the patient's signature on the consent and notify the physician about the conversation with the patient.

D) The patient should not be asked to sign a consent form unless the procedure has been explained to the satisfaction of the patient. The nurse should notify the physician, who has the responsibility for obtaining consent.

The nurse is teaching a group of high school students about the prevention of food poisoning. Which comment by the student shows understanding of foodborne illness protection? A) "We like to mix up the ingredients so the flavors will melt before we cook our beef stew." B) "For a snack, I like to eat raw cookie dough from the package instead of baking the cookies." C) "We only have one cutting board, so we cut up our chicken and salad vegetables at the same time." D) "When they gave me a pink hamburger I sent it back and asked for a new bun and clean plate."

D) The student who did not accept the pink hamburger and asked for a new bun and clean plate understood that the pink meat may not have reached 160° and could be contaminated with bacteria. Mixing ingredients and leaving them long enough for the flavors to melt, eating raw cookie dough from a refrigerated package, and only using one cutting board without washing it with hot soapy water between the chicken and salad vegetables could all lead to food poisoning from contamination.

The ED nurse has inspected, auscultated, and palpated the abdomen with no obvious abnormalities, except pain. When the nurse palpates the abdomen for rebound tenderness, there is severe pain. The nurse should know that this could indicate what problem? A) Hepatic cirrhosis B) Hypersplenomegaly C) Gall bladder distention D) Peritoneal inflammation

D) When palpating for rebound tenderness, the problem area of the abdomen will produce pain and severe muscle spasm when there is peritoneal inflammation. Hepatic cirrhosis, hypersplenomegaly, and gall bladder distention do not manifest with rebound tenderness.

The nurse is preparing to administer a scheduled dose of docusate sodium (Colace) when the patient reports an episode of loose stool and does not want to take the medication. What is the appropriate action by the nurse? A) Write an incident report about this untoward event. B) Attempt to have the family convince the patient to take the ordered dose. C) Withhold the medication at this time and try to administer it later in the day. D) Chart the dose as not given on the medical record and explain in the nursing progress notes.

D) Whenever a patient refuses medication, the dose should be charted as not given with an explanation of the reason documented in the nursing progress notes. In this instance, the refusal indicates good judgment by the patient, and the patient should not be encouraged to take it today.

A 44-yr-old man is diagnosed with hypertension and receives a prescription for benazepril (Lotensin). After providing instruction, which statement by the patient indicates correct understanding? A. "If I take this medication, I will not need to follow a special diet." B. "It is normal to have some swelling in my face while taking this medication." C. "I will need to eat foods such as bananas and potatoes that are high in potassium." D. "If I develop a dry cough while taking this medication, I should notify my doctor."

D. Benazepril is an angiotensin-converting enzyme inhibitor. The medication inhibits breakdown of bradykinin, which may cause a dry, hacking cough. Other adverse effects include hyperkalemia. Swelling in the face could indicate angioedema and should be reported immediately to the prescriber. Patients taking drug therapy for hypertension should also attempt lifestyle modifications to lower blood pressure such as a reduced-sodium diet.

A female patient who has type 1 diabetes mellitus has chronic stable angina that is controlled with rest. She states that over the past few months she has required increasing amounts of insulin. What goal should the nurse use to plan care that should help prevent cardiovascular disease progression? A. Exercise almost every day. B. Avoid saturated fat intake. C. Limit calories to daily limit. D. Keep Hgb A1C less than 7%.

D. Keep Hgb A1C less than 7%. If the Hgb A1C is kept below 7%, this means that the patient has had good control of her blood glucose over the past 3 months. The patient indicates that increasing amounts of insulin are being required to control her blood glucose. This patient may not be adhering to the dietary guidelines or therapeutic regimen, so teaching about how to maintain diet, exercise, and medications to maintain stable blood glucose levels will be needed to achieve this goal.

The nurse cares for a 58-year-old woman with breast cancer who is admitted for severe back pain related to a compression fracture. The patient's laboratory values include serum potassium of 4.5 mEq/L, serum sodium of 144 mEq/L, and serum calcium of 14.3 mg/dL. Which signs and symptoms will the nurse expect the patient to exhibit? A. Anxiety, irregular pulse, and weakness B. Muscle stiffness, dysphagia, and dyspnea C. Hyperactive reflexes, tremors, and seizures D. Nausea, vomiting, and altered mental status

D. Nausea, vomiting, and altered mental status Breast cancer can metastasize to the bone. Vertebrae are a common site. Pathologic fractures at the site of metastasis are common because of a weakening of the involved bone. High serum calcium levels result as calcium is released from damaged bones. Normal serum calcium is between 8.6 to 10.2 mg/dL. Clinical manifestations of hypercalcemia include nausea, vomiting, and altered mental status (e.g., lethargy, decreased memory, confusion, personality changes, psychosis, stupor, coma). Other manifestations include weakness, depressed reflexes, anorexia, bone pain, fractures, polyuria, dehydration, and nephrolithiasis. Manifestations of hypomagnesemia include hyperactive reflexes, tremors, and seizures. Symptoms of hyperkalemia include anxiety, irregular pulse, and weakness. Symptoms of hypocalcemia include muscle stiffness, dysphagia, and dyspnea.

When the patient is diagnosed with muscular dystrophy, what information should the nurse include in the teaching about this disorder? A. Prolonged bed rest will be used to decrease fatigue. B. An orthotic jacket will limit mobility and may contribute to deformity. C. Continuous positive airway pressure will be used to facilitate sleeping. D. Remain active to prevent skin breakdown and respiratory complications.

D. Remain active to prevent skin breakdown and respiratory complications. With muscular dystrophy, it is important for the patient to remain active for as long as possible. Prolonged bed rest should be avoided because immobility leads to further muscle wasting. An orthotic jacket may be used to provide stability and prevent further deformity. Continuous positive airway pressure (CPAP) is used as respiratory function decreases, before mechanical ventilation is needed to sustain respiratory function.

When evaluating a patient's knowledge regarding a low-sodium, low-fat cardiac diet, the nurse recognizes additional teaching is needed when the patient selects which food choice? A. Baked flounder B. Angel food cake C. Baked potato with margarine D. Canned chicken noodle soup

D. Canned chicken noodle soup Canned soups are very high in sodium content. Patients need to be taught to read food labels for sodium and fat content.

A patient was admitted to the emergency department (ED) 24 hours earlier with complaints of chest pain that were subsequently attributed to ST-segment-elevation myocardial infarction (STEMI). What complication of MI should the nurse anticipate? A. Unstable angina B. Cardiac tamponade C. Sudden cardiac death D. Cardiac dysrhythmias

D. Cardiac dysrhythmias The most common complication after MI is dysrhythmias, which are present in 80% of patients. Unstable angina is considered a precursor to MI rather than a complication. Cardiac tamponade is a rare event, and sudden cardiac death is defined as an unexpected death from cardiac causes. Cardiac dysfunction in the period following an MI would not be characterized as sudden cardiac death.

A patient experienced sudden cardiac death (SCD) and survived. What should the nurse expect to be used as preventive treatment for the patient? A. External pacemaker B. An electrophysiologic study (EPS) C. Medications to prevent dysrhythmias D. Implantable cardioverter-defibrillator (ICD)

D. Implantable cardioverter-defibrillator (ICD) An ICD is the most common approach to preventing recurrence of SCD. An external pacemaker may be used in the hospital but will not be used for the patient living daily life at home. An EPS may be done to determine if a recurrence is likely and determine the most effective medication treatment. Medications to prevent dysrhythmias are used but are not the best prevention of SCD.

The patient with *type 1 diabetes mellitus* is having a *seizure*. Which medication should the nurse anticipate will be administered first? IV dextrose solution IV diazepam (Valium) IV phenytoin (Dilantin) Oral carbamazepine (Tegretol)

IV dextrose solution This patient's seizure could be caused by low blood glucose, so IV dextrose solution would be given first to stop the seizure

A male patient who has coronary artery disease (CAD) has serum lipid values of LDL cholesterol 98 mg/dL and HDL cholesterol 47 mg/dL. What should the nurse include in the patient teaching? A. Consume a diet low in fats. B. Reduce total caloric intake. C. Increase intake of olive oil. D. The lipid levels are normal.

D. The lipid levels are normal. For men, the recommended LDL is less than 100 mg/dL, and the recommended level for HDL is greater than 40mg/dL. His normal lipid levels should be included in the patient teaching and encourage him to continue taking care of himself. Assessing his need for teaching related to diet should also be done.

A 60-year-old woman has pain on motion in her fingers and asks the nurse whether this is just a result of aging. The best response by the nurse includes the information that a. joint pain with functional limitation is a normal change that affects all people to some extent b. joint pain that develops with age is usually related to previous trauma or infection of the joints c. this is a symptom of a systemic arthritis that eventually affects all joints as the disease progresses d. changes in the cartilage and bones of joints may cause symptoms of pain and loss of function in some people as they age

D. changes in the cartilage and bones of joints may cause symptoms of pain and loss of function in some people as they age.

Characteristics of spondyloarthritides associated with HLA-B27 antigen include a. symmetric polyarticular arthritis b. an absence of extraarticular disease c. presence of rheumatoid factor and autoantibodies d. high level of involvement of sacroiliac joints and the spine

D. high level of involvement is sacroiliac joints and the spine

Laboratory findings that the nurse would expect to be present in the patient with RA include a. polycythemia b. increased IgG c. decreased WBC d. increased C-reactive protein (CRP)

D. increased C-reactive protein (CRP)

An important nursing intervention for the patient with ankylosing spondylitis is to teach the patient to a. wear roomy shoes with good orthotic support b. sleep on the side with the knees and hips flexed c. keep the spine slightly flexed while sitting, standing, or walking d. perform back, neck, and chest stretches and deep breathing exercises

D. perform back, neck and chest stretches and deep breathing exercises

A patient is seen at the outpatient clinic for a sudden onset of inflammation and severe pain in the great toe. A diagnosis of gout is made on the basis of a. a family history of gout b. elevated urine uric acid levels c. elevated serum uric acid levels d. the presence of sodium urate crystals in synovial fluid

D. the presence of sodium urate cystals in synovial fluid

After teaching a patient with RA to use heat and cold therapy to relieve symptoms, the nurse determines that teaching has been effective when the patient says, a. heat treatments should not be used if muscle spasms are present b. cold applications can be applied for 15-20 minutes to relieve joint stiffness c. I should use heat applications for 20 minutes to relieve the symptoms of an acute flare d. when my joints are painful, I can use a bag of frozen corn for 10 to 15 minutes to relieve the pain

D. when my joints are painful, I can use a bag of frozen corn for 10 to 15 minutes to relive the pain

A 72-year-old patient was admitted with epigastric pain due to a gastric ulcer. Which patient assessment warrants an urgent change in the nursing plan of care? A) Chest pain relieved with eating or drinking water B) Back pain 3 or 4 hours after eating a meal C) Burning epigastric pain 90 minutes after breakfast D) Rigid abdomen and vomiting following indigestion

D.A rigid abdomen with vomiting in a patient who has a gastric ulcer indicates a perforation of the ulcer, especially if the manifestations of perforation appear suddenly. Midepigastric pain is relieved by eating, drinking water, or antacids with duodenal ulcers, not gastric ulcers. Back pain 3-4 hours after a meal is more likely to occur with a duodenal ulcer. Burning epigastric pain 1-2 hours after a meal is an expected manifestation of a gastric ulcer related to increased gastric secretions and does not cause an urgent change in the nursing plan of care.

The nurse observes a 74-yr-old man with Parkinson's disease rocking side to side while sitting in the chair. Which action by the nurse is most appropriate? Provide the patient with diversional activities. Document the activity in the patient's health record. Take the patient's blood pressure sitting and standing. Ask if the patient is feeling either anxious or depressed.

Document the activity in the patient's health record. Patients with Parkinson's disease are instructed to rock from side to side to stimulate balance mechanisms and decrease akinesia.

C

Nursing assessment findings of jugular venous distention and pedal edema would be indicative of what complication of chronic obstructive pulmonary disease (COPD)? Acute respiratory failure Secondary respiratory infection Fluid volume excess resulting from cor pulmonale Pulmonary edema caused by left-sided heart failure

A 50-yr-old male patient has been diagnosed with *amyotrophic lateral sclerosis (ALS)*. What strategy will *prevent* a common cause of *death* for patients with ALS? Reduce fat intake. Reduce the risk of aspiration. Decrease injury related to falls. Decrease pain secondary to muscle weakness.

Reduce the risk of aspiration. Reducing the risk of aspiration can help prevent respiratory infections that are a common cause of death from deteriorating muscle function.

Physiological Integrity 25. The nurse is caring for a patient who was admitted to the coronary care unit following an acute myocardial infarction (AMI) and percutaneous coronary intervention the previous day. Teaching for this patient would include a. when cardiac rehabilitation will begin. b. the typical emotional responses to AMI. c. information regarding discharge medications. d. the pathophysiology of coronary artery disease.

A Early after an AMI, the patient will want to know when resumption of usual activities can be expected. At this time, the patient's anxiety level or denial will interfere with good understanding of complex information such as the pathophysiology of coronary artery disease (CAD). Teaching about discharge medications should be done closer to discharge. The nurse should support the patient by decreasing anxiety rather than discussing the typical emotional responses to myocardial infarction (MI). DIF: Cognitive Level: Apply (application) REF: 759-762 TOP: Nursing Process: Planning MSC:

A client with osteoporosis needs education about diet and ways to increase bone density. Which of the following should be included in the teaching plan? Select all that apply. 1. Maintain a diet with adequate amounts of vitamin D, as found in fortified milk and cereals. 2. Choose good calcium sources, such as figs, broccoli, and almonds. 3. Use alcohol in moderation because a moderate intake has no known negative effects. 4. Try swimming as a good exercise to maintain bone mass. 5. Avoid the use of high-fat foods, such as avocados, salad dressings, and fried foods.

1, 2, 3. A diet with adequate amounts of vitamin D aids in the regulation, absorption, and subsequent utilization of calcium and phosphorus, which are necessary for the normal calcification of bone. Figs, broccoli, and almonds are very good sources of calcium. Moderate intake of alcohol has no known negative effects on bone density but excessive alcohol intake does reduce bone density. Swimming, biking, and other non- weight-bearing exercises do not maintain bone mass. Walking and running, which are weight-bearing exercises, do maintain bone mass. The client should eat a balanced diet but does not need to avoid the use of high-fat foods.

Of the clients listed below, who is at risk for developing rheumatoid arthritis (RA)? Select all that apply. 1. Adults between the ages of 20 and 50 years. 2. Adults who have had an infectious disease with the Epstein-Barr virus. 3. Adults that are of the male gender. 4. Adults who possess the genetic link, specifically HLA-DR4. 5. Adults who also have osteoarthritis.

1, 2, 4. Rheumatoid arthritis (RA) affects women three times more often than men, between the ages of 20 and 55 years. Research has determined that RA occurs in clients who have had infectious disease, such as the Epstein-Barr virus. The genetic link, specifically HLA-DR4, has been found in 65% of clients with RA. People with osteoarthritis are not necessarily at risk for developing rheumatoid arthritis.

A postmenopausal client is scheduled for a bone-density scan. To plan for the client's test, what should the nurse communicate to the client? 1. Request that the client remove all metal objects on the day of the scan. 2. Instruct the client to consume foods and beverages with a high content of calcium for 2 days before the test. 3. Inform the client that she will need to ingest 600 mg of calcium gluconate by mouth for 2 weeks before the test. 4. Tell the client that she should report any significant pain to her physician at least 2 days before the test.

1. Metal will interfere with the test. Metallic objects within the examination field, such as jewelry, earrings, and dental amalgams, may inhibit organ visualization and can produce unclear images. Ingesting foods and beverages days before the test will not affect bone mineral status. Short-term calcium gluconate intake will also not influence bone mineral status. The client may already have had chronic pain as a result of a bone fracture or from osteoporosis.

A physician orders a lengthy X-ray examination for a client with osteoarthritis. Which of the following actions by the nurse would demonstrate client advocacy? 1. Contact the X-ray department and ask the technician if the lengthy session can be divided into shorter sessions. 2. Contact the physician to determine if an alternative examination could be scheduled. 3. Provide a dose of acetaminophen (Tylenol). 4. Cancel the examination because of the hard X-ray table.

1. Shorter sessions will allow the client to rest between the sessions. Changing the physician's order to a different examination will not provide the information needed for this client's treatment. Acetaminophen is a nonopioid analgesic and an antipyretic, not an anti-inflammatory agent. Thus, it would not help this client avoid the adverse effects of a lengthy X-ray examination. Although the X-ray table is hard, there are other options for making the client comfortable, rather than canceling the examination.

1. Emerging and reemerging infections affect health care by (select all that apply) a. reevaluating vaccine practices. b. revealing antimicrobial resistance. c. limiting antibiotics to those with life-threatening infection. d. challenging researchers to discover new antimicrobial therapies.

1. Correct answers: a, b, d An emerging infection is an infectious disease whose incidence has increased in the past 20 years or threatens to increase in the immediate future. Reemerging infections are those infections that were previously controlled but have resurfaced. The most common reason for reemerging infectious is low vaccination rates. Ways in which emerging and reemerging infectious have affected the health care system include revising vaccine recommendations for previously controlled infections (e.g., pertussis and measles); discovery of antimicrobial-resistant organisms; and creation of new antiinfective agents to combat new organisms or antimicrobial-resistant infections.

18. Identify methods to specifically prevent osteoporosis in postmenopausal women (select all that apply). a. Eating more beef b. Eating 8 ounces of yogurt daily c. Performing weight-bearing exercise d. Spending 15 minutes in the sun each day e. Taking postmenopausal estrogen replacement

18. b, c. To specifically prevent osteoporosis in postmenopausal women, increased calcium and vitamin D intake and weight-bearing exercises (i.e., walking) are the best methods. Beef is not high in calcium or vitamin D. Although 20 minutes in the sun each day provides vitamin D for most women, it is recommended that postmenopausal women take supplemental vitamin D doses of 800 to 1000 IU per day. Although estrogen replacement will protect the woman against bone loss and fractures, it is no longer given specifically to prevent osteoporosis because of increased risk of heart disease and breast or uterine cancer.

A 25-year-old client taking hydroxychloroquine (Plaquenil) for rheumatoid arthritis reports difficulty seeing out of her left eye. Correct interpretation of this assessment finding indicates which of the following? 1. Development of a cataract. 2. Possible retinal degeneration. 3. Part of the disease process. 4. A coincidental occurrence.

2. Difficulty seeing out of one eye, when evaluated in conjunction with the client's medication therapy regimen, leads to the suspicion of possible retinal degeneration. The possibility of an irreversible retinal degeneration caused by deposits of hydroxychloroquine (Plaquenil) in the layers of the retina requires an ophthalmologic examination before therapy is begun and at 6-month intervals. Although cataracts may develop in young adults, they are less likely, and damage from the hydroxychloroquine is the most obvious at-risk factor. Eyesight is not affected by the disease process of rheumatoid arthritis.

After teaching a group of clients with osteoarthritis about using regular exercise, which of the following client statements indicates effective teaching? 1. "Performing range-of-motion exercises will increase my joint mobility." 2. "Exercise helps to drive synovial fluid through the cartilage." 3. "Joint swelling should determine when to stop exercising." 4. "Exercising in the outdoors year-round promotes joint relaxation."

2. Weight-bearing exercise plays a very important role in stimulating regeneration of cartilage, which lacks blood vessels, by driving synovial fluid through the joint cartilage. Joint mobility is increased by weight-bearing exercises, not range-of-motion exercises, because surrounding muscles, ligaments, and tendons are strengthened. Pain is an early sign of degenerative joint bone problems. Swelling may not occur for some time after pain, if at all. Osteoarthritic pain is worsened in cold, damp weather; therefore, exercising outdoors is not recommended year round in all settings.

2. Which types(s) of isolation precautions is (are) appropriate for a patient with tuberculosis (select all that apply)? a. contact precautions b. droplet precautions c. airborne precautions d. standard precautions e. neutropenic precautions

2. Correct answers: c, d Standard precautions should be used for all patients. In addition to standard precautions, patients with TB should be placed on airborne precautions to minimize risk of transmission and infection of HCPs and visitors.

2. A patient with osteomyelitis has a nursing diagnosis of risk for injury. What is an appropriate nursing intervention for this patient? a. Use careful and appropriate disposal of soiled dressings. b. Gently handle the involved extremity during movement. c. Measure the circumference of the affected extremity daily. d. Provide range-of-motion (ROM) exercise q4hr to the involved extremity.

2. b. The patient with osteomyelitis is at risk for pathologic fractures at the site of the infection because of weakened, devitalized bone and careful handling of the extremity isnecessary. Careful handling of dressings is necessary to prevent the spread of infection to others but is not related to preventing injury to this patient. Splints may be used to immobilize the limb, range-of-motion (ROM) exercises will be limited because of the possibility of spreading infection, and edema is not a common finding in osteomyelitis.

At which of the following times should the nurse instruct the client to take ibuprofen (Motrin), prescribed for left hip pain secondary to osteoarthritis, to minimize gastric mucosal irritation? 1. At bedtime. 2. On arising. 3. Immediately after a meal. 4. On an empty stomach.

3. Drugs that cause gastric irritation, such as ibuprofen, are best taken after or with a meal, when stomach contents help minimize the local irritation. Taking the medication on an empty stomach at any time during the day will lead to gastric irritation. Taking the drug at bedtime with food may cause the client to gain weight, possibly aggravating the osteoarthritis. When the client arises, he is stiff from immobility and should use warmth and stretching until he gets food in his stomach.

The nurse teaches a client about heat and cold treatments to manage arthritis pain. Which of the following client statements indicates that the client still has a knowledge deficit? 1. "I can use heat and cold as often as I want." 2. "With heat, I should apply it for no longer than 20 minutes at a time." 3. "Heat-producing liniments can be used with other heat devices." 4. "Ten to 15 minutes per application is the maximum time for cold applications."

3. Heat-producing liniment can produce a burn if used with other heat devices that could intensify the heat reaction. Heat and cold can be used as often as the client desires. However, each application of heat should not exceed 20 minutes, and each application of cold should not exceed 10 to 15 minutes. Application for longer periods results in the opposite of the intended effect: vasoconstriction instead of vasodilation with heat, and vasodilation instead of vasoconstriction with cold.

On a visit to the clinic, a client reports the onset of early symptoms of rheumatoid arthritis. The nurse should conduct a focused assessment for: 1. Limited motion of joints. 2. Deformed joints of the hands. 3. Early morning stiffness. 4. Rheumatoid nodules.

3. Initially, most clients with early symptoms of rheumatoid arthritis complain of early morning stiffness or stiffness after sitting still for a while. Later symptoms of rheumatoid arthritis include limited joint range of motion; deformed joints, especially of the hand; and rheumatoid nodules.

A client is in the acute phase of rheumatoid arthritis. Which of the following should the nurse identify as lowest priority in the plan of care? 1. Relieving pain. 2. Preserving joint function. 3. Maintaining usual ways of accomplishing tasks. 4. Preventing joint deformity.

3. Maintaining usual ways of accomplishing tasks would be the lowest priority during the acute phase. Rather, the focus is on developing less stressful ways of accomplishing routine tasks. Pain relief is a high priority during the acute phase because pain is typically severe and interferes with the client's ability to function. Preserving joint function and preventing joint deformity are high priorities during the acute phase to promote an optimal level of functioning and reduce the risk of contractures.

3. Transmission of HIV from an infected individual to another most commonly occurs as a result of a. unprotected anal or vaginal sexual intercourse. b. low levels of virus in the blood and high levels of CD4+ T cells. c. transmission from mother to infant during labor and delivery and breastfeeding. d. sharing of drug-using equipment, including needles, syringes, pipes, and straws.

3. Correct answer: a Unprotected sexual contact (semen, vaginal secretions, or blood) with a partner infected with human immunodeficiency virus (HIV) is the most common mode of HIV transmission.

3. A patient who experienced an open fracture of the humerus 2 weeks ago is having increased pain at the fracture site. To identify a possible causative agent of osteomyelitis at the site, what should the nurse expect testing to include? a. X-rays b. CT scan c. Bone biopsy d. WBC count and erythrocyte sedimentation rate (ESR)

3. c. Because large doses of appropriate antibiotics are necessary in the treatment of acute osteomyelitis, it is important to identify the causative microorganism. The definitive way to determine the causative agent is by bone biopsy or biopsy of the soft tissue surrounding the site. The other tests may help to establish the diagnosis but do not identify the causative agent.

A triage nurse in the ED admits a 50 year old male client with second degree burns on the anterior and posterior portions of both legs. Based on the Rule of Nines, what percentage of his body is burned? Record your answer using a whole number.

36 The anterior and posterior portions of one leg are 18%, if both legs are burned, the total is 36%.

After teaching the client with rheumatoid arthritis about measures to conserve energy in activities of daily living involving the small joints, which of the following, if stated by the client, would indicate the need for additional teaching? 1. Pushing with palms when rising from a chair. 2. Holding packages close to the body. 3. Sliding objects. 4. Carrying a laundry basket with clinched fingers and fists.

4. Carrying a laundry basket with clinched fingers and fists is not an example of conserving energy of small joints. The laundry basket should be held with both hands opened as wide as possible and with outstretched arms so that pressure is not placed on the small joints of the fingers. When rising from a chair, the palms should be used instead of the fingers so as to distribute weight over the larger area of the palms. Holding packages close to the body provides greater support to the shoulder, elbow, and wrist joints because muscles of the arms and hands are used to stabilize the weight against the body. This decreases the stress and weight or pull on small joints such as the fingers. Objects can be slid with the palm of the hand, which distributes weight over the larger area of the palms instead of stressing the small joints of the fingers to pick up the weight of the object to move it to another place.

Which of the following should the nurse assess when completing the history and physical examination of a client diagnosed with osteoarthritis? 1. Anemia. 2. Osteoporosis. 3. Weight loss. 4. Local joint pain.

4. Osteoarthritis is a degenerative joint disease with local manifestations such as local joint pain, unlike rheumatoid arthritis, which has systemic manifestation such as anemia and osteoporosis. Weight loss occurs in rheumatoid arthritis, whereas most clients with osteoarthritis are overweight.

The teaching plan for the client with rheumatoid arthritis includes rest promotion. Which of the following would the nurse expect to instruct the client to avoid during rest periods? 1. Proper body alignment. 2. Elevating the part. 3. Prone lying positions. 4. Positions of flexion.

4. Positions of flexion should be avoided to prevent loss of functional ability of affected joints. Proper body alignment during rest periods is encouraged to maintain correct muscle and joint placement. Lying in the prone position is encouraged to avoid further curvature of the spine and internal rotation of the shoulders.

A client with rheumatoid arthritis tells the nurse, "I know it is important to exercise my joints so that I won't lose mobility, but my joints are so stiff and painful that exercising is difficult." Which of the following responses by the nurse would be most appropriate? 1. "You are probably exercising too much. Decrease your exercise to every other day." 2. "Tell the physician about your symptoms. Maybe your analgesic medication can be increased." 3. "Stiffness and pain are part of the disease. Learn to cope by focusing on activities you enjoy." 4. "Take a warm tub bath or shower before exercising. This may help with your discomfort."

4. Superficial heat applications, such as tub baths, showers, and warm compresses, can be helpful in relieving pain and stiffness. Exercises can be performed more comfortably and more effectively after heat applications. The client with rheumatoid arthritis must balance rest with exercise every day, not every other day. Typically, large doses of analgesics, which can lead to hepatotoxic effects, are not necessary. Learning to cope with the pain by refocusing is inappropriate.

9. What does radicular pain that radiates down the buttock and below the knee, along the distribution of the sciatic nerve, generally indicate? a. Cervical disc herniation b. Acute lumbosacral strain c. Degenerative disc disease d. Herniated intervertebral disc

9. d. Intervertebral disc herniation is generally indicated by radicular pain radiating down the buttock, below the knee, and along the distribution of the sciatic nerve. Cervical disc disease has pain radiating into the arms and hands. Acute lumbosacral strain causes acute low back pain. Degenerative disc disease is a structural degeneration of discs that is a normal process of aging and results in intervertebral discs losing their elasticity, flexibility, and shock-absorbing capabilities.

2. When teaching a patient infected with HIV regarding transmission of the virus to others, which statement made by the patient would indicate a need for further teaching? A. "I will need to isolate any tissues I use so as not to infect my family." B. "I will notify all of my sexual partners so they can get tested for HIV." C. "Unprotected sexual contact is the most common mode of transmission." D. "I do not need to worry about spreading this virus to others by sweating at the gym."

A HIV is not spread casually. The virus cannot be transmitted through hugging, dry kissing, shaking hands, sharing eating utensils, using toilet seats, or attending school with an HIV-infected person. It is not transmitted through tears, saliva, urine, emesis, sputum, feces, or sweat.

17. A patient has acquired immunodeficiency syndrome (AIDS) and the viral load is reported as undetectable. What patient teaching should be provided by the nurse related to this laboratory study result? A. The patient has the virus present and can transmit the infection to others. B. The patient is not able to transmit the virus to others through sexual contact. C. The patient will be prescribed lower doses of antiretroviral medications for 2 months. D. The syndrome has been cured, and the patient will be able to discontinue all medications.

A In human immunodeficiency virus (HIV) infections, viral loads are reported as real numbers of copies/μL or as undetectable. "Undetectable" indicates that the viral load is lower than the test is able to report. "Undetectable" does not mean that the virus has been eliminated from the body or that the individual can no longer transmit HIV to others.

11. An older adult patient is brought to the primary health care provider by an adult child reporting confusion. What testing should the nurse anticipate obtaining from this patient? A. Urinalysis B. Sputum culture C. Red blood cell count D. White blood cell count

A The developments of urinary tract infections commonly contribute to atypical manifestations such as cognitive and behavior changes in older adults. Sputum culture, red blood cell count, and white blood cell count may be done, but the first step would be to assess for a possible urinary tract infection.

8. A patient has been diagnosed with human immunodeficiency virus (HIV) infection. What rationale for taking more than one antiretroviral medication should the nurse give to the patient to improve compliance? A. Viral replication will be inhibited. B. They will decrease CD4+ T cell counts. C. It will prevent interaction with other drugs. D. More than one drug has a better chance of curing HIV.

A The major advantage of using several classes of antiretroviral drugs is that viral replication can be inhibited in several ways, making it more difficult for the virus to recover and decreasing the likelihood of drug resistance that is a major problem with monotherapy. Combination therapy also delays disease progression and decreases HIV symptoms and opportunistic diseases. HIV cannot be cured. CD4+ T-cell counts increase with therapy. There are dangerous interactions with many antiretroviral drugs and other commonly used drugs.

12. A patient who is infected with human immunodeficiency virus (HIV) is being taught by the nurse about health promotion activities such as good nutrition; avoiding alcohol, tobacco, drug use, and exposure to infectious agents; keeping up to date with vaccines; getting adequate rest; and stress management. What is the rationale behind these interventions that the nurse knows? A. Delaying disease progression B. Preventing disease transmission C. Helping to cure the HIV infection D. Enabling an increase in self-care activities

A These health promotion activities along with mental health counseling, support groups, and a therapeutic relationship with health care providers will promote a healthy immune system, which may delay disease progression. These measures will not cure HIV infection, prevent disease transmission, or increase self-care activities.

Physiological Integrity 29. When caring for a patient who has just arrived on the medical-surgical unit after having cardiac catheterization, which nursing intervention should the nurse delegate to a licensed practical/vocational nurse (LPN/LVN)? a. Give the scheduled aspirin and lipid-lowering medication. b. Perform the initial assessment of the catheter insertion site. c. Teach the patient about the usual postprocedure plan of care. d. Titrate the heparin infusion according to the agency protocol.

A Administration of oral medications is within the scope of practice for LPNs/LVNs. The initial assessment of the patient, patient teaching, and titration of IV anticoagulant medications should be done by the registered nurse (RN). DIF: Cognitive Level: Apply (application) REF: 758 OBJ: Special Questions: Delegation TOP: Nursing Process: Planning MSC:

The nurse is scheduled to administer seasonal influenza vaccinations to the residents of a long-term care facility. What would be a contraindication to the administration of the vaccine to a resident? A. Hypersensitivity to eggs B. Age older than 80 years C. History of upper respiratory infections D. Chronic obstructive pulmonary disease (COPD)

A Although current vaccines are highly purified and reactions are extremely uncommon, a hypersensitivity to eggs precludes vaccination because the vaccine is produced in eggs. Advanced age and a history of respiratory illness are not contraindications for influenza vaccination.

The nurse observes clear nasal drainage in a patient newly admitted with facial trauma with a nasal fracture. What is the nurse's priority action? A. Test the drainage for the presence of glucose. B. Suction the nose to maintain airway clearance. C. Document the findings and continue monitoring. D. Apply a drip pad and reassure the patient this is normal.

A Clear nasal drainage suggests leakage of cerebrospinal fluid (CSF). The drainage should be tested for the presence of glucose, which would indicate the presence of CSF. Suctioning should not be done. Documenting the findings and monitoring are important after notifying the health care provider. A drip pad may be applied, but the patient should not be reassured that this is normal.

Physiological Integrity 34. Which information about a patient who has been receiving thrombolytic therapy for an acute myocardial infarction (AMI) is most important for the nurse to communicate to the health care provider? a. No change in the patient's chest pain b. An increase in troponin levels from baseline c. A large bruise at the patient's IV insertion site d. A decrease in ST-segment elevation on the electrocardiogram

A Continued chest pain suggests that the thrombolytic therapy is not effective and that other interventions such as percutaneous coronary intervention (PCI) may be needed. Bruising is a possible side effect of thrombolytic therapy, but it is not an indication that therapy should be discontinued. The decrease of the ST-segment elevation indicates that thrombolysis is occurring and perfusion is returning to the injured myocardium. An increase in troponin levels is expected with reperfusion and is related to the washout of cardiac markers into the circulation as the blocked vessel is opened. DIF: Cognitive Level: Apply (application) REF: 752 OBJ: Special Questions: Prioritization TOP: Nursing Process: Evaluation MSC:

Physiological Integrity 27. A patient who is being admitted to the emergency department with intermittent chest pain gives the following list of medications to the nurse. Which medication has the most immediate implications for the patient's care? a. Sildenafil (Viagra) b. Furosemide (Lasix) c. Captopril (Capoten) d. Warfarin (Coumadin)

A The nurse will need to avoid giving nitrates to the patient because nitrate administration is contraindicated in patients who are using sildenafil because of the risk of severe hypotension caused by vasodilation. The other home medications also should be documented and reported to the health care provider but do not have as immediate an impact on decisions about the patient's treatment. DIF: Cognitive Level: Apply (application) REF: 745 OBJ: Special Questions: Prioritization TOP: Nursing Process: Assessment MSC:

Physiological Integrity 19. Three days after experiencing a myocardial infarction (MI), a patient who is scheduled for discharge asks for assistance with hygiene activities, saying, "I am too nervous to take care of myself." Based on this information, which nursing diagnosis is appropriate? a. Ineffective coping related to anxiety b. Activity intolerance related to weakness c. Denial related to lack of acceptance of the MI d. Disturbed personal identity related to understanding of illness

A The patient data indicate that ineffective coping after the MI caused by anxiety about the impact of the MI is a concern. The other nursing diagnoses may be appropriate for some patients after an MI, but the data for this patient do not support denial, activity intolerance, or disturbed personal identity. DIF: Cognitive Level: Apply (application) REF: 757 TOP: Nursing Process: Diagnosis MSC:

A nurse assesses a 38-year-old patient with joint pain and stiffness who was diagnosed with Stage III rheumatoid arthritis (RA). What characteristics should the nurse expect to observe (select all that apply)? A Nodules present B Consistent muscle strength C Localized disease symptoms D No destructive changes on x-ray E Subluxation of joints without fibrous ankylosis

A nodules present E Subluxation of joints without fibrous ankylosis

C

A 68-yr-old patient with bronchiectasis has copious thick respiratory secretions. Which intervention should the nurse add to the plan of care for this patient? Use the incentive spirometer for at least 10 breaths every 2 hours. Administer prescribed antibiotics and antitussives on a scheduled basis. Increase intake to at least 12 eight-ounce glasses of fluid every 24 hours. Provide nutritional supplements that are high in protein and carbohydrates.

When caring for elderly patients with hypertension, which information should the nurse consider when planning care (select all that apply.)? A. Systolic blood pressure increases with aging. B. Blood pressures should be maintained near 120/80 mm Hg. C. White coat syndrome is prevalent in elderly patients. D. Volume depletion contributes to orthostatic hypotension. E. Blood pressure drops 1 hour postprandially in many older patients. F. Older patients will require higher doses of antihypertensive medications.

A C D E Systolic blood pressure increases with age and patients older than age 60 years should be maintained below 150/90 mm Hg. Older patients have significantly higher blood pressure readings when taken by health care providers (white coat syndrome). Older patients experience orthostatic hypotension related to dehydration, reduced compensatory mechanisms, and medications. One hour after eating, many older patients experience a drop in blood pressure. Lower doses of medications may be needed to control blood pressures in older adults related to decreased absorption rates and excretion ability.

D

A patient has been receiving oxygen per nasal cannula while hospitalized for chronic obstructive pulmonary disease (COPD). The patient asks the nurse whether oxygen use will be needed at home. What is the most appropriate response by the nurse? "Long-term home oxygen therapy should be used to prevent respiratory failure." "Oxygen will not be needed until or unless you are in the terminal stages of this disease." "Long-term home oxygen therapy should be used to prevent heart problems related to COPD." "You will not need oxygen until your oxygen saturation drops to 88% and you have symptoms of hypoxia."

B

A patient with an acute exacerbation of chronic obstructive pulmonary disease (COPD) needs to receive precise amounts of oxygen. Which equipment should the nurse prepare to use? Oxygen tent Venturi mask Nasal cannula Oxygen-conserving cannula

A 54-year-old patient with acute osteomyelitis asks the nurse how this problem will be treated. Which response by the nurse is most appropriate? A. "IV antibiotics are usually required for several weeks." B. "Oral antibiotics are often required for several months." C. "Surgery is almost always necessary to remove the dead tissue that is likely to be present." D. "Drainage of the foot and instillation of antibiotics into the affected area is the usual therapy."

A. "IV antibiotics are usually required for several weeks." The standard treatment for acute osteomyelitis consists of several weeks of IV antibiotic therapy. This is because bone is denser and less vascular than other tissues, and it takes time for the antibiotic therapy to eradicate all of the microorganisms. Surgery may be used for chronic osteomyelitis, which may include debridement of the devitalized and infected tissue and irrigation of the affected bone with antibiotics.

When preparing a patient for a capsule endoscopy study, what should the nurse do? A) Ensure the patient understands the required bowel preparation. B) Have the patient return to the procedure room for removal of the capsule. C) Teach the patient to maintain a clear liquid diet throughout the procedure. D) Explain to the patient that conscious sedation will be used during placement of the capsule.

A) A capsule endoscopy study involves the patient performing a bowel prep to cleanse the bowel before swallowing the capsule. The patient will be on a clear liquid diet for 1 to 2 days before the procedure and will remain NPO for 4 to 6 hours after swallowing the capsule. The capsule is disposable and will pass naturally with the bowel movement, although the monitoring device will need to be removed.

After administering a dose of promethazine (Phenergan) to a patient with nausea and vomiting, the nurse explains that which of the following may be experienced as a common temporary adverse effect of the medication? A) Drowsiness B) Reduced hearing C) Sensation of falling D) Photosensitivity

A) Drowsiness (Although being given to this patient as an antiemetic, promethazine also has sedative and amnesic properties. For this reason, the patient is likely to experience drowsiness as an adverse effect of the medication.)

A patient had a stomach resection for stomach cancer. The nurse should teach the patient about the loss of the hormone that stimulates gastric acid secretion and motility and maintains lower esophageal sphincter tone. Which hormone will be decreased with a gastric resection? A) Gastrin B) Secretin C) Cholecystokinin D) Gastric inhibitory peptide

A) Gastrin is the hormone activated in the stomach (and duodenal mucosa) by stomach distention that stimulates gastric acid secretion and motility and maintains lower esophageal sphincter tone. Secretin inhibits gastric motility and acid secretion and stimulates pancreatic bicarbonate secretion. Cholecystokinin allows increased flow of bile into the duodenum and release of pancreatic digestive enzymes. Gastric inhibitory peptide inhibits gastric acid secretion and motility.

The patient with cirrhosis is being taught self-care. Which statement indicates the patient needs more teaching? A) "If I notice a fast heart rate or irregular beats, this is normal for cirrhosis." B) "I need to take good care of my belly and ankle skin where it is swollen." C) "A scrotal support may be more comfortable when I have scrotal edema." D) "I can use pillows to support my head to help me breathe when I am in bed."

A) If the patient with cirrhosis experiences a fast or irregular heart rate, it may be indicative of hypokalemia and should be reported to the health care provider, as this is not normal for cirrhosis. Edematous tissue is subject to breakdown and needs meticulous skin care. Pillows and a semi-Fowler's or Fowler's position will increase respiratory efficiency. A scrotal support may improve comfort if there is scrotal edema

The wound, ostomy, and continence (WOC) nurse selects the site where the ostomy will be placed. What should be included in the consideration for the site? A) The patient must be able to see the site. B) Outside the rectus muscle area is the best site. C) It is easier to seal the drainage bag to a protruding area. D) The ostomy will need irrigation, so area should not be tender.

A) In selection of the ostomy site, the WOC nurse will want a site visible to the patient so the patient can take care of it, within the rectus muscle to avoid hernias, and on a flat surface to more easily create a good seal with the drainage bag.

The nurse is conducting discharge teaching for a patient with metastatic lung cancer who was admitted with a bowel impaction. Which of the following instructions would be most helpful to prevent further episodes of constipation? A) Maintain a high intake of fluid and fiber in the diet. B) Reduce intake of medications causing constipation. C) Eat several small meals per day to maintain bowel motility. D) Sit upright during meals to increase bowel motility by gravity.

A) Increased fluid intake and a high-fiber diet reduce the incidence of constipation caused by immobility, medications, and other factors. Fluid and fiber provide bulk that in turn increases peristalsis and bowel motility. Analgesics taken for lung cancer probably cannot be reduced. Other medications may decrease constipation, but it is best to avoid laxatives. Eating several small meals per day and position do not facilitate bowel motility. Defecation is easiest when the person sits on the commode with the knees higher than the hips.

A stroke patient who primarily uses a wheelchair for mobility has diarrhea with fecal incontinence. What should the nurse assess first? A) Fecal impaction B) Perineal hygiene C) Dietary fiber intake D) Antidiarrheal agent use

A) Patients with limited mobility are at risk for fecal impactions due to constipation that may lead to liquid stool leaking around the hardened impacted feces, so assessing for fecal impaction is the priority. Perineal hygiene can be assessed at the same time. Assessing the dietary fiber and fluid intake and antidiarrheal agent use will be assessed and considered next.

18. What should the nurse teach the patients in the assisted living facility to decrease their risk for antibiotic-resistant infection (select all that apply.)? A. Wash hands frequently. B. Take antibiotics as prescribed. C. Take the antibiotic until it is gone. D. Take antibiotics to prevent illnesses like colds. E. Save leftover antibiotics to take if needed later.

A, B, C To decrease the risk for antibiotic-resistant infections, people should wash their hands frequently, follow the directions when taking the antibiotics, finish the antibiotic, do not request antibiotics for colds or flu, do not save leftover antibiotics, or take antibiotics to prevent an illness without them being prescribed by a health care provider.

Which individuals would be at high risk for low back pain (select all that apply)? a.A 63-year-old man who is a long-distance truck driver b.A 36-year-old 6 ft, 2 in construction worker who weighs 260 lb c.A 28-year-old female yoga instructor who is 5 ft, 6 in and weighs 130 lb d.A 30-year-old male nurse who works on an orthopedic unit and smokes e.A 44-year-old female chef with prior compression fracture of the spine

A, B, D, E Risk factors associated with low back pain include a lack of muscle tone and excess body weight, stress, poor posture, cigarette smoking, pregnancy, prior compression fractures of the spine, spinal problems since birth, and a family history of back pain. Jobs that require repetitive heavy lifting, vibration (such as a jackhammer operator), and prolonged periods of sitting are also associated with low back pain. Low back pain is most often caused by a musculoskeletal problem. The causes of low back pain of musculoskeletal origin include (1) acute lumbosacral strain, (2) instability of the lumbosacral bony mechanism, (3) osteoarthritis of the lumbosacral vertebrae, (4) degenerative disc disease, and (5) herniation of an intervertebral disc. Health care personnel are at high risk for the development of low back pain. Lifting and moving patients, excessive time being stooped over or leaning forward, and frequent twisting can result in low back pain.

The 40-year-old African American woman has had Raynaud's phenomenon for some time. She is now reporting red spots on the hands, forearms, palms, face, and lips. What other manifestations should the nurse assess for when she is assessing for scleroderma (select all that apply)? A) Calcinosis B) Weight loss C) Sclerodactyly D) Difficulty swallowing E) Weakened leg muscles

A, C, D) This 40-year-old African American woman is at risk for scleroderma. The acronym CREST represents the clinical manifestations. C: calcinosis, painful calcium deposits in the skin; R: Raynaud's phenomenon; E: Esophageal dysfunction, difficulty swallowing; S: sclerodactyly, tightening of skin on fingers and toes; T: telangiectasia. Weight loss and weakened leg muscles are associated with polymyositis and dermatomyositis not scleroderma.

A nurse assesses a 38-year-old patient with joint pain and stiffness who was diagnosed with Stage III rheumatoid arthritis (RA). What characteristics should the nurse expect to observe (select all that apply)? A) Nodules present B) Consistent muscle strength C) Localized disease symptoms D) No destructive changes on x-ray E) Subluxation of joints without fibrous ankylosis

A, E) In Stage III severe RA, there may be extraarticular soft tissue lesions or nodules present, and there is subluxation without fibrous or bony ankylosis. The muscle strength is decreased because there is extensive muscle atrophy. The manifestations are systemic not localized. There is x-ray evidence of cartilage and bone destruction in addition to osteoporosis.

When caring for a patient with a biliary obstruction, the nurse will anticipate administering which of the following vitamin supplements (select all that apply)? A. Vitamin A B. Vitamin D C. Vitamin E D. Vitamin K E. Vitamin B

A,B,C,D) Biliary obstruction prevents bile from entering the small intestine and thus prevents the absorption of fat-soluble vitamins. Vitamins A, D, E, and K are all fat-soluble and thus would need to be supplemented in a patient with biliary obstruction.

Which individuals would the nurse identify as having the highest risk for CAD? A. A 45-year-old depressed male with a high-stress job B. A 60-year-old male with below normal homocysteine levels C. A 54-year-old female vegetarian with elevated high-density lipoprotein (HDL) levels D. A 62-year-old female who has a sedentary lifestyle and body mass index (BMI) of 23 kg/m2

A. A 45-year-old depressed male with a high-stress job The 45-year-old depressed male with a high-stress job is at the highest risk for CAD. Studies demonstrate that depression and stressful states can contribute to the development of CAD. Elevated HDL levels and low homocysteine levels actually help to prevent CAD. Although a sedentary lifestyle is a risk factor, a BMI of 23 kg/m2 depicts normal weight, and thus the patient with two risk factors is at greatest risk for developing CAD.

When providing discharge teaching for the patient after a laparoscopic cholecystectomy, what information should the nurse include? A) A lower-fat diet may be better tolerated for several weeks. B) Do not return to work or normal activities for 3 weeks. C) Bile-colored drainage will probably drain from the incision. D) Keep the bandages on and the puncture site dry until it heals.

A. Although the usual diet can be resumed, a low-fat diet is usually better tolerated for several weeks following surgery. Normal activities can be gradually resumed as the patient tolerates. Bile-colored drainage or pus, redness, swelling, severe pain, and fever may all indicate infection. The bandage may be removed the day after surgery, and the patient can shower.

The nurse is admitting a patient to the nursing unit with a history of a herniated lumbar disc and low back pain. In completing a more thorough pain assessment, the nurse should ask the patient if which action aggravates the pain? A. Bending or lifting B. Application of warm moist heat C. Sleeping in a side-lying position D. Sitting in a fully extended recliner

A. Bending or lifting Back pain that is related to a herniated lumbar disc often is aggravated by events and activities that increase the stress and strain on the spine, such as bending or lifting, coughing, sneezing, and lifting the leg with the knee straight (straight leg-raising test). Application of moist heat, sleeping position, and ability to sit in a fully extended recliner do not aggravate the pain of a herniated lumbar disc.

The nurse is caring for a patient hospitalized with exacerbation of chronic bronchitis and herniated lumbar disc. Which breakfast choice would be most appropriate for the nurse to encourage the patient to check on the breakfast menu? A. Bran muffin B. Scrambled eggs C. Puffed rice cereal D. Buttered white toast

A. Bran muffin Each meal should contain one or more sources of fiber, which will reduce the risk of constipation and straining with defecation, which increases back pain. Bran is typically a high-fiber food choice and is appropriate for selection from the menu. Scrambled eggs, puffed rice cereal, and buttered white toast do not have as much fiber.

Evolve The 40-year-old African American woman has had Raynaud's phenomenon for some time. She is now reporting red spots on the hands, forearms, palms, face, and lips. What other manifestations should the nurse assess for when she is assessing for scleroderma (select all that apply)? A. Calcinosis B. Weight loss C. Sclerodactyly D. Difficulty swallowing E. Weakened leg muscles

A. Calcinosis C. Scelorodactyly D. Difficulty swallowing

Evolve The patient with fibromyalgia is suffering with pain at 12 of the 18 identification sites, including the neck and upper back and the knees. The patient also reports nonrefreshing sleep, depression, and being anxious when dealing with multiple tasks. The nurse should teach this patient about what treatments (select all that apply)? A. Low-impact aerobic exercise B. Relaxation strategy (biofeedback) C. Antiseizure drug pregabalin (Lyrica) D. Morphine sulfate extended-release tablets E. Serotonin reuptake inhibitor (e.g., sertraline [Zoloft])

A. Low-impact aerobic exercise B. Relaxation strategy (biofeedback) C. Antiseizure drug pregabalin (Lyrica) E. Serotonin reuptake inhibitor (e.g., sertraline [Zoloft])

When planning emergent care for a patient with a suspected MI, what should the nurse anticipate administrating? A. Oxygen, nitroglycerin, aspirin, and morphine B. Oxygen, furosemide (Lasix), nitroglycerin, and meperidine C. Aspirin, nitroprusside (Nipride), dopamine (Intropin), and oxygen D. Nitroglycerin, lorazepam (Ativan), oxygen, and warfarin (Coumadin)

A. Oxygen, nitroglycerin, aspirin, and morphine **MONA The American Heart Association's guidelines for emergency care of the patient with chest pain include the administration of oxygen, nitroglycerin, aspirin, and morphine. These interventions serve to relieve chest pain, improve oxygenation, decrease myocardial workload, and prevent further platelet aggregation. The other medications may be used later in the patient's treatment.

Which nursing intervention is most appropriate when turning a patient following spinal surgery? A. Placing a pillow between the patient's legs and turning the body as a unit B. Having the patient turn to the side by grasping the side rails to help turn over C. Elevating the head of bed 30 degrees and having the patient extend the legs while turning D. Turning the patient's head and shoulders and then the hips, keeping the patient's body centered in the bed

A. Placing a pillow between the patient's legs and turning the body as a unit Placing a pillow between the legs and turning the patient as a unit (logrolling) helps to keep the spine in good alignment and reduces pain and discomfort following spinal surgery. Having the patient turn by grasping the side rail to help, elevating the head of the bed, and turning with extended legs or turning the patient's head and shoulders and then the hips will not maintain proper spine alignment and may cause damage.

Evolve The nurse is caring for four newly diagnosed patients with various connective tissue disorders. The nurse should be most aware of safety issues and interstitial lung involvement in the patient with which diagnosis?

A. Polymyositis

A patient with OA asks the nurse whether he could try glucosamine and chondroitin for control of his symptoms. The best response by the nurse includes the information that a. some patients find these supplements helpful for relieving arthritis knee pain and improving mobility b. although these substances may not help, there is no evidence that they can cause any untoward effects c. these supplements are a fad that has not been shown to reduce pain or increase joint mobility in patients with OA d. only dosages of these supplements available by prescription are high enough to provide any benefit in treatment of OA

A. Some patients find these supplements helpful for relieving arthritis knee pain and improving mobilitiy

The community health nurse is planning health promotion teaching targeted at preventing coronary artery disease (CAD). Which ethnic group would the nurse select as the highest priority for this intervention? A. White male B. Hispanic male C. African American male D. Native American female

A. White male The incidence of CAD and myocardial infarction (MI) is highest among white, middle-aged men. Hispanic individuals have lower rates of CAD than non-Hispanic whites or African Americans. African Americans have an earlier age of onset and more severe CAD than whites and more than twice the mortality rate of whites of the same age. Native Americans have increased mortality in less than 35-year-olds and have major modifiable risk factors such as diabetes.

A school nurse is providing information to high school students about influenza prevention. What should the nurse emphasize in teaching to prevent the transmission of the virus (select all that apply.)? A. Cover the nose when coughing. B. Obtain an influenza vaccination. C. Stay at home when symptomatic. D. Drink noncaffeinated fluids daily. E. Obtain antibiotic therapy promptly.

ABC Covering the nose and mouth when coughing is an effective way to prevent the spread of the virus. Obtaining an influenza vaccination helps prevent the flu. Staying at home helps prevent direct exposure of others to the virus. Drinking fluids helps liquefy secretions but does not prevent influenza. Antibiotic therapy is not used unless the patient develops a secondary bacterial infection.

COMPLETION 1. The nurse obtains a blood pressure of 180/75 mm Hg for a patient. What is the patient's mean arterial pressure (MAP)? ____________________

ANS: 110 MAP = (SBP + 2 DBP)/3

A 63-yr-old patient who began experiencing right arm and leg weakness is admitted to the emergency department. In which order will the nurse implement these actions included in the stroke protocol? (Put a comma and a space between each answer choice [A, B, C, D].) a. Obtain CT scan without contrast. b. Infuse tissue plasminogen activator (tPA). c. Administer oxygen to keep O2 saturation >95%. d. Use National Institute of Health Stroke Scale to assess patient.

ANS: C, D, A, B The initial actions should be those that help with airway, breathing, and circulation. Baseline neurologic assessments should be done next. A CT scan will be needed to rule out hemorrhagic stroke before tPA can be administered. DIF: Cognitive Level: Analyze (analysis) REF: 1354 OBJ: Special Questions: Prioritization TOP: Nursing Process: Implementation MSC: NCLEX: Physiological Integrity

A patient with neurogenic shock after a spinal cord injury is to receive lactated Ringer's solution 400 mL over 20 minutes. When setting the IV pump to deliver the IV fluid, the nurse will set the rate at how many milliliters per hour?

ANS: 1200 To administer 400 mL in 20 minutes, the nurse will need to set the pump to run at 1200 mL/hour. DIF: Cognitive Level: Understand (comprehension)

B

The nurse is teaching a patient how to self-administer ipratropium via a metered-dose inhaler (MDI). Which instruction given by the nurse is most appropriate to help the patient learn the proper inhalation technique? "Avoid shaking the inhaler before use." "Breathe out slowly before positioning the inhaler." "Using a spacer should be avoided for this type of medication." "After taking a puff, hold the breath for 30 seconds before exhaling."

D

The nurse supervises a team including another registered nurse (RN), a licensed practical/vocational nurse (LPN/LVN), and unlicensed assistive personnel (UAP) on a medical unit. The team is caring for many patients with respiratory problems. In what situation should the nurse intervene with teaching for a team member? LPN/LVN obtained a pulse oximetry reading of 94% but did not report it. RN taught the patient about home oxygen safety in preparation for discharge. UAP report to the nurse that the patient is complaining of difficulty breathing. LPN/LVN changed the type of oxygen device based on arterial blood gas results.

B

The nurse teaches a 53-yr-old male patient with chronic obstructive pulmonary disease (COPD) how to administer fluticasone by metered-dose inhaler (MDI). Which statement by the patient to the nurse indicates correct understanding of the instructions? "I should not use a spacer device with this inhaler." "I will rinse my mouth each time after I use this inhaler." "I will feel my breathing improve over the next 2 to 3 days." "I should use this inhaler immediately if I have trouble breathing."

C

The nurse teaches pursed-lip breathing to a patient who is newly diagnosed with chronic obstructive pulmonary disease (COPD). The nurse reinforces that this technique will assist respiration by which mechanism? Loosening secretions so that they may be coughed up more easily Promoting maximal inhalation for better oxygenation of the lungs Preventing bronchial collapse and air trapping in the lungs during exhalation Increasing the respiratory rate and giving the patient control of respiratory patterns

A

The physician has prescribed salmeterol (Serevent) for a patient with asthma. In reviewing the use of dry powder inhalers (DPIs) with the patient, what instructions should the nurse provide? "Close lips tightly around the mouthpiece and breathe in deeply and quickly." "To administer a DPI, you must use a spacer that holds the medicine so that you can inhale it." "You will know you have correctly used the DPI when you taste or sense the medicine going into your lungs." "Hold the inhaler several inches in front of your mouth and breathe in slowly, holding the medicine as long as possible."

Which characteristic will the nurse associate with a focal seizure? The patient lost consciousness during the seizure. The seizure involved both sides of the patient's brain. The seizure involved lip smacking and repetitive movements. The patient fell to the ground and became stiff for 20 seconds.

The seizure involved lip smacking and repetitive movements. Complex *focal seizure* is characterized commonly by *lip smacking and automatisms* (repetitive movements that may not be appropriate).

Which care measure is a priority for a patient with multiple sclerosis (MS)? Vigilant infection control and adherence to standard precautions Careful monitoring of neurologic assessment and frequent reorientation Maintenance of a calorie count and hourly assessment of intake and output Assessment of blood pressure and monitoring for signs of orthostatic hypotension

Vigilant infection control and adherence to standard precautions Infection control is a priority in the care of patients with MS because infection is the most common cause of an exacerbation of the disease

ABCE

When admitting a patient with a diagnosis of asthma exacerbation, the nurse will assess for what potential triggers (select all that apply.)? Exercise Allergies Emotional stress Decreased humidity Upper respiratory infections

B

When caring for a patient with chronic obstructive pulmonary disease (COPD), the nurse identifies a nursing diagnosis of imbalanced nutrition: less than body requirements after noting a weight loss of 30 lb. Which intervention should the nurse add to the plan of care for this patient? Order fruits and fruit juices to be offered between meals. Order a high-calorie, high-protein diet with six small meals a day. Teach the patient to use frozen meals at home that can be microwaved. Provide a high-calorie, high-carbohydrate, nonirritating, frequent feeding diet.

A

When teaching the patient with bronchiectasis about manifestations to report to the health care provider, which manifestation should be included? Increasing dyspnea Temperature below 98.6°F Decreased sputum production Unable to drink 3 L of low-sodium fluids

A

While teaching a patient with asthma about the appropriate use of a peak flow meter, what should the nurse instruct the patient to do? Keep a record of the peak flow meter numbers if symptoms of asthma are getting worse. Use the flow meter each morning after taking medications to evaluate their effectiveness. Increase the doses of the long-term control medication if the peak flow numbers decrease. Empty the lungs and then inhale quickly through the mouthpiece to measure how fast air can be inhaled.

5. A patient with newly discovered high BP has an average reading of 158/98 mm Hg after 3 months of exercise and diet modifications. Which management strategy will be a priority for this patient? a. Medication will be required because the BP is still not at goal. b. BP monitoring should continue for another 3 months to confirm a diagnosis of hypertension. c. Lifestyle changes are less important, since they were not effective, and medications will be started. d. More vigorous changes in the patient's lifestyle are needed for a longer time before starting medications.

a

Lewis Teach the patient with fibromyalgia the importance of limiting intake of which foods (select all that apply)? a. Sugar b. Alcohol c. Caffeine d. Red meat e. Root vegetables

a, b, c

A patient with *Parkinson's disease* is admitted to the hospital for *treatment of pneumonia*. Which nursing interventions will be included in the plan of care (select all that apply)? a. Provide an elevated toilet seat. b. Cut patient's food into small pieces. c. Serve high-protein foods at each meal. d. Place an armchair at the patient's bedside. e. Observe for sudden exacerbation of symptoms.

a, b, d a. Provide an elevated toilet seat. b. Cut patient's food into small pieces. d. Place an armchair at the patient's bedside. Because the patient with Parkinson's disease has difficulty chewing, food should be cut into small pieces. An armchair should be used when the patient is seated so that the patient can use the arms to assist with getting up from the chair. An elevated toilet seat will facilitate getting on and off the toilet. High-protein foods will decrease the effectiveness of L-dopa. Parkinson's disease is a steadily progressive disease without acute exacerbations.

1. A patient who has been treated for *status epilepticus* in the emergency department will be transferred to the medical nursing unit. Which *equipment* should the nurse have available in the patient's assigned room (select all that apply)? a. Side-rail pads b. Tongue blade c. Oxygen mask d. Suction tubing e. Urinary catheter f. Nasogastric tube

a, c, d a. Side-rail pads c. Oxygen mask d. Suction tubing The patient is at risk for further seizures, and O2 and suctioning may be needed after any seizures to clear the airway and maximize oxygenation.

The nurse is planning care for a patient with partial- and full-thickness skin destruction related to burn injury of the lower extremities. Which of the following interventions would the nurse expect to include in this patient's care (select all that apply)? A.Escharotomy B.Administration of diuretics C.IV and oral pain medications D.Daily cleansing and debridement E.Application of topical antimicrobial agent

a, c, d, e. An escharotomy (a scalpel incision through full-thickness eschar) is frequently required to restore circulation to compromised extremities. Daily cleansing and debridement as well as application of an antimicrobial ointment are expected interventions used to minimize infection and enhance wound healing. With full-thickness burns, myoglobin and hemoglobin released into the bloodstream can occlude renal tubules. Adequate fluid replacement is used to prevent this occlusion. Pain control is essential in the care of a patient with a burn injury

A patient is admitted to the emergency department with first- and second-degree burns after being involved in a house fire. Which of the following assessment findings would alert you to the presence of an inhalation injury (select all that apply)? A.Singed nasal hair B.Generalized pallor C.Painful swallowing D.Burns on the upper extremities E.History of being involved in a large fire

a, c. Reliable clues to the occurrence of inhalation injury is the presence of facial burns, singed nasal hair, hoarseness, painful swallowing, darkened oral and nasal membranes, carbonaceous sputum, history of being burned in an enclosed space, and cherry red skin color

When assessing a pt with a partial thickness burn, the nurse would expect to find (select all that apply) a. blisters b. exposed fascia c. exposed muscles d. intact nerve endings e. red, shiny, wet appearance

a, d, e

1. Which BP-regulating mechanism(s) can result in the development of hypertension if defective (select all that apply)? a. Release of norepinephrine b. Secretion of prostaglandins c. Stimulation of the sympathetic nervous system d. Stimulation of the parasympathetic nervous system e. Activation of the renin-angiotensin-aldosterone system

a,c,e

6. A patient is admitted to the hospital in hypertensive emergency (BP 244/142 mm Hg). Sodium nitroprusside is started to treat the elevated BP. Which management strategy(ies) would be most appropriate for this patient (select all that apply)? a. Measuring hourly urine output b. Decreasing the MAP by 50% within the first hour c. Continuous BP monitoring with an arterial line d. Maintaining bed rest and providing tranquilizers to lower the BP e. Assessing the patient for signs and symptoms of heart failure and changes in mental status

a,c,e

The injury that is least likely to result in a full thickness burn is: a. sunburn b. scald injury c. chemical burn d. electrical injury

a.

20. The 20 year old female client diagnosed with advanced unremitting RA is being admitted to receive a regimen of immunosuppressive medications. Which question should the nurse ask during the admission process regarding the medications? a. "Are you sexually active, and if so, are you using birth control?" b. "Have you discussed taking these drugs with your parents?" c. "Which arm do you prefer to have an IV in for 4 days?" d. "Have you signed an informed consent for investigational drugs?"

a. "Are you sexually active, and if so, are you using birth control?"

The nurse is preparing to insert a nasogastric (NG) tube into a 68-year-old female patient who is nauseated and vomiting. She has an abdominal mass and suspected small intestinal obstruction. The patient asks the nurse why this procedure is necessary. What response by the nurse is most appropriate? a. "The tube will help to drain the stomach contents and prevent further vomiting." b. "The tube will push past the area that is blocked and thus help to stop the vomiting." c. "The tube is just a standard procedure before many types of surgery to the abdomen." d. "The tube will let us measure your stomach contents so that we can plan what type of IV fluid replacement would be best."

a. "The tube will help to drain the stomach contents and prevent further vomiting." The NG tube is used to decompress the stomach by draining stomach contents and thereby prevent further vomiting. The NG tube will not push past the blocked area. Potential surgery is not currently indicated. The location of the obstruction will determine the type of fluid to use, not measure the amount of stomach contents.

1. A patient is concerned that he may have asthma. Of the symptoms that he relates to the nurse, which ones suggest asthma or risk factors for asthma (select all that apply)? a. Allergic rhinitis b. Prolonged inhalation c. History of skin allergies d. Cough, especially at night e. Gastric reflux or heartburn

a. Allergic rhinitis c. History of skin allergies d. Cough, especially at night e. Gastric reflux or heartburn

4. The occupational health nurse is teaching a class on the risk factors for developing OA. Which is a modifiable risk for developing OA? a. Being overweight b. Increasing age c. Previous joint damage d. Genetic susceptibility

a. Being overweight

8. A patient who has bronchiectasis asks the nurse, "What conditions would warrant a call to the clinic?" a. Blood clots in the sputum b. Sticky sputum on a hot day c. Increased shortness of breath after eating a large meal d. Production of large amounts of sputum on a daily basis

a. Blood clots in the sputum

A patient tells the nurse about using acetaminophen (Tylenol) several times every day for recurrent bilateral headaches. Which action will the nurse plan to take first? a. Discuss the need to stop taking the acetaminophen. b. Suggest the use of biofeedback for headache control. c. Describe the use of botulism toxin (Botox) for headaches. d. Teach the patient about magnetic resonance imaging (MRI).

a. Discuss the need to stop taking the acetaminophen. The headache description suggests that the patient is experiencing *medication overuse headache*.

The nurse is preparing to care for a burn client scheduled for an escharotomy procedure being performed for a third degree circumferential arm burn. The nurse understands that the anticipated therapeutic outcome of the escharotomy is: a. return of distal pulses b. brisk bleeding from the site c. decreasing edema formation d. formation of granulation tissue

a. Escharotomies arepreformed to relieve the compartment syndrome that can occur when edema forms under nondistensible eschar in a circumferential third degree burn.

A stroke patient who primarily uses a wheelchair for mobility has diarrhea with fecal incontinence. What should the nurse assess first? a. Fecal impaction b. Perineal hygiene c. Dietary fiber intake d. Antidiarrheal agent use

a. Fecal impaction Patients with limited mobility are at risk for fecal impactions due to constipation that may lead to liquid stool leaking around the hardened impacted feces, so assessing for fecal impaction is the priority. Perineal hygiene can be assessed at the same time. Assessing the dietary fiber and fluid intake and antidiarrheal agent use will be assessed and considered next.

A patient has been taking phenytoin (Dilantin) for 2 years. Which action will the nurse take when evaluating for adverse effects of the medication? a. Inspect the oral mucosa. c. Auscultate the bowel sounds. b. Listen to the lung sounds. d. Check pupil reaction to light.

a. Inspect the oral mucosa. *Phenytoin* can cause *gingival hyperplasia*, but does not affect bowel sounds, lung sounds, or pupil reaction to light.

22. The nurse is preparing to administer morning medications. Which medication should the nurse administer first? a. The pain medication to a client diagnosed with RA b. The diuretic medication to a client diagnosed with Lupus (SLE) c. The steroid to a client diagnosed with polymyositis d. The appetite stimulant to a client diagnosed with OA

a. The pain medication to a client diagnosed with RA

The wound, ostomy, and continence (WOC) nurse selects the site where the ostomy will be placed. What should be included in the consideration for the site? a. The patient must be able to see the site. b. Outside the rectus muscle area is the best site. c. It is easier to seal the drainage bag to a protruding area. d. The ostomy will need irrigation, so area should not be tender.

a. The patient must be able to see the site. In selection of the ostomy site, the WOC nurse will want a site visible to the patient so the patient can take care of it, within the rectus muscle to avoid hernias, and on a flat surface to more easily create a good seal with the drainage bag.

A male burn pt who was struck by lightning arrives at the emergency department with full thickness burns to the arms and chest and with a cervical collar in place. Which assessment finding is the nurse's priority? a. serum K+ of 5.6 mEq/L b. Arterial blood ph of 7.35 c. Cervical spine fracture d. hemoglobin 18g/dL

a. The pt's potassium level puts them at risk for life threatening cardiac dysrhythmias.

The nurse explains to the patient undergoing ostomy surgery that the procedure that maintains the most normal functioning of the bowel is: a. a sigmoid colostomy b. a transverse colostomy c. a descending colostomy d. an ascending colostomy

a. a sigmoid colostomy Rationale: The more distal the ostomy is, the more the intestinal contents resemble feces eliminated from an intact colon and rectum. Output from a sigmoid colostomy resembles normally formed stool, and some patients are able to regulate emptying time so they do not need to wear a collection bag.

A therapeutic measure used to prevent hypertrophic scarring during the rehabilitative phase of burn recovery is: a. applying pressure garments. b. repositioning the pt every 2 hours c. performing active ROM at least every 4 hours d. massaging the new tissue with water based moisturizers

a. applying pressure garments. Pressure can help keep a scar flat and reduce hypertrophic scarring. Gentle pressure can be maintained on the healed burn with custom-fitted pressure garments.

The nurse observes a patient ambulating in the hospital hall when the patient's arms and legs suddenly jerk and the patient falls to the floor. The nurse will *first*: a. assess the patient for a possible injury. b. give the scheduled divalproex (Depakote). c. document the timing and description of the seizure. d. notify the patient's health care provider about the seizure.

a. assess the patient for a possible injury.

A child was admitted to the ED with a thermal burn to the right arm and leg. Which assessment by the nurse requires immediate action? a. coughing and wheezing b. bright red skin with small blister on the burn sites c. thirst d. singed hair

a. coughing and wheezing may indicate that the child has inhaled smoke or toxic fumes. Maintaining airway patency is the highest nursing priority in this situation.

3. A plan of care for the patient with COPD could include (select all that apply) a. exercise such as walking. b. high flow rate of O2 administration. c. low-dose chronic oral corticosteroid therapy. d. use of peak flow meter to monitor the progression of COPD. e. breathing exercises such as pursed-lip breathing that focus on exhalation.

a. exercise such as walking e. breathing exercises such as pursed-lip breathing that focus on exhalation.

Two days following a colectomy for an abdominal mass, a patient reports gas pains and abdominal distention. The nurse plans care for the patient based on the knowledge that the symptoms are occurring as a result of a. impaired peristalsis. b. irritation of the bowel. c. nasogastric suctioning. d. inflammation of the incision site.

a. impaired peristalsis. Until peristalsis returns to normal following anesthesia, the patient may experience slowed gastrointestinal motility leading to gas pains and abdominal distention. Irritation of the bowel, nasogastric suctioning, and inflammation of the surgical site do not cause gas pains or abdominal distention.

A client is admitted to a burn intensive care unit with extensive full thickness burns. What should be the nurse's initial concern? a. fluid status b. risk for infection c. body image d. level of pain

a. in early burn care, the client's greatest need has to do with fluid resuscitation because of large volume fluid loss through the damaged skin.

A

The nurse is caring for a patient with an acute exacerbation of asthma. After initial treatment, what finding indicates to the nurse that the patient's respiratory status is improving? Wheezing becomes louder. Cough remains nonproductive. Vesicular breath sounds decrease. Aerosol bronchodilators stimulate coughing.

B

The nurse is evaluating if a patient understands how to safely determine whether a metered-dose inhaler (MDI) is empty. The nurse interprets that the patient understands this important information to prevent medication underdosing when the patient describes which method to check the inhaler? Place it in water to see if it floats. Keep track of the number of inhalations used. Shake the canister while holding it next to the ear. Check the indicator line on the side of the canister.

The nurse is caring for a patient with superficial partial-thickness burns of the face sustained within the last 12 hours. Upon assessment, the nurse would expect to find which of the following symptoms? A. Blisters B. Reddening of the skin C. Destruction of all skin layers D. Damage to sebaceous glands

b. The clinical appearance of superficial partial-thickness burns includes erythema, blanching with pressure, and pain and minimal swelling with no vesicles or blistering during the first 24 hours.

The health care provider is considering the use of *sumatriptan (Imitrex)* for a 54-yr-old male patient with *migraine headaches*. Which information obtained by the nurse is most important to report to the health care provider? a. The patient drinks 1 to 2 cups of coffee daily. b. The patient had a recent acute myocardial infarction. c. The patient has had migraine headaches for 30 years. d. The patient has taken topiramate (Topamax) for 2 months.

b. The patient had a recent acute myocardial infarction. The *triptans* cause coronary *artery vasoconstriction* and should be *avoided* in patients *with coronary artery disease*.

When a *74-yr*-old patient is seen in the health clinic with new development of a *stooped posture, shuffling gait, and pill rolling-type tremor*, the nurse will anticipate teaching the patient about a. oral corticosteroids. b. antiparkinsonian drugs. c. magnetic resonance imaging (MRI). d. electroencephalogram (EEG) testing.

b. antiparkinsonian drugs. The clinical diagnosis of *Parkinson's* is made when *tremor, rigidity, and akinesia, and postural instability* are present.

To maintain a positive nitrogen balance in a major burn, the patient must a. increase normal caloric intake by about 3 times b. eat a high-protein, low-fat, high-carbohydrate diet. c. eat at least 1500 calories per day in small, frequent meals. d. eat rice and whole wheat for the chemical effect on nitrogen balance

b. eat a high-protein, low-fat, high-carbohydrate diet. The patient should be encouraged to eat high-protein, high-carbohydrate foods to meet increased caloric needs. Massive catabolism can occur and is characterized by protein breakdown and increased gluconeogenesis. Failure to supply adequate calories and protein leads to malnutrition and delayed healing.

4. The effects of cigarette smoking on the respiratory system include a. hypertrophy of capillaries causing hemoptysis. b. hyperplasia of goblet cells and increased production of mucus. c. increased proliferation of cilia and decreased clearance of mucus. d. proliferation of alveolar macrophages to decrease the risk for infection.

b. hyperplasia of goblet cells and increased production of mucus.

When obtaining a health history and physical assessment for a 36-yr-old female patient with possible *multiple sclerosis (MS)*, the nurse should a. assess for the presence of chest pain. b. inquire about urinary tract problems. c. inspect the skin for rashes or discoloration. d. ask the patient about any increase in libido.

b. inquire about urinary tract problems. *Urinary tract problems* with incontinence or retention are common symptoms of *MS*.

In contrast to diverticulitis, the patient with diverticulosis: a. has rectal bleeding b. often has no symptoms c. has localized cramping pain d. frequently develops peritonitis

b. often has no symptoms Rationale: Many people with diverticulosis have no symptoms. Patients with diverticulitis have symptoms of inflammation. Diverticulitis can lead to obstruction or perforation.

The nurse is administering fluids intravenously as prescribed to a client who sustained superficial partial thickness burn injuries of the back and legs. In evaluating the adequacy of fluid resuscitation, the nurse understands that which of the following would provide the most reliable indicator for determining the adequacy? a. vital signs b. urine output c. mental status d. peripheral pulses

b. successful or adequate fluid resuscitation in the client is signaled by stable vital signs, adequate urine output, palpable peripheral pulses and clear sensorium.

Pain management for the burn patient is most effective when a. opoids are administered on a set schedule around the clock b. the patient has as much control over the management of the pain as possible. c. there is flexibility to administer opioids withing a dosage and frequency range d. painful dressing changes are delayed until the pt's pain is totally relieved.

b. the patient has as much control over the management of the pain as possible. The more control the patient has in managing the pain, the more successful the chosen strategies. Active patient participation has been found to be effective for some patients in anticipating and coping with treatment-induced pain.

A patient has been diagnosed with osteosarcoma of the humerus. He shows an understanding of his treatment options when he states a."I accept that I have to lose my arm with surgery." b."The chemotherapy before surgery will shrink the tumor." c."This tumor is related to the melanoma I had 3 years ago." d."I'm glad they can take out the cancer with such a small scar."

b."The chemotherapy before surgery will shrink the tumor." A patient with osteosarcoma usually has preoperative chemotherapy to decrease tumor size before surgery. As a result, limb-salvage procedures, including a wide surgical resection of the tumor, are being used more often. Osteosarcoma is a primary bone tumor that is extremely aggressive and rapidly metastasizes to distant sites.

In caring for a patient after a spinal fusion, the nurse would immediately report to the physician which patient symptom? a.The patient experiences a single episode of emesis. b.The patient is unable to move the lower extremities. c. The patient is nauseated and has not voided in 4 hours. d. The patient complains of pain at the bone graft donor site.

b.The patient is unable to move the lower extremities. After spinal fusion surgery, the nurse should frequently monitor peripheral neurologic signs. Movement of the arms and legs and assessment of sensation should be unchanged in comparison with the preoperative status. These assessments are usually repeated every 2 to 4 hours during the first 48 hours after surgery, and findings are compared with those of the preoperative assessment. Paresthesias, such as numbness and tingling sensation, may not be relieved immediately after surgery. The nurse should document any new muscle weakness or paresthesias and report this to the surgeon immediately.

Which clinical manifestations of inflammatory bowel disease are common to both patients with ulcerative colitis (UC) and Crohn's disease (select all that apply)? a. Restricted to rectum b. Strictures are common. c. Bloody, diarrhea stools d. Cramping abdominal pain e. Lesions penetrate intestine.

c & d Clinical manifestations of UC and Crohn's disease include bloody diarrhea, cramping abdominal pain, and nutritional disorders. Intestinal lesions associated with UC are usually restricted to the rectum before moving into the colon. Lesions that penetrate the intestine or cause strictures are characteristic of Crohn's disease.

When establishing a diagnosis of multiple sclerosis (MS), which diagnostic tests will the nurse expect (select all that apply.)? Select all that apply. a. EEG b. ECG c. CT scan d. Carotid duplex scan e. Evoked response testing f. Cerebrospinal fluid analysis

c, e, f c. CT scan e. Evoked response testing f. Cerebrospinal fluid analysis

10. The nurse is admitting the client with OA to the medical floor. Which statement by the client indicates a complementary form of treatment for OA? a. "I take medication every two hours for my pain" b. "I use a heating pad when I go to bed at night" c. "I wear a copper bracelet to help with my OA" d. "I always wear my ankle splints when I sleep"

c. "I wear a copper bracelet to help with my OA"

The nurse determines that teaching about management of migraine headaches has been effective when the patient says which of the following? a. "I can take the (Topamax) as soon as a headache starts." b. "A glass of wine might help me relax and prevent a headache." c. "I will lie down someplace dark and quiet when the headaches begin." d. "I should avoid taking aspirin and sumatriptan (Imitrex) at the same time."

c. "I will lie down someplace dark and quiet when the headaches begin." It is recommended that the patient with a migraine rest in a dark, quiet area. Topiramate (Topamax) is used to prevent migraines and must be taken for several months to determine effectiveness. Aspirin or other nonsteroidal antiinflammatory medications can be taken with the triptans. Alcohol may precipitate migraine headaches.

A client with burn injury asks the nurse what the term full thickness means. The nurse should respond that burns classified as full thickness involve tissue destruction down to which level? a. epidermis b. dermis c. subcutaneous tissue d. internal organs

c. A full thickness burn involves all skin layers, including the epidermis and dermis, and may extend into the subcutaneous tissue and fat.

Which *prescribed intervention* will the nurse implement first for a patient in the emergency department who is experiencing continuous *tonic-clonic seizures*? a. Give phenytoin (Dilantin) 100 mg IV. b. Monitor level of consciousness (LOC). c. Administer lorazepam (Ativan) 4 mg IV. d. Obtain computed tomography (CT) scan.

c. Administer lorazepam (Ativan) 4 mg IV. To *prevent ongoing seizures*, the nurse should administer rapidly acting *antiseizure medications* such as the *benzodiazepines*.

A 22-yr-old patient seen at the health clinic with a severe migraine headache tells the nurse about having similar headaches recently. Which initial action should the nurse take? a. Teach about the use of triptan drugs. b. Refer the patient for stress counseling. c. Ask the patient to keep a headache diary. d. Suggest the use of muscle-relaxation techniques.

c. Ask the patient to keep a headache diary The *initial nursing action* should be further *assessment* of the precipitating causes of the headaches, quality, and location of pain.

A nurse is caring for a patient with second- and third-degree burns to 50% of the body. The nurse prepares fluid resuscitation based on knowledge of the Parkland (Baxter) formula that includes which of the following recommendations? A. The total 24-hour fluid requirement should be administered in the first 8 hours. B. One half of the total 24-hour fluid requirement should be administered in the first 8 hours. C. One third of the total 24-hour fluid requirement should be administered in the first 4 hours. D. One half of the total 24-hour fluid requirement should be administered in the first 4 hours.

c. Fluid resuscitation with the Parkland (Baxter) formula recommends that one half of the total fluid requirement should be administered in the first 8 hours, one quarter of total fluid requirement should be administered in the second 8 hours, and one quarter of total fluid requirement should be administered in the third 8 hours.

A 33-yr-old patient with *multiple sclerosis (MS)* is to begin treatment with *glatiramer acetate (Copaxone)*. Which information will the nurse include in patient teaching? a. Recommendation to drink at least 4 L of fluid daily b. Need to avoid driving or operating heavy machinery c. How to draw up and administer injections of the medication d. Use of contraceptive methods other than oral contraceptives

c. How to draw up and administer injections of the medication *Copaxone* is administered by *self-injection*

Which information about a patient who has a new prescription for *phenytoin (Dilantin)* indicates that the nurse should consult with the health care provider *before administration* of the medication? a. Patient has tonic-clonic seizures. b. Patient experiences an aura before seizures. c. Patient has minor elevations in the liver function tests. d. Patient's most recent blood pressure is 156/92 mm Hg.

c. Patient has minor elevations in the liver function tests. Many *older patients* (especially with *compromised liver function*) may *not be able to metabolize phenytoin*.

3. Which member of the health care team should the nurse refer the client diagnosed with OA who is complaining of not being able to get in and out of the bathtub? a. Physiatrist b. Social worker c. Physical therapist d. Counselor

c. Physical therapist

11. The client diagnosed with RA is receiving care through a nurse practitioner clinic. Which preventive care should the nurse include in the regularly scheduled clinic visits? a. Perform joint x-rays to determine progression of the disease b. Send blood to the lab for an erythrocyte sedimentation rate (ESR) c. Recommend the flu and pneumonia vaccines d. Assess the client for increasing joint involvement

c. Recommend the flu and pneumonia vaccines

The appropriate collaborative therapy for the patient with acute diarrhea caused by a viral infection is to: a. increase fluid intake b. administer an antibiotic c. administer antimotality drugs d. quarantine the patient to prevent spread of the virus

a. increase fluid intake Rationale: Acute diarrhea resulting from infectious causes (e.g., virus) is usually self-limiting. The major concerns are transmission prevention, fluid and electrolyte replacement, and resolution of the diarrhea. Antidiarrheal agents are contraindicated in the treatment of infectious diarrhea because they potentially prolong exposure to the infectious organism. Antibiotics are rarely used to treat acute diarrhea. To prevent transmission of diarrhea caused by a virus, hands should be washed before and after contact with the patient and when body fluids of any kind are handled. Vomitus and stool should be flushed down the toilet, and contaminated clothing should be washed immediately with soap and hot water.

The nurse advises a patient with *myasthenia gravis (MG)* to a. perform physically demanding activities early in the day. b. anticipate the need for weekly plasmapheresis treatments. c. do frequent weight-bearing exercise to prevent muscle atrophy. d. protect the extremities from injury due to poor sensory perception.

a. perform physically demanding activities early in the day. *Muscles* are generally *strongest in the morning*, and activities involving muscle activity should be scheduled then.

Assessment findings suggestive of peritonitis include a. rebound abdominal pain b. a soft, distended abdomen c. dull, continuous abdominal pain d. observing that the patient is restless

a. rebound abdominal pain Rationale: With peritoneal irritation, the abdomen is hard, like a board, and the patient has severe abdominal pain that is worse with any sudden movement. The patient lies very still. Palpating the abdomen and releasing the hands suddenly causes sudden movement within the abdomen and severe pain. This is called rebound tenderness.

The nurse determines that the goals of dietary teaching have been met when the patient with celiac disease selects from the menu: a. scrambled eggs and sausage b. buckwheat pancakes with syrup c. oatmeal, skim milk, and orange juice d. yogurt, strawberries, and rye toast with butter

a. scrambled eggs and sausage Rationale: Celiac disease is treated with lifelong avoidance of dietary gluten. Wheat, barley, oats, and rye products must be avoided. Although pure oats do not contain gluten, oat products can become contaminated with wheat, rye, and barley during the milling process. Gluten is also found in some medications and in many food additives, preservatives, and stabilizers.

A pt in the emergent phase of burn care for thermal burns on 20% of the total body surface area is unconscious. Which assessment data is the most important for the nurse's evaluation of the pt's injuries? a. condition of the oropharynx b. percentage of TBSA affected c. location of the pt in the fire d. comorbidities of the pt

a. the pt is likely to have suffered a smoke inhalation injury because thermal burns are caused by flames that emit smoke and because the pt is unconscious.

A patient with osteomyelitis is treated with surgical debridement with implantation of antibiotic beads. When the patient asks why the beads are used, the nurse answers (select all that apply) a."The beads are used to directly deliver antibiotics to the site of the infection." b."There are no effective oral or IV antibiotics to treat most cases of bone infection." c."This is the safest method of delivering long-term antibiotic therapy for a bone infection." d."The beads are an adjunct to debridement and oral and IV antibiotics for deep infections." e."The ischemia and bone death that occur with osteomyelitis are impenetrable to IV antibiotics."

a."The beads are used to directly deliver antibiotics to the site of the infection." d."The beads are an adjunct to debridement and oral and IV antibiotics for deep infections." Treatment of chronic osteomyelitis includes surgical removal of the poorly vascularized tissue and dead bone and the extended use of IV and oral antibiotics. Antibiotic-impregnated polymethylmethacrylate bead chains may be implanted during surgery to aid in combating the infection.

Before discharge from the same-day surgery unit, instruct the patient who has had a surgical correction of bilateral hallux valgus to a.rest frequently with the feet elevated. b.soak the feet in warm water several times a day. c.expect the feet to be numb for the next few days. d.expect continued pain in the feet, since this is not uncommon.

a.rest frequently with the feet elevated. After surgical correction of bilateral hallux valgus, the feet should be elevated with the heel off the bed to help reduce discomfort and prevent edema.

2. While obtaining subjective assessment data from a patient with hypertension, the nurse recognizes that a modifiable risk factor for the development of hypertension is a. a low-calcium diet. b. excessive alcohol intake. c. a family history of hypertension. d. consumption of a high-protein diet.

b

A high school teacher who has been diagnosed with epilepsy after having a generalized tonic-clonic seizure tells the nurse, "I cannot teach any more. It will be too upsetting if I have a seizure at work." Which response by the nurse specifically addresses the patient's concern? a. "You might benefit from some psychologic counseling." b. "Epilepsy usually can be well controlled with medications." c. "You will want to contact the Epilepsy Foundation for assistance." d. "The Department of Vocational Rehabilitation can help with work retraining."

b. "Epilepsy usually can be well controlled with medications." The nurse should inform the patient that most patients with seizure disorders are controlled with medication.

The nurse is caring for a client who suffered an inhalation injury from a wood stove. The carbon monoxide blood report reveals a level of 12%. Based on this level, the nurse would anticipate which of the following signs in the client? a. coma b. flushing c. dizziness d. tachycardia

b. 11-20% - signs include flushing, headache, decreased visual acuity, decreased cerebral functioning, and slight breathlessness. 21-40% - signs include nausea, vomiting, dizziness, tinnitus, vertigo, confusion, drowsiness, pale to reddish-purple skin, tachycardia; levels of 41-60% result in seizure and come and levels higher than 60% result in death

A patient who is given a bisacodyl (Dulcolax) suppository asks the nurse how long it will take to work. The nurse replies that the patient will probably need to use the bedpan or commode within which time frame after administration? a. 2-5 minutes b. 15-60 minutes c. 2-4 hours d. 6-8 hours

b. 15-60 minutes Bisacodyl suppositories usually are effective within 15 to 60 minutes of administration, so the nurse should plan accordingly to assist the patient to use the bedpan or commode.

A hospitalized patient complains of a bilateral headache (4/10 on the pain scale) that radiates from the base of the skull. Which prescribed PRN medications should the nurse administer *initially*? a. Lorazepam (Ativan) c. Morphine sulfate (MS Contin) b. Acetaminophen (Tylenol) d. Butalbital and aspirin (Fiorinal)

b. Acetaminophen (Tylenol) The patient's symptoms are consistent with a *tension headache*, and initial therapy usually involves a *nonopioid analgesic* such as *acetaminophen*, which is sometimes combined with a sedative or muscle relaxant.

A colectomy is scheduled for a 38-year-old woman with ulcerative colitis. The nurse should plan to include what prescribed measure in the preoperative preparation of this patient? a. Instruction on irrigating a colostomy b. Administration of a cleansing enema c. A high-fiber diet the day before surgery d. Administration of IV antibiotics for bowel preparation

b. Administration of a cleansing enema Preoperative preparation for bowel surgery typically includes bowel cleansing with antibiotics, such as oral neomycin and cleansing enemas, including Fleet enemas. Instructions to irrigate the colostomy will be done postoperatively. Oral antibiotics are given preoperatively, and an IV antibiotic may be used in the OR. A clear liquid diet will be used the day before surgery with the bowel cleansing.

A patient with chronic osteomyelitis has been hospitalized for a surgical debridement procedure. What does the nurse explain to the patient as the rationale for the surgical treatment? a. Removal of the infection prevents the need for bone and skin grafting. b. Formation of scar tissue has led to a protected area of bacterial growth. c. The process of depositing new bone blocks the vascular supply to the bone. d. Antibiotics are not effective against microorganisms that cause chronic osteomyelitis.

b. Chronic infection of the bone leads to formation of scar tissue from the granulation tissue. This avascular scar tissue provides an ideal site for continued microorganism growth and is impenetrable to antibiotics. Surgical debridement is often necessary to remove the poorly vascularized tissue and dead bone and to instill antibiotics directly to the area. Involucrum is new bone laid down at the infection site, which seals off areas of sequestra. Antibiotics can be effective during acute osteomyelitis and prevention of chronic osteomyelitis requires early antibiotic treatment. Bone and skin grafting may be necessary following surgical removal of infection if destruction is extensive.

The nurse is planning care for a 68-year-old patient with an abdominal mass and suspected bowel obstruction. Which factor in the patient's history increases the patient's risk for colorectal cancer? a. Osteoarthritis b. History of colorectal polyps c. History of lactose intolerance d. Use of herbs as dietary supplements

b. History of colorectal polyps A history of colorectal polyps places this patient at risk for colorectal cancer. This tissue can degenerate over time and become malignant. Osteoarthritis, lactose intolerance, and the use of herbs do not pose additional risk to the patient.

What information would have the highest priority to be included in preoperative teaching for a 68-year-old patient scheduled for a colectomy? a. How to care for the wound b. How to deep breathe and cough c. The location and care of drains after surgery d. Which medications will be used during surgery

b. How to deep breathe and cough Because anesthesia, an abdominal incision, and pain can impair the patient's respiratory status in the postoperative period, it is of high priority to teach the patient to cough and deep breathe. Otherwise, the patient could develop atelectasis and pneumonia, which would delay early recovery from surgery and hospital discharge. Care for the wound and location and care of the drains will be briefly discussed preoperatively, but done again with higher priority after surgery. Knowing which drugs will be used during surgery may not be meaningful to the patient and should be reviewed with the patient by the anesthesiologist.

9. The HCP prescribes glucosamine and chondroitin for a client diagnosed with OA. What is the scientific rationale for prescribing this medication? a. It will help decrease the inflammation in the joints b. It improves tissue function and may decrease breakdown of cartilage c. It is a potent medication that decreases the client's joint pain d. It increases the production of synovial fluid in the joint

b. It improves tissue function and may decrease breakdown of cartilage

5. The client is diagnosed with osteoarthritis. Which sign/symptom would the nurse expect the client to exhibit? a. Severe bone deformity b. Joint stiffness c. Waddling gait d. Swan neck fingers

b. Joint stiffness

The nurse should administer an as-needed dose of magnesium hydroxide (MOM) after noting what information while reviewing a patient's medical record? a. Abdominal pain and bloating b. No bowel movement for 3 days c. A decrease in appetite by 50% over 24 hours d. Muscle tremors and other signs of hypomagnesemia

b. No bowel movement for 3 days MOM is an osmotic laxative that produces a soft, semisolid stool usually within 15 minutes to 3 hours. This medication would benefit the patient who has not had a bowel movement for 3 days. MOM would not be given for abdominal pain and bloating, decreased appetite, or signs of hypomagnesemia.

After a *thymectomy*, a patient with *myasthenia gravis* receives the usual dose of *pyridostigmine (Mestinon)*. An hour later, the patient complains of *nausea and severe abdominal cramps*. Which action should the nurse take first? a. Auscultate the patient's bowel sounds. b. Notify the patient's health care provider. c. Administer the prescribed PRN antiemetic drug. d. Give the scheduled dose of prednisone (Deltasone).

b. Notify the patient's health care provider. The patient's history and symptoms indicate a possible *cholinergic crisis*. The health care provider should be notified immediately, and it is likely that *atropine will be prescribed*.

13. The client diagnosed with RA has developed swan-neck fingers. Which referral would be the most appropriate for the client? a. Physical therapy b. Occupational therapy c. Psychiatric counselor d. Home health nurse

b. Occupational therapy

Which intervention will the nurse include in the plan of care for a patient with primary restless legs syndrome (RLS) who is having difficulty sleeping? a. Teach about the use of antihistamines to improve sleep. b. Suggest that the patient exercise regularly during the day. c. Make a referral to a massage therapist for deep massage of the legs. d. Assure the patient that the problem is transient and likely to resolve.

b. Suggest that the patient exercise regularly during the day. *Nondrug interventions* such as getting regular exercise are initially suggested to improve sleep quality in patients with *RLS*.

A patient with *Parkinson's disease* has *bradykinesia*. Which action will the nurse include in the plan of care? a. Instruct the patient in activities that can be done while lying or sitting. b. Suggest that the patient rock from side to side to initiate leg movement. c. Have the patient take small steps in a straight line directly in front of the feet. d. Teach the patient to keep the feet in contact with the floor and slide them forward.

b. Suggest that the patient rock from side to side to initiate leg movement. *Rocking the body* from side to side *stimulates balance* and improves mobility.

Which action will the nurse plan to take for a patient with multiple sclerosis who has *urinary retention* caused by a *flaccid bladder*? a. Encourage a decreased evening intake of fluid. b. Teach the patient how to use the Credé method. c. Suggest the use of adult incontinence briefs for nighttime only. d. Assist the patient to the commode every 2 hours during the day.

b. Teach the patient how to use the Credé method. The *Credé method* can be used to improve *bladder emptying*

In which order will the nurse perform the following actions when caring for a patient with possible C5 spinal cord trauma who is admitted to the emergency department? (Put a comma and a space between each answer choice [A, B, C, D, E].) a. Infuse normal saline at 150 mL/hr. b. Monitor cardiac rhythm and blood pressure. c. Administer O2 using a nonrebreather mask. d. Immobilize the patient's head, neck, and spine. e. Transfer the patient to radiology for spinal computed tomography (CT).

ANS: D, C, B, A, E The first action should be to prevent further injury by stabilizing the patient's spinal cord if the patient does not have penetrating trauma. Maintenance of oxygenation by administration of 100% O2 is the second priority. Because neurogenic shock is a possible complication, monitoring of heart rhythm and BP are indicated followed by infusing normal saline for volume replacement. A CT scan to determine the extent and level of injury is needed once initial assessment and stabilization are accomplished. DIF: Cognitive Level: Analyze (analysis)

Which action will the nurse include in the plan of care for a 40-year-old with newly diagnosed ankylosing spondylitis? a. Advise the patient to sleep on the back with a flat pillow. b. Emphasize that application of heat may worsen symptoms. c. Schedule annual laboratory assessment for the HLA-B27 antigen. d. Assist patient to choose physical activities that allow the spine to flex.

ANS: A Because ankylosing spondylitis results in flexion deformity of the spine, postures that extend the spine (such as sleeping on the back and with a flat pillow) are recommended. HLA-B27 antigen levels are used for initial diagnosis, but are not needed annually. To counteract the development of flexion deformities, the patient should choose activities that extend the spine, such as swimming. Heat application is used to decrease localized pain

A 28-year-old with psoriatic arthritis and back pain is receiving etanercept (Enbrel). Which finding is most important for the nurse to report to the health care provider? a. Crackles are heard in both lung bases. b. Red, scaly patches are noted on the arms. c. Hemoglobin level is 11.1g/dL and hematocrit is 35%. d. Patient reports continued back pain after a week of etanercept therapy.

ANS: A Because heart failure is a possible adverse effect of etanercept, the medication may need to be discontinued. The other information will also be reported to the health care provider but does not indicate a need for a change in treatment. Red, scaly patches of skin and mild anemia are commonly seen with psoriatic arthritis. Treatment with biologic therapies requires time to improve symptoms.

Following bowel resection, a patient has a nasogastric (NG) tube to suction, but complains of nausea and abdominal distention. The nurse irrigates the tube as necessary as ordered, but the irrigating fluid does not return. What should be the priority action by the nurse? a. Notify the physician. b. Auscultate for bowel sounds. c. Reposition the tube and check for placement. d. Remove the tube and replace it with a new one.

c. Reposition the tube and check for placement. The tube may be resting against the stomach wall. The first action by the nurse (since this is intestinal surgery and not gastric surgery) is to reposition the tube and check it again for placement. The physician does not need to be notified unless the tube function cannot be restored by the nurse. The patient does not have bowel sounds, which is why the NG tube is in place. The NG tube would not be removed and replaced unless it was no longer in the stomach or the obstruction of the tube could not be relieved.

Which assessment is *most* important for the nurse to make regarding a patient with myasthenia gravis? a. Pupil size c. Respiratory effort b. Grip strength d. Level of consciousness

c. Respiratory effort Because respiratory insufficiency may be life threatening, it will be most important to monitor respiratory function.

15. The nurse is caring for clients on a medical floor. Which client should the nurse assess first? a. The client diagnosed with RA who is complaining of pain at a "3" on a 1-10 scale b. The client diagnosed with Systemic Lupus Erythematosus who has a rash across the bridge of the nose c. The client diagnosed with advanced RA who is receiving antineoplastic drugs IV d. The client diagnosed with scleroderma who has hard, waxylike skin near the eyes

c. The client diagnosed with advanced RA who is receiving antineoplastic drugs IV

Which information about a 60-yr-old patient with *multiple sclerosis* indicates that the nurse should consult with the health care provider before giving the prescribed dose of *dalfampridine (Ampyra)*? a. The patient walks a mile each day for exercise. b. The patient complains of pain with neck flexion. c. The patient has an increased serum creatinine level. d. The patient has the relapsing-remitting form of MS.

c. The patient has an increased serum creatinine level. *Dalfampridine* should *not* be given to patients with *impaired renal function*. The other information will not impact whether the dalfampridine should be administered.

What is the best method for preventing hypovolemic shock in a client admitted with severe burns? a. administering dopamine b. applying medical antishock trousers c. infusing i.v. fluids d. infusing fresh frozen plasma

c. during the early postburn period, large amounts of plasma fluid extravasates into interstitial spaces. Restoring the fluid loss is necessary to prevent hypovolemic shock; this is best accomplished with crystalloid and colloid solutions.

A client is undergoing fluid replacement after being burned on 20% of her body 12 hours ago. The nursing assessment reveals a blood pressure of 90/50, a pulse of 110, and urine output of 20 mL over the past hour. The nurse reports the findings to the physician and anticipates which of the following prescriptions? a. transfusing 1 unit of packed red blood cells b. administering a diuretic to increase urine output c. increasing the amount of IV lactated Ringers solution administered per hour d. changing the IV lactated Ringer's solution to one that contains dextrose in water.

c. fluid management during the first 24 hours following a burn injury generally includes the infusion of LR solution. Fluid resuscitation is determined by urine output and hourly urine output should be at least 30mL/hr. The client's urine output is indicative of insufficient fluid resuscitation, which places the client at risk for inadequate perfusion of the brain, heart, kidneys, and other body organs. Therefore, should expect ↑ of LR's.

A patient with stage I colorectal cancer is scheduled for surgery. Patient teaching for this patient would include an explanation that: a. chemotherapy will begin after the patient recovers from the surgery b. both chemotherapy and radiation can be used as palliative treatments c. follow-up colonoscopies will be needed to ensure that the cancer does not occur d. a wound, ostomy, and continence nurse will visit the patient to identify an abdominal site for the ostomy

c. follow-up colonoscopies will be needed to ensure that the cancer does not occur Rationale: Stage 1 colorectal cancer is treated with surgical removal of the tumor and reanastomosis, and so there is no ostomy. Chemotherapy is not recommended for stage I tumors. Follow-up colonoscopy is recommended because colorectal cancer can recur.

In planning care for the patient with Crohn's disease, the nurse recognizes that a major difference between ulcerative colitis and Crohn's disease is that Crohn's disease: a. frequently results in toxic megacolon b. causes fever nutritional deficiencies than ulcerative colitis c. often recurs after surgery, whereas ulcerative colitis is curable with a colectomy d. is manifested by rectal bleeding and anemia more frequently than is ulcerative colitis

c. often recurs after surgery, whereas ulcerative colitis is curable with a colectomy Rationale: Ulcerative colitis affects only the colon and rectum; it can cause megacolon and rectal bleeding, but not nutrient malabsorption. Surgical removal of the colon and rectum cures it. Crohn's disease usually involves the ileum, where bile salts and vitamin cobalamin are absorbed. After surgical treatment, disease recurrence at the site is common.

A 40-yr-old patient is diagnosed with *early Huntington's disease (HD)*. When *teaching* the patient, spouse, and adult children about this disorder, the nurse will provide information about the a. use of levodopa-carbidopa (Sinemet) to help reduce HD symptoms. b. prophylactic antibiotics to decrease the risk for aspiration pneumonia. c. option of genetic testing for the patient's children to determine their own HD risks. d. lifestyle changes of improved nutrition and exercise that delay disease progression.

c. option of genetic testing for the patient's children to determine their own HD risks. *Genetic testing* is available to determine whether an asymptomatic individual has the *HD gene*.

The nurse determines that colchicine has been effective for a patient with an acute attack of gout upon finding a. relief of joint pain. b. increased urine output. c. elevated serum uric acid. d. increased white blood cells (WBC).

ANS: A Colchicine produces pain relief in 24 to 48 hours by decreasing inflammation. The recommended increase in fluid intake of 2 to 3 L/day would increase urine output but would not indicate the effectiveness of colchicine. Elevated uric acid levels would result in increased symptoms. The WBC count might decrease with decreased inflammation, but would not increase.

A female patient who had a stroke 24 hours ago has expressive aphasia. An appropriate nursing intervention to help the patient communicate is to a. ask questions that the patient can answer with "yes" or "no." b. develop a list of words that the patient can read and practice reciting. c. have the patient practice her facial and tongue exercises with a mirror. d. prevent embarrassing the patient by answering for her if she does not respond.

ANS: A Communication will be facilitated and less frustrating to the patient when questions that require a "yes" or "no" response are used. When the language areas of the brain are injured, the patient might not be able to read or recite words, which will frustrate the patient without improving communication. Expressive aphasia is caused by damage to the language areas of the brain, not by the areas that control the motor aspects of speech. The nurse should allow time for the patient to respond. DIF: Cognitive Level: Apply (application) REF: 1361 TOP: Nursing Process: Implementation MSC: NCLEX: Physiological Integrity

A patient with carotid atherosclerosis asks the nurse to describe a carotid endarterectomy. Which response by the nurse is accurate? a. "The obstructing plaque is surgically removed from inside an artery in the neck." b. "The diseased portion of the artery in the brain is replaced with a synthetic graft." c. "A wire is threaded through an artery in the leg to the clots in the carotid artery, and the clots are removed." d. "A catheter with a deflated balloon is positioned at the narrow area, and the balloon is inflated to flatten the plaque."

ANS: A In a carotid endarterectomy, the carotid artery is incised, and the plaque is removed. The response beginning, "The diseased portion of the artery in the brain is replaced" describes an arterial graft procedure. The answer beginning, "A catheter with a deflated balloon is positioned at the narrow area" describes an angioplasty. The final response beginning, "A wire is threaded through the artery" describes the mechanical embolus removal in cerebral ischemia (MERCI) procedure. DIF: Cognitive Level: Understand (comprehension) REF: 1353 TOP: Nursing Process: Implementation MSC: NCLEX: Physiological Integrity

Which finding will the nurse expect when assessing a 58-year-old patient who has osteoarthritis (OA) of the knee? a. Discomfort with joint movement b. Heberden's and Bouchard's nodes c. Redness and swelling of the knee joint d. Stiffness that increases with movement

ANS: A Initial symptoms of OA include pain with joint movement. Heberden's nodules occur on the fingers. Redness of the joint is more strongly associated with rheumatoid arthritis (RA). Stiffness in OA is worse right after the patient rests and decreases with joint movement.

6. A patient is diagnosed with hypertension and nadolol (Corgard) is prescribed. The nurse should consult with the health care provider before giving this medication upon finding a history of a. asthma. b. peptic ulcer disease. c. alcohol dependency. d. myocardial infarction (MI).

ANS: A Nonselective b-blockers block b1- and b2-adrenergic receptors and can cause bronchospasm, especially in patients with a history of asthma. b-blockers will have no effect on the patient's peptic ulcer disease or alcohol dependency. b-blocker therapy is recommended after MI.

An assessment finding for a 55-year-old patient that alerts the nurse to the presence of osteoporosis is a. a measurable loss of height. b. the presence of bowed legs. c. the aversion to dairy products. d. a statement about frequent falls.

ANS: A Osteoporosis occurring in the vertebrae produces a gradual loss of height. Bowed legs are associated with osteomalacia. Low intake of dairy products is a risk factor for osteoporosis, but it does not indicate that osteoporosis is present. Frequent falls increase the risk for fractures but are not an indicator of osteoporosis

Which action will the nurse include in the plan of care for a patient who has a cauda equina spinal cord injury? a. Catheterize patient every 3 to 4 hours. b. Assist patient to ambulate 4 times daily. c. Administer medications to reduce bladder spasm. d. Stabilize the neck when repositioning the patient.

ANS: A Patients with cauda equina syndrome have areflexic bladder, and intermittent catheterization will be used for emptying the bladder. Because the bladder is flaccid, antispasmodic medications will not be used. The legs are flaccid with cauda equina syndrome, and the patient will be unable to ambulate. The head and neck will not need to be stabilized after a cauda equina injury, which affects the lumbar and sacral nerve roots. DIF: Cognitive Level: Apply (application)

Which finding for a patient who is taking hydroxychloroquine (Plaquenil) to treat rheumatoid arthritis is likely to be an adverse effect of the medication? a. Blurred vision b. Joint tenderness c. Abdominal cramping d. Elevated blood pressure

ANS: A Plaquenil can cause retinopathy. The medication should be stopped. The other findings are not related to the medication although they will also be reported.

Which assessment finding about a patient who has been using naproxen (Naprosyn) for 6 weeks to treat osteoarthritis is most important for the nurse to report to the health care provider? a. The patient has gained 3 pounds. b. The patient has dark-colored stools. c. The patient's pain has become more severe. d. The patient is using capsaicin cream (Zostrix).

ANS: B Dark-colored stools may indicate that the patient is experiencing gastrointestinal bleeding caused by the naproxen. The information about the patient's ongoing pain and weight gain also will be reported and may indicate a need for a different treatment and/or counseling about avoiding weight gain, but these are not as large a concern as the possibility of gastrointestinal bleeding. Use of capsaicin cream with oral medications is appropriate.

Which information will the nurse include when teaching a 38-year-old male patient with newly diagnosed ankylosing spondylitis (AS) about the management of the condition? a. Exercise by taking long walks. b. Do daily deep-breathing exercises. c. Sleep on the side with hips flexed. d. Take frequent naps during the day.

ANS: B Deep-breathing exercises are used to decrease the risk for pulmonary complications that may occur with the reduced chest expansion that can occur with ankylosing spondylitis (AS). Patients should sleep on the back and avoid flexed positions. Prolonged standing and walking should be avoided. There is no need for frequent naps

Several weeks after a stroke, a 50-yr-old male patient has impaired awareness of bladder fullness, resulting in urinary incontinence. Which nursing intervention should be planned to begin an effective bladder training program? a. Limit fluid intake to 1200 mL daily to reduce urine volume. b. Assist the patient onto the bedside commode every 2 hours. c. Perform intermittent catheterization after each voiding to check for residual urine. d. Use an external "condom" catheter to protect the skin and prevent embarrassment.

ANS: B Developing a regular voiding schedule will prevent incontinence and may increase patient awareness of a full bladder. A 1200-mL fluid restriction may lead to dehydration. Intermittent catheterization and use of a condom catheter are appropriate in the acute phase of stroke, but should not be considered solutions for long-term management because of the risks for urinary tract infection and skin breakdown. DIF: Cognitive Level: Apply (application) REF: 1360 TOP: Nursing Process: Planning MSC: NCLEX: Physiological Integrity

After the nurse has taught a 28-year-old with fibromyalgia, which statement by the patient indicates a good understanding of effective self-management? a. "I am going to join a soccer team to get more exercise." b. "I will need to stop drinking so much coffee and soda." c. "I will call the doctor every time my symptoms get worse." d. "I should avoid using over-the-counter medications for pain."

ANS: B Dietitians frequently suggest that patients with fibromyalgia limit their intake of caffeine and sugar because these substances are muscle irritants. Mild exercise such as walking is recommended for patients with fibromyalgia, but vigorous exercise is likely to make symptoms worse. Because symptoms may fluctuate from day to day, the patient should be able to adapt the regimen independently, rather than calling the provider whenever symptoms get worse. Over-the-counter medications such as ibuprofen and acetaminophen are frequently used for symptom management

Which assessment data for a patient who has Guillain-Barré syndrome will require the nurse's most immediate action? a. The patient's sacral area skin is reddened. b. The patient is continuously drooling saliva. c. The patient complains of severe pain in the feet. d. The patient's blood pressure (BP) is 150/82 mm Hg.

ANS: B Drooling indicates decreased ability to swallow, which places the patient at risk for aspiration and requires rapid nursing and collaborative actions such as suctioning and possible endotracheal intubation. The foot pain should be treated with appropriate analgesics, the BP requires ongoing monitoring, and the skin integrity requires intervention, but these actions are not as urgently needed as maintenance of respiratory function. DIF: Cognitive Level: Analyze (analysis)

Which nursing action has the highest priority for a patient who was admitted 16 hours earlier with a C5 spinal cord injury? a. Cardiac monitoring for bradycardia b. Assessment of respiratory rate and effort c. Administration of low-molecular-weight heparin d. Application of pneumatic compression devices to legs

ANS: B Edema around the area of injury may lead to damage above the C4 level, so the highest priority is assessment of the patient's respiratory function. The other actions also are appropriate for preventing deterioration or complications but are not as important as assessment of respiratory effort. DIF: Cognitive Level: Analyze (analysis)

15. The nurse is reviewing the laboratory tests for a patient who has recently been diagnosed with hypertension. Which result is most important to communicate to the health care provider? a. Serum creatinine of 2.6 mg/dL b. Serum potassium of 3.8 mEq/L c. Serum hemoglobin of 14.7 g/dL d. Blood glucose level of 98 mg/dL

ANS: A The elevated creatinine indicates renal damage caused by the hypertension. The other laboratory results are normal.

A 37-year-old patient with 2 school-age children who has recently been diagnosed with rheumatoid arthritis (RA) tells the nurse that home life is very stressful. Which response by the nurse is most appropriate? a. "Tell me more about situations that are causing you stress." b. "You need to see a family therapist for some help with stress." c. "Your family should understand the impact of your rheumatoid arthritis." d. "Perhaps it would be helpful for your family to be involved in a support group."

ANS: A The initial action by the nurse should be further assessment. The other three responses might be appropriate based on the information the nurse obtains with further assessment

22. Which nursing action should the nurse take first in order to assist a patient with newly diagnosed stage 1 hypertension in making needed dietary changes? a. Have the patient record dietary intake for 3 days. b. Give the patient a detailed list of low-sodium foods. c. Teach the patient about foods that are high in sodium. d. Help the patient make an appointment with a dietitian.

ANS: A The initial nursing action should be assessment of the patient's baseline dietary intake through a 3-day food diary. The other actions may be appropriate, but assessment of the patient's baseline should occur first.

A patient who had arthroscopic surgery of the right knee 7 days ago is admitted with a red, swollen, and hot knee. Which assessment finding by the nurse should be reported to the health care provider immediately? a. The blood pressure is 86/50 mm Hg. b. The white blood cell count is 11,500/µL. c. The patient is taking ibuprofen (Motrin). d. The patient says the knee pain is severe.

ANS: A The low blood pressure suggests that the patient may be developing septicemia as a complication of septic arthritis. Immediate blood cultures and initiation of antibiotic therapy are indicated. The other information is typical of septic arthritis and should also be reported to the health care provider, but it does not indicate any immediately life-threatening problems.

A patient is being evaluated for a possible spinal cord tumor. Which finding by the nurse requires the most immediate action? a. The patient has new-onset weakness of both legs. b. The patient complains of chronic severe back pain. c. The patient starts to cry and says, "I feel hopeless." d. The patient expresses anxiety about having surgery.

ANS: A The new symptoms indicate spinal cord compression, an emergency that requires rapid treatment to avoid permanent loss of function. The other patient assessments also need nursing action but do not require intervention as rapidly as the new-onset weakness. DIF: Cognitive Level: Analyze (analysis)

A patient is admitted with possible botulism poisoning after eating home-canned green beans. Which intervention ordered by the health care provider will the nurse question? a. Encourage oral fluids to 3 L/day. b. Document neurologic symptoms. c. Position patient lying on the side. d. Observe respiratory status closely.

ANS: A The patient should be maintained on NPO status because neuromuscular weakness increases risk for aspiration. Side-lying position is not contraindicated. Assessment of neurologic and respiratory status is appropriate. DIF: Cognitive Level: Apply (application)

A patient has a ruptured cerebral aneurysm and subarachnoid hemorrhage. Which intervention will the nurse include in the plan of care? a. Apply intermittent pneumatic compression stockings. b. Assist to dangle on edge of bed and assess for dizziness. c. Encourage patient to cough and deep breathe every 4 hours. d. Insert an oropharyngeal airway to prevent airway obstruction.

ANS: A The patient with a subarachnoid hemorrhage usually has minimal activity to prevent cerebral vasospasm or further bleeding and is at risk for venous thromboembolism. Activities such as coughing and sitting up that might increase intracranial pressure or decrease cerebral blood flow are avoided. Because there is no indication that the patient is unconscious, an oropharyngeal airway is inappropriate. DIF: Cognitive Level: Apply (application) REF: 1359 TOP: Nursing Process: Planning MSC: NCLEX: Physiological Integrity

Which action will the nurse include in the plan of care for a patient who is experiencing pain from trigeminal neuralgia? a. Assess fluid and dietary intake. b. Apply ice packs for 20 minutes. c. Teach facial relaxation techniques. d. Spend time talking with the patient.

ANS: A The patient with an acute episode of trigeminal neuralgia may be unwilling to eat or drink, so assessment of nutritional and hydration status is important. Because stimulation by touch is the precipitating factor for pain, relaxation of the facial muscles will not improve symptoms. Application of ice is likely to precipitate pain. The patient will not want to engage in conversation, which may precipitate attacks. DIF: Cognitive Level: Apply (application)

After receiving change-of-shift report on the following four patients, which patient should the nurse see first? a. A 60-yr-old patient with right-sided weakness who has an infusion of tPA prescribed b. A 50-yr-old patient who has atrial fibrillation and a new order for warfarin (Coumadin) c. A 30-yr-old patient with a subarachnoid hemorrhage 2 days ago who has nimodipine scheduled d. A 40-yr-old patient who experienced a transient ischemic attack yesterday who has a dose of aspirin due

ANS: A tPA needs to be infused within the first few hours after stroke symptoms start in order to be effective in minimizing brain injury. The other medications should also be given as quickly as possible, but timing of the medications is not as critical. DIF: Cognitive Level: Analyze (analysis) REF: 1355 OBJ: Special Questions: Prioritization | Special Questions: Multiple Patients TOP: Nursing Process: Implementation MSC: NCLEX: Safe and Effective Care Environment

A patient whose work involves lifting has a history of chronic back pain. After the nurse has taught the patient about correct body mechanics, which patient statement indicates that the teaching has been effective? a. "I plan to start doing exercises to strengthen the muscles of my back." b. "I will try to sleep with my hips and knees extended to prevent back strain." c. "I can tell my boss that I need to change to a job where I can work at a desk." d. "I will keep my back straight when I need to lift anything higher than my waist."

ANS: A Exercises can help to strengthen the muscles that support the back. Flexion of the hips and knees places less strain on the back. Modifications in the way the patient lifts boxes are needed, but sitting for prolonged periods can aggravate back pain. The patient should not lift above the level of the elbows.

A patient is hospitalized for initiation of regional antibiotic irrigation for acute osteomyelitis of the right femur. Which intervention will be included in the plan of care? a. Immobilization of the right leg b. Frequent weight-bearing exercise c. Avoiding administration of nonsteroidal anti-inflammatory drugs (NSAIDs) d. Support of the right leg in a flexed position

ANS: A Immobilization of the affected leg helps decrease pain and reduce the risk for pathologic fractures. Weight-bearing exercise increases the risk for pathologic fractures. NSAIDs are frequently prescribed to treat pain. Flexion of the affected limb is avoided to prevent contractures.

Which statement by a patient who is scheduled for an above-the-knee amputation for treatment of an osteosarcoma of the right tibia indicates that patient teaching is needed? a. "I did not have this bone cancer until my leg broke a week ago." b. "I wish that I did not have to have chemotherapy after this surgery." c. "I know that I will need to participate in physical therapy after surgery." d. "I will use the patient-controlled analgesia (PCA) to control postoperative pain."

ANS: A Osteogenic sarcoma may be diagnosed following a fracture, but it is not caused by the injury. The other patient statements indicate that patient teaching has been effective.

An assessment finding that alerts the nurse to the presence of osteoporosis in a middle-aged patient is a. measurable loss of height. b. the presence of bowed legs. c. an aversion to dairy products. d. statements about frequent falls.

ANS: A Osteoporosis occurring in the vertebrae produces a gradual loss of height. Bowed legs are associated with osteomalacia. Low intake of dairy products is a risk factor for osteoporosis, but it does not indicate that osteoporosis is present. Frequent falls increase the risk for fractures but are not an indicator of osteoporosis.

When caring for a patient who experienced a T2 spinal cord transection 24 hours ago, which collaborative and nursing actions will the nurse include in the plan of care (select all that apply)? a. Urinary catheter care b. Nasogastric (NG) tube feeding c. Continuous cardiac monitoring d. Administration of H2 receptor blockers e. Maintenance of a warm room temperature

ANS: A, C, D, E The patient is at risk for bradycardia and poikilothermia caused by sympathetic nervous system dysfunction and should have continuous cardiac monitoring and maintenance of a relatively warm room temperature. To avoid bladder distention, a urinary retention catheter is used during this acute phase. Stress ulcers are a common complication, but can be avoided through the use of the H2 receptor blockers such as famotidine. Gastrointestinal motility is decreased initially, and NG suctioning is indicated. DIF: Cognitive Level: Apply (application)

Which action will the nurse include in the plan of care for a 33-year-old patient with a new diagnosis of rheumatoid arthritis? a. Instruct the patient to purchase a soft mattress. b. Suggest that the patient take a nap in the afternoon. c. Teach the patient to use lukewarm water when bathing. d. Suggest exercise with light weights several times daily.

ANS: B Adequate rest helps decrease the fatigue and pain that are associated with rheumatoid arthritis. Patients are taught to avoid stressing joints, to use warm baths to relieve stiffness, and to use a firm mattress. When stabilized, a therapeutic exercise program is usually developed by a physical therapist to include exercises that improve the flexibility and strength of the affected joints, and the patient's overall endurance.

21. Which assessment finding for a patient who is receiving furosemide (Lasix) to treat stage 2 hypertension is most important to report to the health care provider? a. Blood glucose level of 180 mg/dL b. Blood potassium level of 3.0 mEq/L c. Early morning BP reading of 164/96 mm Hg d. Orthostatic systolic BP decrease of 12 mm Hg

ANS: B Hypokalemia is a frequent adverse effect of the loop diuretics and can cause life-threatening dysrhythmias. The health care provider should be notified of the potassium level immediately and administration of potassium supplements initiated. The elevated blood glucose and BP also indicate a need for collaborative interventions but will not require action as urgently as the hypokalemia. An orthostatic drop of 12 mm Hg is common and will require intervention only if the patient is symptomatic.

A construction worker arrives at an urgent care center with a deep puncture wound from a rusty nail. The patient reports having had a tetanus booster 6 years ago. The nurse will anticipate a. IV infusion of tetanus immune globulin (TIG). b. administration of the tetanus-diphtheria (Td) booster. c. intradermal injection of an immune globulin test dose. d. initiation of the tetanus-diphtheria immunization series.

ANS: B If the patient has not been immunized in the past 5 years, administration of the Td booster is indicated because the wound is deep. Immune globulin administration is given by the IM route if the patient has no previous immunization. Administration of a series of immunization is not indicated. TIG is not indicated for this patient, and a test dose is not needed for immune globulin. DIF: Cognitive Level: Apply (application)

A 40-year-old African American patient has scleroderma manifested by CREST (calcinosis, Raynaud's phenomenon, esophageal dysfunction, sclerodactyly, and telangiectasia) syndrome. Which action will the nurse include in the plan of care? a. Avoid use of capsaicin cream on hands. b. Keep environment warm and draft free. c. Obtain capillary blood glucose before meals. d. Assist to bathroom every 2 hours while awake.

ANS: B Keeping the room warm will decrease the incidence of Raynaud's phenomenon, one aspect of the CREST syndrome. Capsaicin cream may be used to improve circulation and decrease pain. There is no need to obtain blood glucose levels or to assist the patient to the bathroom every 2 hours.

The health care provider has prescribed the following collaborative interventions for a patient who is taking azathioprine (Imuran) for systemic lupus erythematosus. Which order will the nurse question? a. Draw anti-DNA blood titer. b. Administer varicella vaccine. c. Naproxen (Aleve) 200 mg BID. d. Famotidine (Pepcid) 20 mg daily.

ANS: B Live virus vaccines, such as varicella, are contraindicated in a patient taking immunosuppressive drugs. The other orders are appropriate for the patient

A patient with gout has a new prescription for losartan (Cozaar) to control the condition. The nurse will plan to monitor a. blood glucose. b. blood pressure. c. erythrocyte count. d. lymphocyte count.

ANS: B Losartan, an angiotensin II receptor antagonist, will lower blood pressure. It does not affect blood glucose, red blood cell (RBC) count, or lymphocytes.

A 31-year-old woman is taking methotrexate (Rheumatrex) to treat rheumatoid arthritis. Which information from the patient's health history is important for the nurse to report to the health care provider about the methotrexate? a. The patient had a history of infectious mononucleosis as a teenager. b. The patient is trying to get pregnant before her disease becomes more severe. c. The patient has a family history of age-related macular degeneration of the retina. d. The patient has been using large doses of vitamins and health foods to treat the RA.

ANS: B Methotrexate is teratogenic, and the patient should be taking contraceptives during methotrexate therapy. The other information will not impact the choice of methotrexate as therapy.

Which nursing action can the registered nurse (RN) delegate to unlicensed assistive personnel (UAP) who are assisting with the care of a patient with scleroderma? a. Monitor for difficulty in breathing. b. Document the patient's oral intake. c. Check finger strength and movement. d. Apply capsaicin (Zostrix) cream to hands.

ANS: B Monitoring and documenting patients' oral intake is included in UAP education and scope of practice. Assessments for changes in physical status and administration of medications require more education and scope of practice, and should be done by licensed nurses

The nurse is admitting a patient who has a neck fracture at the C6 level to the intensive care unit. Which assessment findings indicate neurogenic shock? a. Involuntary and spastic movement b. Hypotension and warm extremities c. Hyperactive reflexes below the injury d. Lack of sensation or movement below the injury

ANS: B Neurogenic shock is characterized by hypotension, bradycardia, and vasodilation leading to warm skin temperature. Spasticity and hyperactive reflexes do not occur at this stage of spinal cord injury. Lack of movement and sensation indicate spinal cord injury but not neurogenic shock. DIF: Cognitive Level: Understand (comprehension)

After the nurse has finished teaching a 68-year-old patient with osteoarthritis (OA) of the right hip about how to manage the OA, which patient statement indicates a need for more teaching? a. "I can take glucosamine to help decrease my knee pain." b. "I will take 1 g of acetaminophen (Tylenol) every 4 hours." c. "I will take a shower in the morning to help relieve stiffness." d. "I can use a cane to decrease the pressure and pain in my hip."

ANS: B No more than 4 g of acetaminophen should be taken daily to avoid liver damage. The other patient statements are correct and indicate good understanding of OA management.

A patient in the clinic reports a recent episode of dysphasia and left-sided weakness at home that resolved after 2 hours. The nurse will anticipate teaching the patient about a. tPA. c. warfarin (Coumadin). b. aspirin . d. nimodipine

ANS: B After a transient ischemic attack, patients typically are started on medications such as aspirin to inhibit platelet function and decrease stroke risk. tPA is used for acute ischemic stroke. Coumadin is usually used for patients with atrial fibrillation. Nimodipine is used to prevent cerebral vasospasm after a subarachnoid hemorrhage. DIF: Cognitive Level: Apply (application) REF: 1353 TOP: Nursing Process: Planning MSC: NCLEX: Physiological Integrity

A patient with paraplegia resulting from a T9 spinal cord injury has a neurogenic reflexic bladder. Which action will the nurse include in the plan of care? a. Teach the patient the Credé method. b. Instruct the patient how to self-catheterize. c. Catheterize for residual urine after voiding. d. Assist the patient to the toilet every 2 hours.

ANS: B Because the patient's bladder is spastic and will empty in response to overstretching of the bladder wall, the most appropriate method is to avoid incontinence by emptying the bladder at regular intervals through intermittent catheterization. Assisting the patient to the toilet will not be helpful because the bladder will not empty. The Credé method is more appropriate for a bladder that is flaccid, such as occurs with areflexic neurogenic bladder. Catheterization after voiding will not resolve the patient's incontinence. DIF: Cognitive Level: Apply (application)

Which nursing action is a priority for a patient who has suffered a burn injury while working on an electrical power line? a. Obtain the blood pressure. b. Stabilize the cervical spine. c. Assess for the contact points. d. Check alertness and orientation.

ANS: B Cervical spine injuries are commonly associated with electrical burns. Therefore stabilization of the cervical spine takes precedence after airway management. The other actions are also included in the emergent care after electrical burns, but the most important action is to avoid spinal cord injury.

Which information will the nurse include when preparing teaching materials for patients with exacerbations of rheumatoid arthritis? a. Affected joints should not be exercised when pain is present. b. Application of cold packs before exercise may decrease joint pain. c. Exercises should be performed passively by someone other than the patient. d. Walking may substitute for range-of-motion (ROM) exercises on some days.

ANS: B Cold application is helpful in reducing pain during periods of exacerbation of RA. Because the joint pain is chronic, patients are instructed to exercise even when joints are painful. ROM exercises are intended to strengthen joints and improve flexibility, so passive ROM alone is not sufficient. Recreational exercise is encouraged but is not a replacement for ROM exercises.

A 46-year-old male patient with dermatomyositis is receiving long-term prednisone (Deltasone) therapy. Which assessment finding by the nurse is most important to report to the health care provider? a. The blood glucose is 112 mg/dL. b. The patient has painful hematuria. c. Acne is noted on the patient's face. d. The patient has an increased appetite.

ANS: B Corticosteroid use is associated with an increased risk for infection, so the nurse should report the urinary tract symptoms immediately to the health care provider. The increase in blood glucose, increased appetite, and acne are also adverse effects of corticosteroid use but do not need diagnosis and treatment as rapidly as the probable urinary tract infection.

Which patient assessment will help the nurse identify potential complications of trigeminal neuralgia? a. Have the patient clench the jaws. b. Inspect the oral mucosa and teeth. c. Palpate the face to compare skin temperature bilaterally. d. Identify trigger zones by lightly touching the affected side.

ANS: B Oral hygiene is frequently neglected because of fear of triggering facial pain and may lead to gum disease, dental caries, or an abscess. Having the patient clench the facial muscles will not be useful because the sensory branches (rather than motor branches) of the nerve are affected by trigeminal neuralgia. Light touch and palpation may be triggers for pain and should be avoided. DIF: Cognitive Level: Apply (application)

Which statement by a patient with systemic lupus erythematosus (SLE) indicates that the patient has understood the nurse's teaching about the condition? a. "I will exercise even if I am tired." b. "I will use sunscreen when I am outside." c. "I should take birth control pills to keep from getting pregnant." d. "I should avoid aspirin or nonsteroidal antiinflammatory drugs."

ANS: B Severe skin reactions can occur in patients with SLE who are exposed to the sun. Patients should avoid fatigue by balancing exercise with rest periods as needed. Oral contraceptives can exacerbate lupus. Aspirin and nonsteroidal antiinflammatory drugs are used to treat the musculoskeletal manifestations of SLE.

A nurse is caring for a patient who has burns of the ears, head, neck, and right arm and hand. The nurse should place the patient in which position? a. Place the right arm and hand flexed in a position of comfort. b. Elevate the right arm and hand on pillows and extend the fingers. c. Assist the patient to a supine position with a small pillow under the head. d. Position the patient in a side-lying position with rolled towel under the neck.

ANS: B The right hand and arm should be elevated to reduce swelling and the fingers extended to avoid flexion contractures (even though this position may not be comfortable for the patient). The patient with burns of the ears should not use a pillow for the head because this will put pressure on the ears, and the pillow may stick to the ears. Patients with neck burns should not use a pillow because the head should be maintained in an extended position in order to avoid contractures.

Before administering botulinum antitoxin to a patient in the emergency department, it is most important for the nurse to a. obtain the patient's temperature. b. administer an intradermal test dose. c. document the neurologic symptoms. d. ask the patient about an allergy to eggs.

ANS: B To assess for possible allergic reactions, an intradermal test dose of the antitoxin should be administered. Although temperature, allergy history, and symptom assessment and documentation are appropriate, these assessments will not affect the decision to administer the antitoxin. DIF: Cognitive Level: Analyze (analysis)

The nurse is caring for a patient who has just returned after having left carotid artery angioplasty and stenting. Which assessment information is of most concern to the nurse? a. The pulse rate is 102 beats/min. b. The patient has difficulty speaking. c. The blood pressure is 144/86 mm Hg. d. There are fine crackles at the lung bases.

ANS: B Small emboli can occur during carotid artery angioplasty and stenting, and the aphasia indicates a possible stroke during the procedure. Slightly elevated pulse rate and blood pressure are not unusual because of anxiety associated with the procedure. Fine crackles at the lung bases may indicate atelectasis caused by immobility during the procedure. The nurse should have the patient take some deep breaths. DIF: Cognitive Level: Analyze (analysis) REF: 1351 OBJ: Special Questions: Prioritization TOP: Nursing Process: Assessment MSC: NCLEX: Physiological Integrity

Which result for a 30-year-old patient with systemic lupus erythematosus (SLE) is most important for the nurse to communicate to the health care provider? a. Decreased C-reactive protein (CRP) b. Elevated blood urea nitrogen (BUN) c. Positive antinuclear antibodies (ANA) d. Positive lupus erythematosus cell prep

ANS: B The elevated BUN and creatinine levels indicate possible lupus nephritis and a need for a change in therapy to avoid further renal damage. The positive lupus erythematosus (LE) cell prep and ANA would be expected in a patient with SLE. A drop in CRP shows an improvement in the inflammatory process.

Which information about the patient who has had a subarachnoid hemorrhage is most important to communicate to the health care provider? a. The patient complains of having a stiff neck. b. The patient's blood pressure (BP) is 90/50 mm Hg. c. The patient reports a severe and unrelenting headache. d. The cerebrospinal fluid (CSF) report shows red blood cells (RBCs).

ANS: B To prevent cerebral vasospasm and maintain cerebral perfusion, BP needs to be maintained at a level higher than 90 mm Hg systolic after a subarachnoid hemorrhage. A low BP or drop in BP indicates a need to administer fluids and/or vasopressors to increase the BP. An ongoing headache, RBCs in the CSF, and a stiff neck are all typical clinical manifestations of a subarachnoid hemorrhage and do not need to be rapidly communicated to the health care provider. DIF: Cognitive Level: Analyze (analysis) REF: 1350 OBJ: Special Questions: Prioritization TOP: Nursing Process: Assessment MSC: NCLEX: Physiological Integrity

A 70-yr-old female patient with left-sided hemiparesis arrives by ambulance to the emergency department. Which action should the nurse take first? a. Take the patient's blood pressure. b. Check the respiratory rate and effort. c. Assess the Glasgow Coma Scale score. d. Send the patient for a computed tomography (CT) scan.

ANS: B The initial nursing action should be to assess the airway and take any needed actions to ensure a patent airway. The other activities should take place quickly after the ABCs (airway, breathing, and circulation) are completed. DIF: Cognitive Level: Analyze (analysis) REF: 1354 OBJ: Special Questions: Prioritization TOP: Nursing Process: Implementation MSC: NCLEX: Physiological Integrity

While the patient's full-thickness burn wounds to the face are exposed, what is the best nursing action to prevent cross contamination? a. Use sterile gloves when removing old dressings. b. Wear gowns, caps, masks, and gloves during all care of the patient. c. Administer IV antibiotics to prevent bacterial colonization of wounds. d. Turn the room temperature up to at least 70° F (20° C) during dressing changes.

ANS: B Use of gowns, caps, masks, and gloves during all patient care will decrease the possibility of wound contamination for a patient whose burns are not covered. When removing contaminated dressings and washing the dirty wound, use nonsterile, disposable gloves. The room temperature should be kept at approximately 85° F for patients with open burn wounds to prevent shivering. Systemic antibiotics are not well absorbed into deep burns because of the lack of circulation.

A patient has chronic osteomyelitis of the left femur, which is being managed at home with administration of IV antibiotics. The nurse chooses a nursing diagnosis of ineffective health maintenance when the nurse finds that the patient a. takes and records the oral temperature twice a day. b. is unable to plantar flex the foot on the affected side. c. uses crutches to avoid weight bearing on the affected leg. d. is irritable and frustrated with the length of treatment required.

ANS: B Foot drop is an indication that the foot is not being supported in a neutral position by a splint. Using crutches and monitoring the oral temperature are appropriate self-care activities. Frustration with the length of treatment is not an indicator of ineffective health maintenance of the osteomyelitis.

Which information will the nurse include when teaching a patient with newly diagnosed chronic fatigue syndrome about self-management? a. Avoid use of over-the-counter antihistamines or decongestants. b. A low-residue, low-fiber diet will reduce any abdominal distention. c. A gradual increase in your daily exercise may help decrease fatigue. d. Chronic fatigue syndrome usually progresses as patients become older.

ANS: C A graduated exercise program is recommended to avoid fatigue while encouraging ongoing activity. Because many patients with chronic fatigue syndrome have allergies, antihistamines and decongestants are used to treat allergy symptoms. A high-fiber diet is recommended. Chronic fatigue syndrome usually does not progress

A patient is being admitted with a possible stroke. Which information from the assessment indicates that the nurse should consult with the health care provider before giving the prescribed aspirin? a. The patient has dysphasia. b. The patient has atrial fibrillation. c. The patient reports that symptoms began with a severe headache. d. The patient has a history of brief episodes of right-sided hemiplegia.

ANS: C A sudden onset headache is typical of a subarachnoid hemorrhage, and aspirin is contraindicated. Atrial fibrillation, dysphasia, and transient ischemic attack are not contraindications to aspirin use. DIF: Cognitive Level: Apply (application) REF: 1348 TOP: Nursing Process: Assessment MSC: NCLEX: Physiological Integrity

The nurse assessing a patient with newly diagnosed trigeminal neuralgia will ask the patient about a. visual problems caused by ptosis. b. triggers leading to facial discomfort. c. poor appetite caused by loss of taste. d. weakness on the affected side of the face.

ANS: B The major clinical manifestation of trigeminal neuralgia is severe facial pain triggered by cutaneous stimulation of the nerve. Ptosis, loss of taste, and facial weakness are not characteristics of trigeminal neuralgia. DIF: Cognitive Level: Apply (application)

When the nurse is developing a rehabilitation plan for a 30-yr-old patient with a C6 spinal cord injury, an appropriate goal is that the patient will be able to a. drive a car with powered hand controls. b. push a manual wheelchair on a flat surface. c. turn and reposition independently when in bed. d. transfer independently to and from a wheelchair.

ANS: B The patient with a C6 injury will be able to use the hands to push a wheelchair on flat, smooth surfaces. Because flexion of the thumb and fingers is minimal, the patient will not be able to grasp a wheelchair during transfer, drive a car with powered hand controls, or turn independently in bed. DIF: Cognitive Level: Apply (application)

The nurse will explain to the patient who has a T2 spinal cord transection injury that a. use of the shoulders will be limited. b. function of both arms should be retained. c. total loss of respiratory function may occur. d. tachycardia is common with this type of injury.

ANS: B The patient with a T2 injury can expect to retain full motor and sensory function of the arms. Use of only the shoulders is associated with cervical spine injury. Loss of respiratory function occurs with cervical spine injuries. Bradycardia is associated with injuries above the T6 level. DIF: Cognitive Level: Understand (comprehension)

14. The nurse in the emergency department received change-of-shift report on four patients with hypertension. Which patient should the nurse assess first? a. 52-year-old with a BP of 212/90 who has intermittent claudication b. 43-year-old with a BP of 190/102 who is complaining of chest pain c. 50-year-old with a BP of 210/110 who has a creatinine of 1.5 mg/dL d. 48-year-old with a BP of 200/98 whose urine shows microalbuminuria

ANS: B The patient with chest pain may be experiencing acute myocardial infarction and rapid assessment and intervention is needed. The symptoms of the other patients also show target organ damage, but are not indicative of acute processes.

For a patient who had a right hemisphere stroke, the nurse anticipates planning interventions to manage a. impaired physical mobility related to right-sided hemiplegia. b. risk for injury related to denial of deficits and impulsiveness. c. impaired verbal communication related to speech-language deficits. d. ineffective coping related to depression and distress about disability.

ANS: B The patient with right-sided brain damage typically denies any deficits and has poor impulse control, leading to risk for injury when the patient attempts activities such as transferring from a bed to a chair. Right-sided brain damage causes left hemiplegia. Left-sided brain damage typically causes language deficits. Left-sided brain damage is associated with depression and distress about the disability. DIF: Cognitive Level: Apply (application) REF: 1350 TOP: Nursing Process: Diagnosis MSC: NCLEX: Physiological Integrity

A patient with rheumatoid arthritis (RA) complains to the clinic nurse about having chronically dry eyes. Which action by the nurse is most appropriate? a. Teach the patient about adverse effects of the RA medications. b. Suggest that the patient use over-the-counter (OTC) artificial tears. c. Reassure the patient that dry eyes are a common problem with RA. d. Ask the health care provider about discontinuing methotrexate (Rheumatrex) .

ANS: B The patient's dry eyes are consistent with Sjögren's syndrome, a common extraarticular manifestation of RA. Symptomatic therapy such as OTC eye drops is recommended. Dry eyes are not a side effect of methotrexate. Although dry eyes are common in RA, it is more helpful to offer a suggestion to relieve these symptoms than to offer reassurance. The dry eyes are not caused by RA treatment, but by the disease itself

17. The nurse is assessing a patient who has been admitted to the intensive care unit (ICU) with a hypertensive emergency. Which finding is most important to report to the health care provider? a. Urine output over 8 hours is 200 mL less than the fluid intake. b. The patient is unable to move the left arm and leg when asked to do so. c. Tremors are noted in the fingers when the patient extends the arms. d. The patient complains of a severe headache with pain at level 8/10 (0 to 10 scale).

ANS: B The patient's inability to move the left arm and leg indicates that a hemorrhagic stroke may be occurring and will require immediate action to prevent further neurologic damage. The other clinical manifestations also likely are caused by the hypertension and will require rapid nursing actions, but they do not require action as urgently as the neurologic changes.

2. The nurse obtains this information from a patient with prehypertension. Which finding is most important to address with the patient? a. Low dietary fiber intake b. No regular aerobic exercise c. Weight 5 pounds above ideal weight d. Drinks wine with dinner once a week

ANS: B The recommendations for preventing hypertension include exercising aerobically for 30 minutes most days of the week. A weight that is 5 pounds over the ideal body weight is not a risk factor for hyperten- sion. The Dietary Approaches to Stop Hypertension (DASH) diet is high in fiber, but increasing fiber alone will not prevent hypertension from developing. The patient's alcohol intake will not increase the hypertension risk.

What should the nurse instruct the patient to do to best enhance the effectiveness of a daily dose of docusate sodium (Colace)? a. Take a dose of mineral oil at the same time. b. Add extra salt to food on at least one meal tray. c. Ensure dietary intake of 10 g of fiber each day. d. Take each dose with a full glass of water or other liquid.

d. Take each dose with a full glass of water or other liquid. Docusate lowers the surface tension of stool, permitting water and fats to penetrate and soften the stool for easier passage. The patient should take the dose with a full glass of water and should increase overall fluid intake, if able, to enhance effectiveness of the medication. Dietary fiber intake should be a minimum of 20 g daily to prevent constipation. Mineral oil and extra salt are not recommended.

16. Which intervention has the highest priority when caring for a client diagnosed with RA? a. Encourage the client to ventilate feelings about the disease process b. Discuss the effects of disease on the client's career and other life roles c. Instruct the client to perform most important activities in the morning d. Teach the client the proper use of hot and cold therapy to provide pain relief

d. Teach the client the proper use of hot and cold therapy to provide pain relief

12. The nurse is assessing a client diagnosed with RA. Which assessment findings warrant immediate intervention? a. The client complains of joint stiffness and the knees feel warm to the touch b. The client has experienced one kg weight loss and is very tired c. The client requires a heating pad applied to the hips and back to sleep d. The client is crying, has a flat facial affect, and refuses to speak to the nurse

d. The client is crying, has a flat facial affect, and refuses to speak to the nurse

A victim of an industrial accident has chemical spilled on his face and body. The chemical, which has a pH of 7.51, is flushed with water by paramedics. What is the most important information for the receiving nurse to obtain about the pt from the paramedics? a. containment of chemical b. duration of water flushing c. other injuries of the victim d. specific location of accident

d. The nurse must know where the accident occurred to determine if the pt was rescued from an enclosed space. If so, the pt is at high risk for an inhalation injury because the enclosure concentrates the noxious fumes making an inhalation injury more likely.

A 62-yr-old patient who has *Parkinson's* disease is taking *bromocriptine (Parlodel)*. Which information obtained by the nurse may indicate a *need for a decrease* in the dosage? a. The patient has a chronic dry cough. b. The patient has four loose stools in a day. c. The patient develops a deep vein thrombosis. d. The patient's blood pressure is 92/52 mm Hg.

d. The patient's blood pressure is 92/52 mm Hg. *Hypotension* is an *adverse* effect of *bromocriptine*, and the nurse should check with the health care provider before giving the medication.

6. Which guideline would be a part of teaching patients how to use a metered-dose inhaler (MDI)? a. After activating the MDI, breathe in as quickly as you can. b. Estimate the amount of remaining medicine in the MDI by floating the canister in water. c. Disassemble the plastic canister from the inhaler and rinse both pieces under running water every week. d. To determine how long the canister will last, divide the total number of puffs in the canister by the puffs needed per day.

d. To determine how long the canister will last, divide the total number of puffs in the canister by the puffs needed per day.

What should a patient be taught after a hemorrhoidectomy? a. Take a mineral oil before bedtime b. Eat a low-fiber diet to rest the colon c. Administer oil-retention enema to empty the colon d. Use prescribed pain medication before a bowel movement

d. Use prescribed pain medication before a bowel movement Rationale: After a hemorrhoidectomy, the patient usually dreads the first bowel movement and often resists the urge to defecate. Pain medication may be given before the bowel movement to reduce discomfort. The patient should avoid constipation and straining. A high-fiber diet can reduce constipation. A stool softener such as docusate (Colace) is usually ordered for the first few postoperative days. If the patient does not have a bowel movement within 2 to 3 days, an oil-retention enema is administered.

A patient has 25% TBSA burned from a car fire. His wounds have been debrided and covered with a silver-impregnated dressing. Your priority intervention for wound care is to: a. reapply a new dressing without disturbing the wound bed b. observe the wound for signs of infection during dressing changes c. apply cool compresses for pain relief in between dressing changes d. wash the wound aggressively with soap and water three times each day.

d. wash the wound aggressively with soap and water three times each day. Infection is the most serious threat for further tissue injury and possible sepsis.

A female pt in the acute phase of burn care has electrical burns on the left side of her body, type 2 diabetes mellitus, and a serum glucose of 485 mg/dL. What is the nurse's priority for preventing a life threatening complication of hyperglycemia for the burn patient? a. replace the blood lost b. maintain a neutral pH c. Maintain fluid balance d. Replace serum potassium

c. this pt is most likely experiencing hyperglycemic hyperosmolar nonketotic syndrome (HHNKS) which dehydrates a patient rapidly. This increases the pt's risk for hypovolemia and hypotension.

3. In teaching a patient with hypertension about controlling the illness, the nurse recognizes that a. all patients with elevated BP require medication. b. obese persons must achieve a normal weight to lower BP. c. it is not necessary to limit salt in the diet if taking a diuretic. d. lifestyle modifications are indicated for all persons with elevated BP.

d

4. A priority consideration in the management of the older adult with hypertension is to a. prevent primary hypertension from converting to secondary hypertension. b. recognize that the older adult is less likely to adhere to the drug therapy regimen than a younger adult. c. ensure that the patient receives larger initial doses of antihypertensive drugs because of impaired absorption. d. use careful technique in assessing the BP of the patient because of the possible presence of an auscultatory gap.

d

When monitoring initial fluid replacement for the patient with 40% TBSA deep partial-thickness and full-thickness burns, which finding is of most concern to the nurse? a) Serum K+ of 4.5 mEq/L b) Urine output of 35 mL/hr c) Decreased bowel sounds d) Blood pressure of 86/72 mm Hg

d Rationale: Adequacy of fluid replacement is assessed by urine output and cardiac parameters. Urine output should be 0.5 to 1 mL/kg/hr. Mean arterial pressure should be >65 mm Hg, systolic BP >90 mm Hg, and heart rate <120 beats/min. A blood pressure of 86/72 indicates inadequate fluid replacement. However, the MAP is calculated at 77 mm Hg.

The nurse plans emergent care for four male pt's who have burns covering between 40-50% of the total body surface area. Rank these patients according to their risk for an inhalation injury beginning with the pt who has the highest risk. a. has posterior chemical burns from an exhibit at a parking lot b. has osteoporosis and electrical burns of the lower extremities c. has thermal burns of the right side and is a volunteer fireman d. has chronic bronchitis and thermal burns around the abdomen

d, c, a, b.

The client arrives at the emergency department following a burn injury that occurred in the basement at home and an inhalation injury is suspected. Which of the following would the nurse anticipate to be prescribed for the client? a. 100% oxygen via an aerosol mask b. Oxygen via nasal cannula at 15L/min c. Oxygen via nasal cannula at 10L/min d. 100% oxygen via a tight fitting, non rebreather face mask

d.

23. The client recently diagnosed with RA is prescribed aspirin, an NSAID medication. Which comment by the client would warrant immediate intervention by the nurse a. "I always take the aspirin with food" b. "If I have dark stools, I will call my HCP" c. "Aspirin will not cure my arthritis" d. "I am having some ringing in my ears"

d. "I am having some ringing in my ears"

A patient with a T4 spinal cord injury asks the nurse if he will be able to be sexually active. Which initial response by the nurse is best? a. Reflex erections frequently occur, but orgasm may not be possible. b. Sildenafil (Viagra) is used by many patients with spinal cord injury. c. Multiple options are available to maintain sexuality after spinal cord injury. d. Penile injection, prostheses, or vacuum suction devices are possible options.

ANS: C Although sexuality will be changed by the patient's spinal cord injury, there are options for expression of sexuality and for fertility. The other information also is correct, but the choices will depend on the degrees of injury and the patient's individual feelings about sexuality. DIF: Cognitive Level: Analyze (analysis)

Anakinra (Kineret) is prescribed for a 49-year-old patient who has rheumatoid arthritis (RA). When teaching the patient about this drug, the nurse will include information about a. avoiding concurrently taking aspirin. b. symptoms of gastrointestinal (GI) bleeding. c. self-administration of subcutaneous injections. d. taking the medication with at least 8 oz of fluid.

ANS: C Anakinra is administered by subcutaneous injection. GI bleeding is not a side effect of this medication. Because the medication is injected, instructions to take it with 8 oz of fluid would not be appropriate. The patient is likely to be concurrently taking aspirin or nonsteroidal antiinflammatory drugs (NSAIDs), and these should not be discontinued

A patient with an acute attack of gout in the right great toe has a new prescription for probenecid (Benemid). Which information about the patient's home routine indicates a need for teaching regarding gout management? a. The patient sleeps about 8 to 10 hours every night. b. The patient usually eats beef once or twice a week. c. The patient takes one aspirin a day to prevent angina. d. The patient usually drinks about 3 quarts water daily.

ANS: C Aspirin interferes with the effectiveness of probenecid and should not be taken when the patient is taking probenecid. The patient's sleep pattern will not affect gout management. Drinking 3 quarts of water and eating beef only once or twice a week are appropriate for the patient with gout.

A patient who has a history of a transient ischemic attack (TIA) has an order for aspirin 160 mg daily. When the nurse is administering medications, the patient says, "I don't need the aspirin today. I don't have a fever." Which action should the nurse take? a. Document that the aspirin was refused by the patient. b. Tell the patient that the aspirin is used to prevent a fever. c. Explain that the aspirin is ordered to decrease stroke risk. d. Call the health care provider to clarify the medication order.

ANS: C Aspirin is ordered to prevent stroke in patients who have experienced TIAs. Documentation of the patient's refusal to take the medication is an inadequate response by the nurse. There is no need to clarify the order with the health care provider. The aspirin is not ordered to prevent aches and pains. DIF: Cognitive Level: Apply (application) REF: 1353 TOP: Nursing Process: Implementation MSC: NCLEX: Physiological Integrity

The nurse is planning care for a patient with hypertension and gout who has a red and painful right great toe. Which nursing action will be included in the plan of care? a. Gently palpate the toe to assess swelling. b. Use pillows to keep the right foot elevated. c. Use a footboard to hold bedding away from the toe. d. Teach patient to avoid use of acetaminophen (Tylenol).

ANS: C Because any touch on the area of inflammation may increase pain, bedding should be held away from the toe and touching the toe will be avoided. Elevation of the foot will not reduce the pain, which is caused by urate crystals. Acetaminophen can be used for pain relief.

A patient admitted with possible stroke has been aphasic for 3 hours, and his current blood pressure (BP) is 174/94 mm Hg. Which order by the health care provider should the nurse question? a. Keep head of bed elevated at least 30 degrees. b. Infuse normal saline intravenously at 75 mL/hr. c. Start a labetalol drip to keep BP less than 140/90 mm Hg. d. Administer tissue plasminogen activator (tPA) intravenously per protocol.

ANS: C Because elevated BP may be a protective response to maintain cerebral perfusion, antihypertensive therapy is recommended only if mean arterial pressure (MAP) is greater than130 mm Hg or systolic pressure is greater than 220 mm Hg. Fluid intake should be 1500 to 2000 mL/day to maintain cerebral blood flow. The head of the bed should be elevated to at least 30 degrees unless the patient has symptoms of poor tissue perfusion. tPA may be administered if the patient meets the other criteria for tPA use. DIF: Cognitive Level: Apply (application) REF: 1354 TOP: Nursing Process: Implementation MSC: NCLEX: Physiological Integrity

When evaluating outcomes of a glycerol rhizotomy for a patient with trigeminal neuralgia, the nurse will a. assess if the patient is doing daily facial exercises. b. question if the patient is using an eye shield at night. c. ask the patient about social activities with family and friends. d. remind the patient to chew on the unaffected side of the mouth.

ANS: C Because withdrawal from social activities is a common manifestation of trigeminal neuralgia, asking about social activities will help in evaluating if the patient's symptoms have improved. Glycerol rhizotomy does not damage the corneal reflex or motor functions of the trigeminal nerve, so there is no need to use an eye shield, do facial exercises, or take precautions with chewing. DIF: Cognitive Level: Apply (application)

Which laboratory data is important to communicate to the health care provider for a patient who is taking methotrexate (Rheumatrex) to treat rheumatoid arthritis? a. The blood glucose is 90 mg/dL. b. The rheumatoid factor is positive. c. The white blood cell (WBC) count is 1500/µL. d. The erythrocyte sedimentation rate is elevated.

ANS: C Bone marrow suppression is a possible side effect of methotrexate, and the patient's low WBC count places the patient at high risk for infection. The elevated erythrocyte sedimentation rate and positive rheumatoid factor are expected in rheumatoid arthritis. The blood glucose is normal.

Which laboratory result will the nurse monitor to determine whether prednisone (Deltasone) has been effective for a 30-year-old patient with an acute exacerbation of rheumatoid arthritis? a. Blood glucose test b. Liver function tests c. C-reactive protein level d. Serum electrolyte levels

ANS: C C-reactive protein is a marker for inflammation, and a decrease would indicate that the corticosteroid therapy was effective. Blood glucose and serum electrolyte levels will also be monitored to check for side effects of prednisone. Liver function is not routinely monitored in patients receiving corticosteroids.

The nurse will anticipate the need to teach a 57-year-old patient who has osteoarthritis (OA) about which medication? a. Adalimumab (Humira) b. Prednisone (Deltasone) c. Capsaicin cream (Zostrix) d. Sulfasalazine (Azulfidine)

ANS: C Capsaicin cream blocks the transmission of pain impulses and is helpful for some patients in treating OA. The other medications would be used for patients with RA.

Which finding in a patient with a spinal cord tumor requires an immediate report to the health care provider? a. Depression about the diagnosis b. Anxiety about scheduled surgery c. Decreased ability to move the legs d. Back pain that worsens with coughing

ANS: C Decreasing sensation and leg movement indicates spinal cord compression, an emergency that will require rapid action (such as surgery) to prevent paralysis. The other findings will also require nursing action but are not emergencies. DIF: Cognitive Level: Apply (application)

When the nurse brings medications to a patient with rheumatoid arthritis, the patient refuses the prescribed methotrexate (Rheumatrex). The patient tells the nurse, "My arthritis isn't that bad yet. The side effects of methotrexate are worse than the arthritis." The most appropriate response by the nurse is a. "You have the right to refuse to take the methotrexate." b. "Methotrexate is less expensive than some of the newer drugs." c. "It is important to start methotrexate early to decrease the extent of joint damage." d. "Methotrexate is effective and has fewer side effects than some of the other drugs."

ANS: C Disease-modifying antirheumatic drugs (DMARDs) are prescribed early to prevent the joint degeneration that occurs as soon as the first year with RA. The other statements are accurate, but the most important point for the patient to understand is that it is important to start DMARDs as quickly as possible.

When caring for a patient with a new right-sided homonymous hemianopsia resulting from a stroke, which intervention should the nurse include in the plan of care? a. Apply an eye patch to the right eye. b. Approach the patient from the right side. c. Place needed objects on the patient's left side. d. Teach the patient that the left visual deficit will resolve.

ANS: C During the acute period, the nurse should place objects on the patient's unaffected side. Because there is a visual defect in the right half of each eye, an eye patch is not appropriate. The patient should be approached from the left side. The visual deficit may not resolve, although the patient can learn to compensate for the defect. DIF: Cognitive Level: Apply (application) REF: 1362 TOP: Nursing Process: Planning MSC: NCLEX: Physiological Integrity

To prevent autonomic hyperreflexia, which nursing action will the home health nurse include in the plan of care for a patient who has paraplegia at the T4 level ? a. Support selection of a high-protein diet. b. Discuss options for sexuality and fertility. c. Assist in planning a prescribed bowel program. d. Use quad coughing to strengthen cough efforts.

ANS: C Fecal impaction is a common stimulus for autonomic hyperreflexia. Dietary protein, coughing, and discussing sexuality and fertility should be included in the plan of care but will not reduce the risk for autonomic hyperreflexia. DIF: Cognitive Level: Apply (application)

19. The RN is caring for a patient with a hypertensive crisis who is receiving sodium nitroprusside (Nipride). Which of the following nursing actions can the nurse delegate to an experienced LPN/LVN? a. Titrate nitroprusside to maintain BP at 160/100 mm Hg. b. Evaluate effectiveness of nitroprusside therapy on BP. c. Set up the automatic blood pressure machine to take BP every 15 minutes. d. Assess the patient's environment for adverse stimuli that might increase BP.

ANS: C LPN/LVN education and scope of practice include correct use of common equipment such as automatic blood pressure machines. The other actions require more nursing judgment and education and should be done by RNs.

3. After giving a patient the initial dose of oral labetalol (Normodyne) for treatment of hypertension, which action should the nurse take? a. Encourage oral fluids to prevent dry mouth or dehydration. b. Instruct the patient to ask for help if heart palpitations occur. c. Ask the patient to request assistance when getting out of bed. d. Teach the patient that headaches may occur with this medication.

ANS: C Labetalol decreases sympathetic nervous system activity by blocking both α- and β-adrenergic receptors, leading to vasodilation and a decrease in heart rate, which can cause severe orthostatic hypotension. Heart palpitations, dehydration, and headaches are possible side effects of other antihypertensives.

A 19-year-old patient hospitalized with a fever and red, hot, and painful knees is suspected of having septic arthritis. Information obtained during the nursing history that indicates a risk factor for septic arthritis is that the patient a. had several knee injuries as a teenager. b. recently returned from South America. c. is sexually active with multiple partners. d. has a parent who has rheumatoid arthritis.

ANS: C Neisseria gonorrhoeae is the most common cause for septic arthritis in sexually active young adults. The other information does not point to any risk for septic arthritis.

Which patient seen by the nurse in the outpatient clinic is most likely to require teaching about ways to reduce risk for osteoarthritis (OA)? a. A 38-year-old man who plays on a summer softball team b. A 56-year-old man who is a member of a construction crew c. A 56-year-old woman who works on an automotive assembly line d. A 49-year-old woman who is newly diagnosed with diabetes mellitus

ANS: C OA is more likely to occur in women as a result of estrogen reduction at menopause and in individuals whose work involves repetitive movements and lifting. Moderate exercise, such as softball, reduces risk for OA. Diabetes is not a risk factor for OA. Working on a construction crew would involve nonrepetitive work and thus would not be as risky.

A patient will attempt oral feedings for the first time after having a stroke. The nurse should assess the gag reflex and then a. order a varied pureed diet. c. assist the patient into a chair. b. assess the patient's appetite. d. offer the patient a sip of juice.

ANS: C The patient should be as upright as possible before attempting feeding to make swallowing easier and decrease aspiration risk. To assess swallowing ability, the nurse should initially offer water or ice to the patient. Pureed diets are not recommended because the texture is too smooth. The patient may have a poor appetite, but the oral feeding should be attempted. DIF: Cognitive Level: Apply (application) REF: 1360 TOP: Nursing Process: Implementation MSC: NCLEX: Physiological Integrity

1. Which action will the nurse in the hypertension clinic take in order to obtain an accurate baseline blood pressure (BP) for a new patient? a. Obtain a BP reading in each arm and average the results. b. Deflate the BP cuff at a rate of 5 to 10 mm Hg per second. c. Have the patient sit in a chair with the feet flat on the floor. d. Assist the patient to the supine position for BP measurements.

ANS: C The patient should be seated with the feet flat on the floor. The BP is obtained in both arms, but the results of the two arms are not averaged. The patient does not need to be in the supine position. The cuff should be deflated at 2 to 3 mm Hg per second.

A patient has an incomplete left spinal cord lesion at the level of T7, resulting in Brown-Séquard syndrome. Which nursing action should be included in the plan of care? a. Assessment of the patient for right arm weakness b. Assessment of the patient for increased right leg pain c. Positioning the patient's left leg when turning the patient d. Teaching the patient to look at the right leg to verify its position

ANS: C The patient with Brown-Séquard syndrome has loss of motor function on the ipsilateral side and will require the nurse to move the left leg. Pain sensation will be lost in the patient's right leg. Arm weakness will not be a problem for a patient with a T7 injury. The patient will retain position sense for the right leg. DIF: Cognitive Level: Apply (application)

A patient with left-sided weakness that started 60 minutes earlier is admitted to the emergency department and diagnostic tests are ordered. Which test should be done first? a. Complete blood count (CBC) b. Chest radiograph (chest x-ray) c. Computed tomography (CT) scan d. 12-Lead electrocardiogram (ECG)

ANS: C Rapid screening with a noncontrast CT scan is needed before administration of tissue plasminogen activator (tPA), which must be given within 4.5 hours of the onset of clinical manifestations of the stroke. The sooner the tPA is given, the less brain injury. The other diagnostic tests give information about possible causes of the stroke and do not need to be completed as urgently as the CT scan. DIF: Cognitive Level: Analyze (analysis) REF: 1354 OBJ: Special Questions: Prioritization TOP: Nursing Process: Implementation MSC: NCLEX: Physiological Integrity

A 29-year-old patient reporting painful urination and knee pain is diagnosed with reactive arthritis. The nurse will plan to teach the patient about the need for several months of therapy with a. anakinra (Kineret). b. etanercept (Enbrel). c. doxycycline (Vibramycin). d. methotrexate (Rheumatrex).

ANS: C Reactive arthritis associated with urethritis is usually caused by infection with Chlamydia trachomatis and requires 3 months of treatment with doxycycline. The other medications are used for chronic inflammatory problems such as rheumatoid arthritis.

A patient with rheumatoid arthritis being seen in the clinic has rheumatoid nodules on the elbows. Which action will the nurse take? a. Draw blood for rheumatoid factor analysis. b. Teach the patient about injections for the nodules. c. Assess the nodules for skin breakdown or infection. d. Discuss the need for surgical removal of the nodules.

ANS: C Rheumatoid nodules can break down or become infected. They are not associated with changes in rheumatoid factor, and injection is not needed. Rheumatoid nodules are usually not removed surgically because of a high probability of recurrence.

16. A patient with a history of hypertension treated with a diuretic and an angiotensin-converting enzyme (ACE) inhibitor arrives in the emergency department complaining of a severe headache and has a BP of 240/118 mm Hg. Which question should the nurse ask first? a. Did you take any acetaminophen (Tylenol) today? b. Do you have any recent stressful events in your life? c. Have you been consistently taking your medications? d. Have you recently taken any antihistamine medications?

ANS: C Sudden withdrawal of antihypertensive medications can cause rebound hypertension and hypertensive crisis. Although many over-the-counter medications can cause hypertension, antihistamines and acetaminophen do not increase BP. Stressful events will increase BP but not usually to the level seen in this patient.

20. The charge nurse observes a new RN doing discharge teaching for a hypertensive patient who has a new prescription for enalapril (Vasotec). The charge nurse will need to intervene if the new RN tells the patient to a. check the BP with a home BP monitor every day. b. move slowly when moving from lying to standing. c. increase the dietary intake of high-potassium foods. d. make an appointment with the dietitian for teaching.

ANS: C The ACE inhibitors cause retention of potassium by the kidney, so hyperkalemia is a possible adverse effect. The other teaching by the new RN is appropriate for a patient with newly diagnosed hypertension who has just started therapy with enalapril.

A patient admitted with dermal ulcers who has a history of a T3 spinal cord injury tells the nurse, "I have a pounding headache and I feel sick to my stomach." Which action should the nurse take first? a. Check for a fecal impaction. c. Assess the blood pressure (BP). b. Give the prescribed antiemetic. d. Notify the health care provider.

ANS: C The BP should be assessed immediately in a patient with an injury at the T6 level or higher who complains of a headache to determine if autonomic hyperreflexia is occurring. Notification of the patient's health care provider is appropriate after the BP is obtained. Administration of an antiemetic is indicated if autonomic hyperreflexia is ruled out as the cause of the nausea. After checking the BP, the nurse may assess for a fecal impaction using lidocaine jelly to prevent further increased BP. DIF: Cognitive Level: Analyze (analysis)

A new clinic patient with joint swelling and pain is being tested for systemic lupus erythematosus. Which test will provide the most specific findings for the nurse to review? a. Rheumatoid factor (RF) b. Antinuclear antibody (ANA) c. Anti-Smith antibody (Anti-Sm) d. Lupus erythematosus (LE) cell prep

ANS: C The anti-Sm is antibody found almost exclusively in SLE. The other blood tests are also used in screening but are not as specific to SLE

A 38-yr-old patient who has had a spinal cord injury returned home following a stay in a rehabilitation facility. The home care nurse notes the spouse is performing many of the activities that the patient had been managing unassisted during rehabilitation. The appropriate nursing action at this phase of rehabilitation is to a. remind the patient about the importance of independence in daily activities. b. tell the spouse to stop helping because the patient is able to perform activities independently. c. develop a plan to increase the patient's independence in consultation with the patient and the spouse. d. recognize that it is important for the spouse to be involved in the patient's care and encourage participation.

ANS: C The best action by the nurse will be to involve all parties in developing an optimal plan of care. Because family members who will be assisting with the patient's ongoing care need to believe their input is important, telling the spouse that the patient can perform activities independently is not the best choice. Reminding the patient about the importance of independence may not change the behaviors of the spouse. Supporting the activities of the spouse will lead to ongoing dependency by the patient. DIF: Cognitive Level: Apply (application)

During the change of shift report, a nurse is told that a patient has an occluded left posterior cerebral artery. The nurse will anticipate that the patient may have a. dysphasia. c. visual deficits. b. confusion. d. poor judgment.

ANS: C Visual disturbances are expected with posterior cerebral artery occlusion. Aphasia occurs with middle cerebral artery involvement. Cognitive deficits and changes in judgment are more typical of anterior cerebral artery occlusion. DIF: Cognitive Level: Apply (application) REF: 1350 TOP: Nursing Process: Assessment MSC: NCLEX: Physiological Integrity

A patient has muscle spasms and acute low back pain. An appropriate nursing intervention for this problem is to teach the patient to a. avoid the use of cold because it will exacerbate the muscle spasms. b. keep both feet flat on the floor when prolonged standing is required. c. keep the head elevated slightly and flex the knees when resting in bed. d. twist gently from side to side to maintain range of motion in the spine.

ANS: C Resting with the head elevated and knees flexed will reduce the strain on the back and decrease muscle spasms. Twisting from side to side will increase tension on the lumbar area. A pillow placed under the upper back will cause strain on the lumbar spine. Alternate application of cold and heat should be used to decrease pain.

After teaching a patient with a bunion about how to prevent further problems, the nurse will determine that more teaching is needed if the patient says, a. "I will throw away my high heel shoes." b. "I will use the bunion pad to relieve the pain." c. "I will need to wear open sandals at all times." d. "I will take ibuprofen (Motrin) when I need it."

ANS: C The patient can wear shoes that have a wide forefoot. The other patient statements indicate that the teaching has been effective.

A patient with a herniated intravertebral disk undergoes a laminectomy and discectomy. Following the surgery, the nurse should position the patient on the side by a. instructing the patient to move the legs before turning the rest of the body. b. having the patient turn by grasping the side rails and pulling the shoulders over. c. placing a pillow between the patient's legs and turning the entire body as a unit. d. turning the patient's head and shoulders first, followed by the hips, legs, and feet.

ANS: C The spine should be kept in correct alignment after laminectomy. The other positions will create misalignment of the spine.

When administering alendronate (Fosamax) to a patient, the nurse will first a. be sure the patient has recently eaten. b. ask about any leg cramps or hot flashes. c. assist the patient to sit up at the bedside. d. administer the ordered calcium carbonate.

ANS: C To avoid esophageal erosions, the patient taking bisphosphonates should be upright for at least 30 minutes after taking the medication. Fosamax should be taken on an empty stomach, not after taking other medications or eating. Leg cramps and hot flashes are not side effects of bisphosphonates.

The home health nurse is caring for an 81-yr-old who had a stroke 2 months ago. Based on information shown in the accompanying figure from the history, physical assessment, and physical and occupational therapy, which problem is the highest priority? History Well controlled type 2 diabetes for 10 years Married 45 years; spouse has heart failure and chronic obstructive pulmonary disease Physical Assessment Oriented to time, place, person Speech clear Minimal left leg weakness Physical/Occupational Therapy Uses cane with walking Spouse does household cleaning and cooking and assists patient with bathing and dressing a. Risk for hypoglycemia c. Risk for caregiver role strain b. Impaired transfer ability d. Ineffective health maintenance

ANS: C The spouse's household and patient care responsibilities, in combination with chronic illnesses, indicate a high risk for caregiver role strain. The nurse should further assess the situation and take appropriate actions. The data about the control of the patient's diabetes indicates that ineffective health maintenance and risk for unstable blood glucose are not priority concerns at this time. Because the patient is able to ambulate with a cane, the nursing diagnosis of impaired transfer ability is not supported. DIF: Cognitive Level: Analyze (analysis) REF: 1362 OBJ: Special Questions: Prioritization TOP: Nursing Process: Diagnosis MSC: NCLEX: Psychosocial Integrity

The nurse identifies a patient with type 1 diabetes and a history of herpes simplex infection as being at risk for Bell's palsy. Which information should the nurse include in teaching the patient? a. "You may be able to prevent Bell's palsy by doing facial exercises regularly." b. "Prophylactic treatment of herpes with antiviral agents prevents Bell's palsy." c. "Medications to treat Bell's palsy work only if started before paralysis onset." d. "Call the doctor if you experience pain or develop herpes lesions near the ear."

ANS: D Pain or herpes lesions near the ear may indicate the onset of Bell's palsy, and rapid corticosteroid treatment may reduce the duration of Bell's palsy symptoms. Antiviral therapy for herpes simplex does not reduce the risk for Bell's palsy. Corticosteroid therapy will be most effective in reducing symptoms if started before paralysis is complete but will still be somewhat effective when started later. Facial exercises do not prevent Bell's palsy. DIF: Cognitive Level: Apply (application)

The nurse teaching a support group of women with rheumatoid arthritis (RA) about how to manage activities of daily living suggests that they a. stand rather than sit when performing household and yard chores. b. strengthen small hand muscles by wringing sponges or washcloths. c. protect the knee joints by sleeping with a small pillow under the knees. d. avoid activities that require repetitive use of the same muscles and joints.

ANS: D Patients are advised to avoid repetitious movements. Sitting during household chores is recommended to decrease stress on joints. Wringing water out of sponges would increase the joint stress. Patients are encouraged to position joints in the extended position, and sleeping with a pillow behind the knees would decrease the ability of the knee to extend and also decrease knee range of motion (ROM).

A 54-year-old woman who recently reached menopause and has a family history of osteoporosis is diagnosed with osteopenia following densitometry testing. In teaching the woman about her osteoporosis, the nurse explains that a. estrogen replacement therapy must be started to prevent rapid progression to osteoporosis. b. continuous, low-dose corticosteroid treatment is effective in stopping the course of osteoporosis. c. with a family history of osteoporosis, there is no way to prevent or slow gradual bone resorption. d. calcium loss from bones can be slowed by increasing calcium intake and weight-bearing exercise.

ANS: D Progression of osteoporosis can be slowed by increasing calcium intake and weight-bearing exercise. Estrogen replacement therapy does help prevent osteoporosis, but it is not the only treatment and is not appropriate for some patients. Corticosteroid therapy increases the risk for osteoporosis

Nurses in change-of-shift report are discussing the care of a patient with a stroke who has progressively increasing weakness and decreasing level of consciousness. Which patient problem do they determine has the highest priority for the patient? a. Impaired physical mobility related to weakness b. Disturbed sensory perception related to brain injury c. Risk for impaired skin integrity related to immobility d. Risk for aspiration related to inability to protect airway

ANS: D Protection of the airway is the priority of nursing care for a patient having an acute stroke. The other diagnoses are also appropriate, but interventions to prevent aspiration are the priority at this time. DIF: Cognitive Level: Analyze (analysis) REF: 1354 OBJ: Special Questions: Prioritization TOP: Nursing Process: Analysis MSC: NCLEX: Physiological Integrity

A patient with a stroke experiences facial drooping on the right side and right-sided arm and leg paralysis. When admitting the patient, which clinical manifestation will the nurse expect to find? a. Impulsive behavior b. Right-sided neglect c. Hyperactive left-sided tendon reflexes d. Difficulty comprehending instructions

ANS: D Right-sided paralysis indicates a left-brain stroke, which will lead to difficulty with comprehension and use of language. The left-side reflexes are likely to be intact. Impulsive behavior and neglect are more likely with a right-side stroke. DIF: Cognitive Level: Apply (application) REF: 1350 TOP: Nursing Process: Assessment MSC: NCLEX: Physiological Integrity

18. When a patient with hypertension who has a new prescription for atenolol (Tenormin) returns to the health clinic after 2 weeks for a follow-up visit, the BP is unchanged from the previous visit. Which action should the nurse take first? a. Provide information about the use of multiple drugs to treat hypertension. b. Teach the patient about the reasons for a possible change in drug therapy. c. Remind the patient that lifestyle changes also are important in BP control. d. Question the patient about whether the medication is actually being taken.

ANS: D Since noncompliance with antihypertensive therapy is common, the nurse's initial action should be to determine whether the patient is taking the atenolol as prescribed. The other actions also may be implemented, but these would be done after assessing patient compliance with the prescribed therapy.

7. A 52-year-old patient who has no previous history of hypertension or other health problems suddenly develops a BP of 188/106 mm Hg. After reconfirming the BP, it is appropriate for the nurse to tell the patient that a. a BP recheck should be scheduled in a few weeks. b. the dietary sodium and fat content should be decreased. c. there is an immediate danger of a stroke and hospitalization will be required. d. more diagnostic testing may be needed to determine the cause of the hypertension.

ANS: D A sudden increase in BP in a patient over age 50 with no previous hypertension history or risk factors indicates that the hypertension may be secondary to some other problem. The BP will need rapid treatment and ongoing monitoring. If the patient has no other risk factors, a stroke in the immediate future is unlikely. There is no indication that dietary salt or fat intake have contributed to this sudden increase in BP, and reducing intake of salt and fat alone will not be adequate to reduce this BP to an acceptable level.

Which of these nursing actions for a patient with Guillain-Barré syndrome is appropriate for the nurse to delegate to experienced unlicensed assistive personnel (UAP)? a. Nasogastric tube feeding q4hr b. Artificial tear administration q2hr c. Assessment for bladder distention q2hr d. Passive range of motion to extremities q4hr

ANS: D Assisting a patient with movement is included in UAP education and scope of practice. Administration of tube feedings, administration of ordered medications, and assessment are skills requiring more education and expanded scope of practice, and the RN should perform these skills. DIF: Cognitive Level: Apply (application)

A patient hospitalized with a new diagnosis of Guillain-Barré syndrome has numbness and weakness of both feet. The nurse will anticipate teaching the patient about a. infusion of immunoglobulin b. intubation and mechanical ventilation. c. administration of corticosteroid drugs. d. insertion of a nasogastric (NG) feeding tube.

ANS: D Because Guillain-Barré syndrome is in the earliest stages (as evidenced by the symptoms), use of high-dose immunoglobulin is appropriate to reduce the extent and length of symptoms. Mechanical ventilation and tube feedings may be used later in the progression of the syndrome but are not needed now. Corticosteroid use is not helpful in reducing the duration or symptoms of the syndrome. DIF: Cognitive Level: Apply (application)

4. After the nurse teaches the patient with stage 1 hypertension about diet modifications that should be implemented, which diet choice indicates that the teaching has been effective? a. The patient avoids eating nuts or nut butters. b. The patient restricts intake of dietary protein. c. The patient has only one cup of coffee in the morning. d. The patient has a glass of low-fat milk with each meal.

ANS: D The Dietary Approaches to Stop Hypertension (DASH) recommendations for prevention of hypertension include increasing the intake of calcium-rich foods. Caffeine restriction and decreased protein intake are not included in the recommendations. Nuts are high in beneficial nutrients and 4 to 5 servings weekly are recommended in the DASH diet.

A 71-year-old patient who takes multiple medications develops acute gouty arthritis. The nurse will consult with the health care provider before giving the prescribed dose of a. sertraline (Zoloft). b. famotidine (Pepcid). c. oxycodone (Roxicodone). d. hydrochlorothiazide (HydroDIURIL).

ANS: D Diuretic use increases uric acid levels and can precipitate gout attacks. The other medications are safe to administer

5. A patient has just been diagnosed with hypertension and has a new prescription for captopril (Capoten). Which information is important to include when teaching the patient? a. Check BP daily before taking the medication. b. Increase fluid intake if dryness of the mouth is a problem. c. Include high-potassium foods such as bananas in the diet. d. Change position slowly to help prevent dizziness and falls.

ANS: D The angiotensin-converting enzyme (ACE) inhibitors frequently cause orthostatic hypotension, and pa- tients should be taught to change position slowly to allow the vascular system time to compensate for the position change. Increasing fluid intake may counteract the effect of the medication, and the patient is taught to use gum or hard candy to relieve dry mouth. The BP does not need to be checked at home by the patient before taking the medication. Because ACE inhibitors cause potassium retention, increased intake of high-potassium foods is inappropriate.

The home health nurse is doing a follow-up visit to a 41-year-old patient with recently diagnosed rheumatoid arthritis (RA). Which assessment made by the nurse indicates that more patient teaching is needed? a. The patient takes a 2-hour nap each day. b. The patient has been taking 16 aspirins daily. c. The patient sits on a stool while preparing meals. d. The patient sleeps with two pillows under the head.

ANS: D The joints should be maintained in an extended position to avoid contractures, so patients should use a small, flat pillow for sleeping. The other information is appropriate for a patient with RA and indicates that teaching has been effective

A patient arrives in the emergency department with hemiparesis and dysarthria that started 2 hours previously, and health records show a history of several transient ischemic attacks (TIAs). The nurse anticipates preparing the patient for a. surgical endarterectomy. b. transluminal angioplasty. c. intravenous heparin drip administration. d. tissue plasminogen activator (tPA) infusion.

ANS: D The patient's history and clinical manifestations suggest an acute ischemic stroke, and a patient who is seen within 4.5 hours of stroke onset is likely to receive tPA (after screening with a CT scan). Heparin administration in the emergency phase is not indicated. Emergent carotid transluminal angioplasty or endarterectomy is not indicated for the patient who is having an acute ischemic stroke. DIF: Cognitive Level: Apply (application) REF: 1355 TOP: Nursing Process: Planning MSC: NCLEX: Physiological Integrity

A 25-year-old female patient with systemic lupus erythematosus (SLE) who has a facial rash and alopecia tells the nurse, "I never leave my house because I hate the way I look." An appropriate nursing diagnosis for the patient is a. activity intolerance related to fatigue and inactivity. b. impaired social interaction related to lack of social skills. c. impaired skin integrity related to itching and skin sloughing. d. social isolation related to embarrassment about the effects of SLE.

ANS: D The patient's statement about not going anywhere because of hating the way he or she looks supports the diagnosis of social isolation because of embarrassment about the effects of the SLE. Activity intolerance is a possible problem for patients with SLE, but the information about this patient does not support this as a diagnosis. The rash with SLE is nonpruritic. There is no evidence of lack of social skills for this patient.

A 58-year-old woman who has a family history of osteoporosis is diagnosed with osteopenia following densitometry testing. In teaching the woman about her osteoporosis, the nurse explains that a. estrogen replacement therapy must be started to prevent rapid progression to osteoporosis. b. continuous, low-dose corticosteroid treatment is effective in stopping the course of osteoporosis. c. with a family history of osteoporosis, there is no way to prevent or slow gradual bone resorption. d. calcium loss from bones can be slowed by increasing calcium intake and weight-bearing exercise.

ANS: D Progression of osteoporosis can be slowed by increasing calcium intake and weight-bearing exercise. Estrogen replacement therapy does help prevent osteoporosis, but it is not the only treatment and is not appropriate for some patients. Corticosteroid therapy increases the risk for osteoporosis.

Which of these nursing actions included in the care of a patient after laminectomy can the nurse delegate to experienced nursing assistive personnel (NAP)? a. Ask about pain control with the patient-controlled analgesia (PCA). b. Determine the patient's readiness to ambulate. c. Check ability to plantar and dorsiflex the foot. d. Turn the patient from side to side every 2 hours.

ANS: D Repositioning a patient is included in the education and scope of practice of NAP, and experienced NAP will be familiar with how to maintain alignment in the postoperative patient. Evaluation of the effectiveness of pain medications, assessment of neurologic function, and evaluation of a patient's readiness to ambulate after surgery require higher level nursing education and scope of practice.

Following a laminectomy with a spinal fusion, a patient reports numbness and tingling of the right lower leg. The first action indicated by the nurse is to a. report the patient's complaint to the surgeon. b. check the vital signs for indications of hemorrhage. c. turn the patient to the side to relieve pressure on the right leg. d. check the chart for preoperative neuromuscular assessment data.

ANS: D The postoperative movement and sensation of the extremities should be unchanged (or improved) from the preoperative assessment. If the numbness and tingling are new, this information should be immediately reported to the surgeon. Numbness and tingling are not symptoms associated with hemorrhage at the site. Turning the patient will not relieve the numbness.

The nurse is caring for a patient with hypertension who is scheduled to receive a dose of metoprolol (Lopressor). The nurse should withhold the dose and consult the prescribing physician for which vital sign taken just before administration? A. O2 saturation 93% B. Pulse 48 beats/min C. Respirations 24 breaths/min D.Blood pressure 118/74 mm Hg

B Because metoprolol is a β1-adrenergic blocking agent, it can cause hypotension and bradycardia as adverse effects. The nurse should withhold the dose and consult with the health care provider for parameters regarding pulse rate limits.

6. The patient is diagnosed with vancomycin-resistant enterococci (VRE) infection in a surgical wound. What infection precautions should the nurse use to best prevent transmission of the infection to others? A. Droplet precautions B. Contact precautions C. Airborne precautions D. Standard precautions

B Contact precautions are used to minimize the spread of pathogens that are acquired from direct or indirect contact. Droplet precautions are used with pathogens that are spread through the air at close contact and that affect the respiratory system or mucous membranes (e.g., influenza, pertussis). Airborne precautions are used if the organism can cause infection over long distances when suspended in the air (e.g., tuberculosis, rubeola). Standard precautions are used with all patients and included in the transmission-based precautions above.

3. The nurse has experienced a recent increase in the incidence of hospital care-associated infections (HAIs) on the unit. Which nursing action should be prioritized in the response to this trend? A. Use of gloves during patient contact B. Frequent and thorough hand washing C. Prophylactic, broad-spectrum antibiotics D. Fitting and appropriate use of N95 masks

B Hand washing remains the mainstay of the prevention of HAIs. Gloves, masks, and antibiotics may be appropriate in specific circumstances, but none of these replaces the central role of vigilant, thorough hand washing between patients and when moving from one task to another, even with the same patient.

A patient experienced a 33% TBSA burn 72 hours ago. The nurse observes that the patient's hourly urine output has been steadily increasing over the past 24 hours. How should the nurse best respond to this finding? A) Obtain an order to reduce the rate of the patient's IV fluid infusion. B) Report the patient's early signs of acute kidney injury (AKI). C) Recognize that the patient is experiencing an expected onset of diuresis. D) Administer sodium chloride as ordered to compensate for this fluid loss.

Ans: C Feedback: As capillaries regain integrity, 48 or more hours after the burn, fluid moves from the interstitial to the intravascular compartment and diuresis begins. This is an expected development and does not require a reduction in the IV infusion rate or the administration of NaCl. Diuresis is not suggestive of AKI.

The nurse caring for a patient who is recovering from full-thickness burns is aware of the patient's risk for contracture and hypertrophic scarring. How can the nurse best mitigate this risk? A) Apply skin emollients as ordered after granulation has occurred. B) Keep injured areas immobilized whenever possible to promote healing. C) Administer oral or IV corticosteroids as ordered. D) Encourage physical activity and range of motion exercises.

Ans: D Feedback: Exercise and the promotion of mobility can reduce the risk of contracture and hypertrophic scarring. Skin emollients are not normally used in the treatment of burns, and these do not prevent scarring. Steroids are not used to reduce scarring, as they also slow the healing process.

A nurse is performing a home visit to a patient who is recovering following a long course of inpatient treatment for burn injuries. When performing this home visit, the nurse should do which of the following? A) Assess the patient for signs of electrolyte imbalances. B) Administer fluids as ordered. C) Assess the risk for injury recurrence. D) Assess the patient's psychosocial state.

Ans: D Feedback: Recovery from burns can be psychologically challenging; the nurse's assessments must address this reality. Fluid and electrolyte imbalances are infrequent during the rehabilitation phase of recovery. Burns are not typically a health problem that tends to recur; the experience of being burned tends to foster vigilance.

While performing a patient's ordered wound care for the treatment of a burn, the patient has made a series of sarcastic remarks to the nurse and criticized her technique. How should the nurse best interpret this patient's behavior? A) The patient may be experiencing an adverse drug reaction that is affecting his cognition and behavior. B) The patient may be experiencing neurologic or psychiatric complications of his injuries. C) The patient may be experiencing inconsistencies in the care that he is being provided. D) The patient may be experiencing anger about his circumstances that he is deflecting toward the nurse.

Ans: D Feedback: The patient may experience feelings of anger. The anger may be directed outward toward those who escaped unharmed or toward those who are now providing care. While drug reactions, complications, and frustrating inconsistencies in care cannot be automatically ruled out, it is not uncommon for anger to be directed at caregivers.

The nurse provides dietary instructions to the in-home caregiver of a 45-yr-old man with Huntington's disease. The nurse is most concerned if the caregiver makes which statement? "Depression is common and may cause a decrease in appetite." "If swallowing becomes difficult, a feeding tube may be needed." "Calories should be restricted to prevent unnecessary weight gain." "Muscles in the face are affected, and chewing may become impossible."

"Calories should be restricted to prevent unnecessary weight gain."

The nurse provides information to the caregiver of a 68-yr-old man with epilepsy who has tonic-clonic seizures. Which statement by the caregiver indicates a need for further teaching? "It is normal for a person to be sleepy after a seizure." "I should call 911 if breathing stops during the seizure." "The jerking movements may last for 30 to 40 seconds." "Objects should not be placed in the mouth during a seizure."

"I should call 911 if breathing stops during the seizure." Caregivers do not need to call an ambulance or send a person to the hospital after a single seizure unless the seizure is prolonged, another seizure immediately follows, or extensive injury has occurred. Altered breathing is a clinical manifestation of a tonic-clonic seizure. Contact emergency medical services (or call 911) if breathing stops for more than 30 seconds.

The nurse teaches a 38-yr-old woman who has migraine headaches about sumatriptan (Imitrex). Which statement by the patient requires clarification by the nurse? "The injection might feel like a bee sting." "This medicine will prevent a migraine headache." "I can take another dose if the first does not work." "This drug for migraine headaches could cause birth defects."

"This medicine will prevent a migraine headache."

Which of the following statements should the nurse include in the teaching session when preparing a client for arthrocentesis? Select all that apply. 1. "A local anesthetic agent may be injected into the joint site for your comfort." 2. "A syringe and needle will be used to withdraw fluid from your joint." 3. "The procedure, although not painful, will provide immediate relief." 4. "We'll want you to keep your joint active after the procedure to increase blood flow." 5. "You will need to wear a compression bandage for several days after the procedure."

1, 2, 5. An arthrocentesis is performed to aspirate excess synovial fluid, pus, or blood from a joint cavity to relieve pain or to diagnosis inflammatory diseases such as rheumatoid arthritis. A local agent may be used to decrease the pain of the needle insertion through the skin and into the joint cavity. Aspiration of the fluid into the syringe can be very painful because of the size and inflammation of the joint. Usually a steroid medication is injected locally to alleviate the inflammation; a compression bandage is applied to help decrease swelling; and the client is asked to rest the joint for up to 24 hours afterwards to help relieve the pain and promote rest to the inflamed joint. The client may experience pain during this time until the inflammation begins to resolve and swelling decreases.

A client with osteoarthritis will undergo an arthrocentesis on his painful edematous knee. What should be included in the nursing plan of care? Select all that apply. 1. Explain the procedure. 2. Administer preoperative medication 1 hour before surgery. 3. Instruct the client to immobilize the knee for 2 days after the surgery. 4. Assess the site for bleeding. 5. Offer pain medication.

1, 4, 5. To prepare a client for an arthrocentesis, the nurse should tell the client that a local anesthetic administered by the physician will decrease discomfort. There may be bleeding after the procedure, so the nurse should check the dressing. The client may experience pain. The nurse should offer pain medication and evaluate outcomes for pain relief. Because a local anesthetic is used, the client will not require preoperative medication. The client will rest the knee for 24 hours and then should begin range-of-motion and muscle strengthening exercises.

2. A client with rheumatoid arthritis states, "I can't do my household chores without becoming tired. My knees hurt whenever I walk." Which nursing diagnosis would be most appropriate? 1. Activity intolerance related to fatigue and pain. 2. Self-care deficit related to increasing joint pain. 3. Ineffective coping related to chronic pain. 4. Disturbed body image related to fatigue and joint pain.

1. Based on the client's complaints, the most appropriate nursing diagnosis would be Activity intolerance related to fatigue and pain. Nursing interventions would focus on helping the client conserve energy and decrease episodes of fatigue. Although the client may develop a self-care deficit related to the activity intolerance and increasing joint pain, the client is voicing concerns about household chores and difficulty around the house and yard, not self-care issues. Over time, the client may develop ineffective coping or body image disturbance as the disorder becomes chronic with increasing pain and fatigue.

After teaching the client with severe rheumatoid arthritis about prescribed methotrexate (Rheumatrex), which of the following statements indicates the need for further teaching? 1. "I will take my vitamins while I'm on this drug." 2. "I must not drink any alcohol while I'm taking this drug." 3. "I should brush my teeth after every meal." 4. "I will continue taking my birth control pills."

1. Because some over-the-counter vitamin supplements contain folic acid, the client should avoid self-medication with vitamins while taking methotrexate, a folic acid antagonist. Because methotrexate is hepatotoxic, the client should avoid the intake of alcohol, which could increase the risk for hepatotoxicity. Methotrexate can cause bone marrow depression, placing the client at risk for infection. Therefore, meticulous mouth care is essential to minimize the risk of infection. Contraception should be used during methotrexate therapy and for 8 weeks after the therapy has been discontinued because of its effect on mitosis. Methotrexate is considered teratogenic.

Which of the following statements indicates that the client with osteoarthritis understands the effects of capsaicin (Zostrix) cream? 1. "I always wash my hands right after I apply the cream." 2. "After I apply the cream, I wrap my knee with an elastic bandage." 3. "I keep the cream in the cabinet above the stove in the kitchen." 4. "I also use the same cream when I get a cut or a burn."

1. Capsaicin cream, which produces analgesia by preventing the reaccumulation of substance P in the peripheral sensory neurons, is made from the active ingredients of hot peppers. Therefore, clients should wash their hands immediately after applying capsaicin cream if they do not wear gloves, to avoid possible contact between the cream and mucous membranes. Clients are instructed to avoid wearing tight bandages over areas where capsaicin cream has been applied because swelling may occur from inflammation of the arthritis in the joint and lead to constriction on the peripheral neurovascular system. Capsaicin cream should be stored in areas between 59 ° F and 86 ° F (15 ° C and 30 ° C). The cabinet over the stove in the kitchen would be too warm. Capsaicin cream should not come in contact with irritated and broken skin, mucous membranes, or eyes. Therefore it should not be used on cuts or burns.

The client diagnosed with osteoarthritis states, "My friend takes steroid pills for her rheumatoid arthritis. Why don't I take steroids for my osteoarthritis?" Which of the following is the best explanation? 1. Intra-articular corticosteroid injections are used to treat osteoarthritis. 2. Oral corticosteroids can be used in osteoarthritis. 3. A systemic effect is needed in osteoarthritis. 4. Rheumatoid arthritis and osteoarthritis are two similar diseases.

1. Corticosteroids are used for clients with osteoarthritis to obtain a local effect. Therefore, they are given only via intra-articular injection. Oral corticosteroids are avoided because they can cause an acceleration of osteoarthritis. Rheumatoid arthritis and osteoarthritis are two different diseases.

10. Which statement(s) about metabolic side effects of ART is (are) true (select all that apply)? a. These are annoying symptoms that are ultimately harmless. b. ART-related body changes include fat redistribution and peripheral wasting. c. Lipid abnormalities include increases in triglycerides and decreases in high-density cholesterol. d. Insulin resistance and hyperlipidemia can be treated with drugs to control glucose and cholesterol. e. Compared to uninfected people, insulin resistance and hyperlipidemia are more difficult to treat in HIV-infected patients.

10. Correct answers: b, c, d Rationale: Some HIV-infected patients, especially those who have been infected and have received ART for a long time, develop a set of metabolic disorders that include changes in body shape (e.g., fat deposits in the abdomen, upper back, and breasts along with fat loss in the arms, legs, and face) as a result of lipodystrophy, hyperlipidemia (i.e., elevated triglyceride levels and decreases in high-density lipoprotein levels), insulin resistance and hyperglycemia, bone disease (e.g., osteoporosis, osteopenia, avascular necrosis), lactic acidosis, and cardiovascular disease.

10. What should the nurse teach the patient recovering from an episode of acute low back pain? a. Perform daily exercise as a lifelong routine. b. Sit in a chair with the hips higher than the knees. c. Avoid occupations in which the use of the body is required. d. Sleep on the abdomen or on the back with the legs extended.

10. a. Proper daily exercise is an important part of the prevention of back injury, with the goal of maintaining mobility and strength in the back. Patients should sit with the knees higher than the hips and should sleep in a side-lying position, with knees and hips bent, or on the back, with a device to flex the hips and knees. Good body mechanics with proper transfer and turning techniques are necessary in all jobs and activities.

11. Which strategy can the nurse teach the patient to eliminate the risk of HIV transmission? a. Using sterile equipment to inject drugs b. Cleaning equipment used to inject drugs c. Taking lamivudine (Epivir) during pregnancy d. Using latex or polyurethane barriers to cover genitalia during sexual contact

11. Correct answer: a Rationale: Access to sterile equipment is an important risk-elimination tactic. Some communities have needle and syringe exchange programs (NSEPs) that provide sterile equipment to users in exchange for used equipment. Cleaning equipment before use is a risk-reducing activity. It decreases the risk when equipment is shared, but it takes time, and a person in drug withdrawal may have difficulty cleaning equipment. Lamivudine alone is not appropriate for treatment in pregnancy. Barrier methods reduce but do not eliminate risk.

11. A laminectomy and spinal fusion are performed on a patient with a herniated lumbar intervertebral disc. During the postoperative period, which finding is of most concern to the nurse? a. Paralytic ileus b. Urinary incontinence c. Greater pain at the graft site than at the lumbar incision site d. Leg and arm movement and sensation unchanged from preoperative status

11. b. Urinary incontinence following spinal surgery may indicate nerve damage and should be reported to the health care provider. Paralytic ileus is common following surgery and is expected. Pain at the graft site, usually the iliac crest or the fibula, often is more severe than pain at the fused area. Although movement and sensation of the arms and legs should not be more impaired than before surgery, they often are not relieved immediately after surgery.

12. What is the most appropriate nursing intervention to help an HIV-infected patient adhere to a treatment regimen? a. "Set up" a drug pillbox for the patient every week. b. Give the patient a video and a brochure to view and read at home. c. Tell the patient that the side effects of the drugs are bad but that they go away after a while. d. Assess the patient's routines and find adherence cues that fit into the patient's life circumstances.

12. Correct answer: d Rationale: The best approach to improve adherence to a treatment regimen is to learn about the patient's life and assist with problem solving within the confines of that life.

12. Priority Decision: Before repositioning the patient on the side after a lumbar laminectomy, what should be the nurse's first action? a. Raise the head of the bed 30 degrees. b. Have the patient flex the knees and hips. c. Place a pillow between the patient's legs. d. Have the patient grasp the side rail on the opposite side of the bed.

12. c. After spinal surgery, patients are logrolled to maintain straight alignment of the spine at all times, requiring the patient to be turned with a pillow between the legs and moving the body as a unit. The head of the bed is usually kept flat and the legs are extended

16. What are characteristics of Paget's disease (select all that apply)? a. Results from vitamin D deficiency b. Loss of total bone mass and substance c. Abnormal remodeling and resorption of bone d. Most common in bones of spine, hips, and wrists e. Generalized bone decalcification with bone deformity f. Replacement of normal marrow with vascular connective tissue

16. c, f. Paget's disease is abnormal remodeling and resorption of bone with replacement of normal marrow with vascular connective tissue. Osteoporosis is loss of total bone mass and substance with abnormal remodeling and resorption of bone and is most common in bones of the spine, hips, and wrists. Osteomalacia results from vitamin D deficiency and causes generalized bone decalcification with bone deformity.

Which female patients are at risk for developing osteoporosis (select all that apply)? a. 60-year-old white aerobics instructor b. 55-year-old Asian American cigarette smoker c. 62-year-old African American on estrogen therapy d. 68-year-old white who is underweight and inactive e. 58-year-old Native American who started menopause prematurely

17. b, d, e. Risk factors for osteoporosis include age greater than 65, white or Asian ethnicity, cigarette smoking, inactive lifestyle, low body weight, and postmenopausal estrogen deficiency including premature menopause. Other factors include family history, excessive alcohol use, long-term use of medications such as corticosteroids, thyroid replacement, heparin, long-acting sedatives, or antiseizure drugs.

The client with rheumatoid arthritis tells the nurse, "I have a friend who took gold shots and had a wonderful response. Why didn't my physician let me try that?" Which of the following responses by the nurse would be most appropriate? 1. "It's the physician's prerogative to decide how to treat you. The physician has chosen what is best for your situation." 2. "Tell me more about your friend's arthritic condition. Maybe I can answer that question for you." 3. "That drug is used for cases that are worse than yours. It wouldn't help you, so don't worry about it." 4. "Every person is different. What works for one client may not always be effective for another."

4. The nurse's most appropriate response is one that is therapeutic. The basic principle of therapeutic communication and a therapeutic relationship is honesty. Therefore, the nurse needs to explain truthfully that each client is different and that there are various forms of arthritis and arthritis treatment. To state that it is the physician's prerogative to decide how to treat the client implies that the client is not a member of his or her own health care team and is not a participant in his or her care. The statement also is defensive, which serves to block any further communication or questions from the client about the physician. Asking the client to tell more about the friend presumes that the client knows correct and complete information, which is not a valid assumption to make. The nurse does not know about the client's friend and should not make statements about another client's condition. Stating that the drug is for cases that are worse than the client's demonstrates that the nurse is making assumptions that are not necessarily valid or appropriate. Also, telling the client not to worry ignores the underlying emotions associated with the question, totally discounting the client's feelings.

4. During HIV infection a. reverse transcriptase helps HIV fuse with the CD4+ T cell. b. HIV RNA uses the CD4+ T cell's mitochondria to replicate. c. the immune system is impaired predominantly by the eventual widespread destruction of CD4+ T cells. d. a long period of dormancy develops during which HIV cannot be found in the blood and there is little viral replication.

4. Correct answer: c Immune dysfunction in HIV disease is caused predominantly by damage to and destruction of CD4+ T cells (i.e., T helper cells or CD4+ T lymphocytes).

4. Following 2 weeks of IV antibiotic therapy, a patient with acute osteomyelitis of the tibia is prepared for discharge from the hospital. The nurse determines that additional instruction is needed when the patient makes which statement? a. "I will need to continue antibiotic therapy for 4 to 6 weeks." b. "I shouldn't bear weight on my affected leg until healing is complete." c. "I can use a heating pad on my lower leg for comfort and to promote healing." d. "I should notify the health care provider if the pain in my leg becomes worse."

4. c. Activities such as exercise or heat application, which increase circulation and serve as stimuli for the spread of infection, should be avoided by patients with acute osteomyelitis. Oral or IV antibiotic therapy is continued at home for 4 to 6 weeks, weight bearing is contraindicated to prevent pathologic fractures, and notification of the health care provider if increased pain occurs is necessary.

5. Which statements accurately describe HIV infection (select all that apply)? a. Untreated HIV infection has a predictable pattern of progression. b. Late chronic HIV infection is called acquired immunodeficiency syndrome (AIDS). c. Untreated HIV infection can remain in the early chronic stage for a decade or more. d. Untreated HIV infection usually remains in the early chronic stage for 1 year or less. e. Opportunistic diseases occur more often when the CD4+ T cell count is high and the viral load is low.

5. Correct answers: a, b, c The typical course of untreated HIV infection follows a predictable pattern. However, treatment can significantly alter this pattern, and disease progression is highly individualized. Late chronic infection is another term for acquired immunodeficiency syndrome (AIDS). The median interval between untreated HIV infection and a diagnosis of AIDS is about 11 years.

6. A diagnosis of AIDS is made when an HIV-infected patient has a. a CD4+ T cell count below 200/µL. b. a high level of HIV in the blood and saliva. c. lipodystrophy with metabolic abnormalities. d. oral hairy leukoplakia, an infection caused by Epstein-Barr virus.

6. Correct answer: a AIDS is diagnosed when an individual with HIV infection meets one of several criteria; one criterion is a CD4+ T cell count below 200 cells/L. Other criteria are listed in Table 14-10.

7. Which statement describes osteosarcoma? a High rate of local recurrence b. Very malignant and metastasizes early c. Arises in cancellous ends of long bones d. Develops in the medullary cavity of long bones

7. b. Osteosarcoma, the most common primary bone cancer, occurs in the metaphyseal region of long bones of the arms, legs, or pelvis. It is extremely malignant and metastasizes early and is often brought to attention by injury. A high rate of local recurrence occurs with osteoclastoma that arises in cancellous ends of long bones. Ewing's sarcoma develops in the medullary cavity of long bones as above.

8. HIV antiretroviral drugs are used to a. cure acute HIV infection. b. decrease viral RNA levels. c. treat opportunistic diseases. d. decrease pain and symptoms in terminal disease.

8. Correct answer: b Rationale: The goals of drug therapy in HIV infection are to (1) decrease the viral load, (2) maintain or raise CD4+ T cell counts, and (3) delay onset of HIV infection-related symptoms and opportunistic diseases.

9. Opportunistic diseases in HIV infection a. are usually benign. b. are generally slow to develop and progress. c. occur in the presence of immunosuppression. d. are curable with appropriate drug interventions.

9. Correct answer: c Rationale: Management of HIV infection is complicated by the many opportunistic diseases that can develop as the immune system deteriorates (Table 14-10).

Psychosocial Integrity 26. A patient who has recently started taking pravastatin (Pravachol) and niacin (Nicobid) reports the following symptoms to the nurse. Which is most important to communicate to the health care provider? a. Generalized muscle aches and pains b. Dizziness when changing positions quickly c. Nausea when taking the drugs before eating d. Flushing and pruritus after taking the medications

A Muscle aches and pains may indicate myopathy and rhabdomyolysis, which have caused acute kidney injury and death in some patients who have taken the statin medications. These symptoms indicate that the pravastatin may need to be discontinued. The other symptoms are common side effects when taking niacin, and although the nurse should follow-up with the health care provider, they do not indicate that a change in medication is needed. DIF: Cognitive Level: Apply (application) REF: 738-739 OBJ: Special Questions: Prioritization TOP: Nursing Process: Evaluation MSC:

When initially teaching a patient the supraglottic swallow after a radical neck dissection, with which food or fluid should the nurse begin? A. Cola B. Applesauce C. French fries D. White grape juice

A When learning the supraglottic swallow, it may be helpful to start with carbonated beverages because the effervescence provides clues about the liquid's position. Thin, watery fluids should be avoided because they are difficult to swallow and increase the risk of aspiration. Nonpourable pureed foods, such as applesauce, decrease the risk of aspiration.

Physiological Integrity 21. A few days after experiencing a myocardial infarction (MI) and successful percutaneous coronary intervention, the patient states, "I just had a little chest pain. As soon as I get out of here, I'm going for my vacation as planned." Which reply would be most appropriate for the nurse to make? a. "What do you think caused your chest pain?" b. "Where are you planning to go for your vacation?" c. "Sometimes plans need to change after a heart attack." d. "Recovery from a heart attack takes at least a few weeks."

A When the patient is experiencing denial, the nurse should assist the patient in testing reality until the patient has progressed beyond this step of the emotional adjustment to MI. Asking the patient about vacation plans reinforces the patient's plan, which is not appropriate in the immediate post-MI period. Reminding the patient in denial about the MI is likely to make the patient angry and lead to distrust of the nursing staff. DIF: Cognitive Level: Apply (application) REF: 757 TOP: Nursing Process: Implementation MSC:

A

A 45-yr-old man with asthma is brought to the emergency department by automobile. He is short of breath and appears frightened. During the initial nursing assessment, which clinical manifestation might be present as an early manifestation during an exacerbation of asthma? Anxiety Cyanosis Bradycardia Hypercapnia

The nurse is caring for four newly diagnosed patients with various connective tissue disorders. The nurse should be most aware of safety issues and interstitial lung involvement in the patient with which diagnosis? A) Polymyositis B) Reactive arthritis C) Sjögren's syndrome D) Systemic lupus erythematosus (SLE)

A) Polymyositis is an inflammatory disease affecting striated muscle and resulting in muscle weakness that increases the patient's risk of falls and injury. Weakened pharyngeal muscles increase the risk for aspiration with interstitial lung disease in up to 65% of patients. The treatment of polymyositis starts with high-dose corticosteroids that cause immunosuppression. If this does not work, other immunosuppressive drugs may be used. Reactive arthritis (Reiter's syndrome) occurs with urethritis, conjunctivitis, and mucocutaneous lesions with the asymmetric arthritis involving large joints of the lower extremities and toes. This patient is not at increased risk for safety problems. Sjögren's syndrome decreases moisture produced by exocrine glands, especially in the mouth and eyes and is without increased risk of injury or interstitial lung involvement. Systemic lupus erythematosus (SLE) is a multisystem inflammatory autoimmune disorder treated with NSAIDs, antimalarial agents. Safety would not be an important issue early in the disease.

A pt with a history of peptic ulcer disease has presented to the emergency department with complaints of severe abdominal pain and a rigid, boardlike abdomen, prompting the health care team to suspect a perforated ulcer. Which of the following actions should the nurse anticipate? A) Providing IV fluids and inserting a nasogastric tube B) Administering oral bicarbonate and testing the patient's gastric pH level C) Performing a fecal occult blood test and administering IV calcium gluconate D) Starting parenteral nutrition and placing the patient in a high-Fowler's position

A) Providing IV fluids and inserting a nasogastric tube A perforated peptic ulcer requires IV replacement of fluid losses and continued gastric aspiration by NG tube. Nothing is given by mouth and gastric pH testing is not a priority. Calcium gluconate is not a medication directly relevant to the patient's suspected diagnosis and parenteral nutrition is not a priority in the short term.

The nurse is preparing to insert a nasogastric (NG) tube into a 68-year-old female patient who is nauseated and vomiting. She has an abdominal mass and suspected small intestinal obstruction. The patient asks the nurse why this procedure is necessary. What response by the nurse is most appropriate? A) "The tube will help to drain the stomach contents and prevent further vomiting." B) "The tube will push past the area that is blocked and thus help to stop the vomiting." C) "The tube is just a standard procedure before many types of surgery to the abdomen." D) "The tube will let us measure your stomach contents so that we can plan what type of IV fluid replacement would be best."

A) The NG tube is used to decompress the stomach by draining stomach contents and thereby prevent further vomiting. The NG tube will not push past the blocked area. Potential surgery is not currently indicated. The location of the obstruction will determine the type of fluid to use, not measure the amount of stomach contents.

When evaluating the patient's understanding about the care of the ileostomy, what statement by the patient indicates the patient needs more teaching? A) "I will be able to regulate when I have stools." B) "I will be able to wear the pouch until it leaks." C) "Dried fruit and popcorn must be chewed very well." D) "The drainage from my stoma can damage my skin."

A) The ileostomy is in the ileum and drains liquid stool frequently, unlike the colostomy which has more formed stool the further distal the ostomy is in the colon. The ileostomy pouch is usually worn 4-7 days or until it leaks. It must be changed immediately if it leaks because the drainage is very irritating to the skin. To avoid obstruction, popcorn, dried fruit, coconut, mushrooms, olives, stringy vegetables, food with skin, and meats with casings must be chewed extremely well before swallowing because of the narrow diameter of the ileostomy lumen.

A patient with type 2 diabetes and cirrhosis asks the nurse if it would be okay to take silymarin (milk thistle) to help minimize liver damage. The nurse responds based on knowledge that: A. Milk thistle may affect liver enzymes and thus alter drug metabolism. B. Milk thistle is generally safe in recommended doses for up to 10 years. C. is unclear scientific evidence for the use of milk thistle in treating cirrhosis. D. Milk thistle may elevate the serum glucose levels and is thus contraindicated in diabetes.

A) There is good scientific evidence that there is no real benefit from using milk thistle to protect the liver cells from toxic damage in the treatment of cirrhosis. Milk thistle does affect liver enzymes and thus could alter drug metabolism. Therefore patients will need to be monitored for drug interactions. It is noted to be safe for up to 6 years, not 10 years, and it may lower, not elevate, blood glucose levels.

When administered a dose of metoclopramide (Reglan), a patient complains of nausea. The nurse would teach the patient to report which of the following potential adverse effects? A) Tremors B) Constipation C) Double vision D) Numbness in the fingers and dose

A) Tremors (Extrapyramidal side effects, including tremors and dyskinesias, may occur as a result of metoclopramide (Reglan) administration.)

Two days following a colectomy for an abdominal mass, a patient reports gas pains and abdominal distention. The nurse plans care for the patient based on the knowledge that the symptoms are occurring as a result of A) impaired peristalsis. B) irritation of the bowel. C) nasogastric suctioning. D) inflammation of the incision site.

A) Until peristalsis returns to normal following anesthesia, the patient may experience slowed gastrointestinal motility leading to gas pains and abdominal distention. Irritation of the bowel, nasogastric suctioning, and inflammation of the surgical site do not cause gas pains or abdominal distention.

A 90-year-old healthy man is suffering from dysphagia. The nurse explains what age-related change of the GI tract is the most likely cause of his difficulty? A) Xerostomia B) Esophageal cancer C) Decreased taste buds D) Thinner abdominal wall

A) Xerostomia, decreased volume of saliva, leads to dry oral mucosa and dysphagia. Esophageal cancer is not an age-related change. Decreased taste buds and a thinner abdominal wall do not contribute to difficulty swallowing.

When caring for a patient with liver disease, the nurse recognizes the need to prevent bleeding resulting from altered clotting factors and rupture of varices. Which of the following nursing interventions would be appropriate to achieve this outcome (select all that apply)? A. Use smallest gauge possible when giving injections or drawing blood. B. Teach patient to avoid straining at stool, vigorous blowing of nose, and coughing. C. Advise patient to use soft-bristle toothbrush and avoid ingestion of irritating food. D. Apply gentle pressure for the shortest possible time period after performing venipuncture. E. Instruct patient to avoid aspirin and NSAIDs to prevent hemorrhage when varices are present.

A,B,C,E) Using the smallest gauge needle for injections will minimize the risk of bleeding into the tissues. Avoiding straining, nose blowing, and coughing will reduce the risk of hemorrhage at these sites. The use of a soft-bristle toothbrush and avoidance of irritating food will reduce injury to highly vascular mucous membranes. The nurse should apply gentle but prolonged pressure to venipuncture sites to minimize the risk of bleeding. Aspirin and NSAIDs should not be used in patients with liver disease because they interfere with platelet aggregation, thus increasing the risk for bleeding.

The patient with cirrhosis has an increased abdominal girth from ascites. The nurse should know that this fluid gathers in the abdomen for which reasons (select all that apply)? A) There is decreased colloid oncotic pressure from the liver's inability to synthesize albumin. B) Hyperaldosteronism related to damaged hepatocytes increases sodium and fluid retention. C) Portal hypertension pushes proteins from the blood vessels, causing leaking into the peritoneal cavity. D) Osmoreceptors in the hypothalamus stimulate thirst, which causes the stimulation to take in fluids orally. E) Overactivity of the enlarged spleen results in increased removal of blood cells from the circulation, which decreases the vascular pressure.

A,B.C) The ascites related to cirrhosis are caused by decreased colloid oncotic pressure from the lack of albumin from liver inability to synthesize it and the portal hypertension that shifts the protein from the blood vessels to the peritoneal cavity, and hyperaldosteronism which increases sodium and fluid retention. The intake of fluids orally and the removal of blood cells by the spleen do not directly contribute to ascites.

The nurse teaches a patient with hypertension that uncontrolled hypertension may damage organs in the body primarily by which mechanism? A. Hypertension promotes atherosclerosis and damage to the walls of the arteries. B. Hypertension causes direct pressure on organs, resulting in necrosis and replacement of cells with scar tissue. C. Hypertension causes thickening of the capillary membranes, leading to hypoxia of organ systems. D. Hypertension increases blood viscosity, which contributes to intravascular coagulation and tissue necrosis distal to occlusions.

A. Hypertension is a major risk factor for the development of atherosclerosis by mechanisms not yet fully known. However, when atherosclerosis develops, it damages the walls of arteries and reduces circulation to target organs and tissues.

The nurse provides instructions to a 30-year-old female office worker who has low back pain. Which statement by the patient requires an intervention by the nurse? A. "Acupuncture to the lower back would cause irreparable nerve damage." B. "Smoking may aggravate back pain by decreasing blood flow to the spine." C. "Sleeping on my side with knees and hips bent reduces stress on my back." D. "Switching between hot and cold packs provides relief of pain and stiffness."

A. "Acupuncture to the lower back would cause irreparable nerve damage." Acupuncture is a safe therapy when the practitioner has been appropriately trained. Very fine needles are inserted into the skin to stimulate specific anatomic points in the body for therapeutic purposes.

A patient has just been admitted with a 40% total body surface area (TBSA) burn injury. To maintain adequate nutrition, the nurse should plan to take which action? a. Insert a feeding tube and initiate enteral feedings. b. Infuse total parenteral nutrition via a central catheter. c. Encourage an oral intake of at least 5000 kcal per day. d. Administer multiple vitamins and minerals in the IV solution.

ANS: A Enteral feedings can usually be initiated during the emergent phase at low rates and increased over 24 to 48 hours to the goal rate. During the emergent phase, the patient will be unable to eat enough calories to meet nutritional needs and may have a paralytic ileus that prevents adequate nutrient absorption. Vitamins and minerals may be administered during the emergent phase, but these will not assist in meeting the patient's caloric needs. Parenteral nutrition increases the infection risk, does not help preserve gastrointestinal function, and is not routinely used in burn patients.

12. Which BP finding by the nurse indicates that no changes in therapy are needed for a patient with stage 1 hypertension who has a history of heart failure? a. 108/64 mm Hg b. 128/76 mm Hg c. 140/90 mm Hg d. 136/ 82 mm Hg

ANS: B The goal for antihypertensive therapy for a patient with hypertension and heart failure is a BP of <130/80 mm Hg. The BP of 108/64 may indicate overtreatment of the hypertension and an increased risk for adverse drug effects. The other two blood pressures indicate a need for modifications in the patient's treatment.

A 20-yr-old patient who sustained a T2 spinal cord injury 10 days ago tells the nurse, "I want to be transferred to a hospital where the nurses know what they are doing." Which action by the nurse is appropriate? a. Respond that abusive language will not be tolerated. b. Request that the patient provide input for the plan of care. c. Perform care without responding to the patient's comments. d. Reassure the patient about the competence of the nursing staff.

ANS: B The patient is demonstrating behaviors consistent with the anger phase of the grief process, and the nurse should allow expression of anger and seek the patient's input into care. Expression of anger is appropriate at this stage, and should be accepted by the nurse. Reassurance about the competency of the staff will not be helpful in responding to the patient's concerns. Ignoring the patient's comments will increase the patient's anger and sense of helplessness. DIF: Cognitive Level: Apply (application)

8. Which action will be included in the plan of care when the nurse is caring for a patient who is receiving sodium nitroprusside (Nipride) to treat a hypertensive emergency? a. Organize nursing activities so that the patient has undisturbed sleep for 6 to 8 hours at night. b. Assist the patient up in the chair for meals to avoid complications associated with immobility. c. Use an automated noninvasive blood pressure machine to obtain frequent BP measurements. d. Place the patient on NPO status to prevent aspiration caused by nausea and the associated vomiting.

ANS: C Frequent monitoring of BP is needed when the patient is receiving rapid-acting IV antihypertensive medications. This can be most easily accomplished with an automated BP machine or arterial line. The patient will require frequent assessments, so allowing 6 to 8 hours of undisturbed sleep is not appropriate. When patients are receiving IV vasodilators, bed rest is maintained to prevent decreased cerebral perfusion and fainting. There is no indication that this patient is nauseated or at risk for aspiration, so an NPO status is unnecessary.

11. A patient has been diagnosed with possible white coat hypertension. Which action will the nurse plan to take next? a. Schedule the patient for frequent BP checks in the clinic. b. Instruct the patient about the need to decrease stress levels. c. Tell the patient how to self-monitor and record BPs at home. d. Teach the patient about ambulatory blood pressure monitoring.

ANS: C Having the patient self-monitor BPs at home will provide a reliable indication about whether the patient has hypertension. Frequent BP checks in the clinic are likely to be high in a patient with white coat hypertension. Ambulatory blood pressure monitoring may be used if the data from self-monitoring is unclear. Although elevated stress levels may contribute to hypertension, instructing the patient about this is unlikely to reduce BP.

Which stroke risk factor for a 48-yr-old male patient in the clinic is most important for the nurse to address? a. The patient is 25 lb above the ideal weight. b. The patient drinks a glass of red wine with dinner daily. c. The patient's usual blood pressure (BP) is 170/94 mm Hg. d. The patient works at a desk and relaxes by watching television.

ANS: C Hypertension is the single most important modifiable risk factor. People who drink more than 1 (for women) or 2 (for men) alcoholic beverages a day may increase their risk for hypertension. Physical inactivity and obesity contribute to stroke risk but not as much as hypertension. DIF: Cognitive Level: Analyze (analysis) REF: 1347 OBJ: Special Questions: Prioritization TOP: Nursing Process: Assessment MSC: NCLEX: Health Promotion and Maintenance

13. Which information should the nurse include when teaching a patient with newly diagnosed hypertension? a. Dietary sodium restriction will control BP for most patients. b. Most patients are able to control BP through lifestyle changes. c. Hypertension is usually asymptomatic until significant organ damage occurs. d. Annual BP checks are needed to monitor treatment effectiveness.

ANS: C Hypertension is usually asymptomatic until target organ damage has occurred. Lifestyle changes and sodium restriction are used to help manage blood pressure, but drugs are needed for most patients. BP should be checked by the health care provider every 3 to 6 months.

A male patient who has right-sided weakness after a stroke is making progress in learning to use the left hand for feeding and other activities. The nurse observes that when the patient's wife is visiting, she feeds and dresses him. Which nursing diagnosis is most appropriate for the patient? a. Interrupted family processes related to effects of illness of a family member b. Situational low self-esteem related to increasing dependence on spouse for care c. Disabled family coping related to inadequate understanding by patient's spouse d. Impaired nutrition: less than body requirements related to hemiplegia and aphasia

ANS: C The information supports the diagnosis of disabled family coping because the wife does not understand the rehabilitation program. There are no data supporting low self-esteem, and the patient is attempting independence. The data do not support an interruption in family processes because this may be a typical pattern for the couple. There is no indication that the patient has impaired nutrition. DIF: Cognitive Level: Apply (application) REF: 1362 TOP: Nursing Process: Diagnosis MSC: NCLEX: Psychosocial Integrity

10. During change-of-shift report, the nurse obtains this information about a hypertensive patient who received the first dose of propranolol (Inderal) during the previous shift. Which information indicates that the patient needs immediate intervention? a. The patient's most recent BP reading is 156/94 mm Hg. b. The patient's pulse has dropped from 64 to 58 beats/minute. c. The patient has developed wheezes throughout the lung fields. d. The patient complains that the fingers and toes feel quite cold.

ANS: C The most urgent concern for this patient is the wheezes, which indicate that bronchospasm (a common adverse effect of the noncardioselective b-blockers) is occurring. The nurse should immediately obtain an oxygen saturation measurement, apply supplemental oxygen, and notify the health care provider. The mild decrease in heart rate and complaint of cold fingers and toes are associated with b-receptor block- ade but do not require any change in therapy. The BP reading may indicate that a change in medication type or dose may be indicated; however, this is not as urgently needed as addressing the bronchospasm.

The nurse notices a circular lesion with a red border and clear center on the arm of an 18-year-old summer camp counselor who is in the camp clinic complaining of chills and muscle aches. Which action should the nurse take next? a. Palpate the abdomen. b. Auscultate the heart sounds. c. Ask the patient about recent outdoor activities. d. Question the patient about immunization history.

ANS: C The patient's clinical manifestations suggest possible Lyme disease. A history of recent outdoor activities such as hikes will help confirm the diagnosis. The patient's symptoms do not suggest cardiac or abdominal problems or lack of immunization.

On admission to the burn unit, a patient with an approximate 25% total body surface area (TBSA) burn has the following initial laboratory results: Hct 58%, Hgb 18.2 mg/dL (172 g/L), serum K+ 4.9 mEq/L (4.8 mmol/L), and serum Na+ 135 mEq/L (135 mmol/L). Which action will the nurse anticipate taking now? a. Monitor urine output every 4 hours. b. Continue to monitor the laboratory results. c. Increase the rate of the ordered IV solution. d. Type and crossmatch for a blood transfusion.

ANS: C The patient's laboratory data show hemoconcentration, which may lead to a decrease in blood flow to the microcirculation unless fluid intake is increased. Because the hematocrit and hemoglobin are elevated, a transfusion is inappropriate, although transfusions may be needed after the emergent phase once the patient's fluid balance has been restored. On admission to a burn unit, the urine output would be monitored more often than every 4 hours; likely every1 hour.

After a patient experienced a brief episode of tinnitus, diplopia, and dysarthria with no residual effects, the nurse anticipates teaching the patient about a. cerebral aneurysm clipping. c. oral low-dose aspirin therapy. b. heparin intravenous infusion. d. tissue plasminogen activator (tPA).

ANS: C The patient's symptoms are consistent with transient ischemic attack (TIA), and drugs that inhibit platelet aggregation are prescribed after a TIA to prevent a stroke. Continuous heparin infusion is not routinely used after TIA or with acute ischemic stroke. The patient's symptoms are not consistent with a cerebral aneurysm. tPA is used only for acute ischemic stroke, not for TIA. DIF: Cognitive Level: Apply (application) REF: 1353 TOP: Nursing Process: Planning MSC: NCLEX: Physiological Integrity

A patient with a left-brain stroke suddenly bursts into tears when family members visit. The nurse should a. use a calm voice to ask the patient to stop the crying behavior. b. explain to the family that depression is normal following a stroke. c. have the family members leave the patient alone for a few minutes. d. teach the family that emotional outbursts are common after strokes.

ANS: D Patients who have left-sided brain stroke are prone to emotional outbursts that are not necessarily related to the emotional state of the patient. Depression after a stroke is common, but the suddenness of the patient's outburst suggests that depression is not the major cause of the behavior. The family should stay with the patient. The crying is not within the patient's control, and asking the patient to stop will lead to embarrassment. DIF: Cognitive Level: Apply (application) REF: 1364 TOP: Nursing Process: Implementation MSC: NCLEX: Psychosocial Integrity

9. The nurse has just finished teaching a hypertensive patient about the newly prescribed quinapril (Accupril). Which patient statement indicates that more teaching is needed? a. "The medication may not work as well if I take any aspirin." b. "The doctor may order a blood potassium level occasionally." c. "I will call the doctor if I notice that I have a frequent cough." d. "I won't worry if I have a little swelling around my lips and face."

ANS: D Angioedema occurring with angiotensin-converting enzyme (ACE) inhibitor therapy is an indication that the ACE inhibitor should be discontinued. The patient should be taught that if any swelling of the face or oral mucosa occurs, the health care provider should be immediately notified because this could be life threatening. The other patient statements indicate that the patient has an accurate understanding of ACE inhibitor therapy.

After the nurse assesses a 78-year-old who uses naproxen (Aleve) daily for hand and knee osteoarthritis management, which information is most important to report to the health care provider? a. Knee crepitation is noted with normal knee range of motion. b. Patient reports embarrassment about having Heberden's nodes. c. Patient's knee pain while golfing has increased over the last year. d. Laboratory results indicate blood urea nitrogen (BUN) is elevated.

ANS: D Older patients are at increased risk for renal toxicity caused by nonsteroidal antiinflammatory drugs (NSAIDs) such as naproxen. The other information will also be reported to the health care provider but is consistent with the patient's diagnosis of osteoarthritis and will not require an immediate change in the patient's treatment plan

A patient in the emergency department with sudden-onset right-sided weakness is diagnosed with an intracerebral hemorrhage. Which information about the patient is most important to communicate to the health care provider? a. The patient's speech is difficult to understand. b. The patient's blood pressure (BP) is 144/90 mm Hg. c. The patient takes a diuretic because of a history of hypertension. d. The patient has atrial fibrillation and takes warfarin (Coumadin).

ANS: D The use of warfarin probably contributed to the intracerebral bleeding and remains a risk factor for further bleeding. Administration of vitamin K is needed to reverse the effects of the warfarin, especially if the patient is to have surgery to correct the bleeding. The history of hypertension is a risk factor for the patient but has no immediate effect on the patient's care. The BP of 144/90 indicates the need for ongoing monitoring but not for any immediate change in therapy. Slurred speech is consistent with a left-sided stroke, and no change in therapy is indicated. DIF: Cognitive Level: Analyze (analysis) REF: 1349 OBJ: Special Questions: Prioritization TOP: Nursing Process: Assessment MSC: NCLEX: Physiological Integrity

The nurse is caring for a patient who has had a surgical reduction of an open fracture of the left tibia. Which assessment finding is most important to report to the health care provider? a. Left leg muscle spasms b. Serous wound drainage c. Left leg pain with movement d. Temperature 101.4° F (38.6° C)

ANS: D An elevated temperature is suggestive of possible osteomyelitis. The other clinical manifestations are typical after a repair of an open fracture.

A patient is receiving gentamicin (Garamycin) 80 mg IV twice daily for acute osteomyelitis. Which action should the nurse take before administering the gentamicin? a. Ask the patient about any nausea. b. Obtain the patient's oral temperature. c. Change the prescribed wet-to-dry dressing. d. Review the patient's blood urea nitrogen (BUN) and creatinine levels.

ANS: D Gentamicin is nephrotoxic and can cause renal failure. Monitoring the patient's temperature before gentamicin administration is not necessary. Nausea is not a common side effect of IV gentamicin. There is no need to change the dressing before gentamicin administration.

A public health nurse has reviewed local data about the incidence and prevalence of burn injuries in the community. These data are likely to support what health promotion effort? A) Education about home safety B) Education about safe storage of chemicals C) Education about workplace health threats D) Education about safe driving

Ans: A Feedback: A large majority of burns occur in the home setting; educational interventions should address this epidemiologic trend.

A burn patient is transitioning from the acute phase of the injury to the rehabilitation phase. The patient tells the nurse, ìI can't wait to have surgery to reconstruct my face so I look normal again.î What would be the nurse's best response? A) ìThat's something that you and your doctor will likely talk about after your scars mature.î B) ìThat is something for you to talk to your doctor about because it's not a nursing responsibility.î C) ìI know this is really important to you, but you have to realize that no one can make you look like you used to.î D) ìUnfortunately, it's likely that you will have most of these scars for the rest of your life.î

Ans: A Feedback: Burn reconstruction is a treatment option after all scars have matured and is discussed within the first few years after injury. Even though this is not a nursing responsibility, the nurse should still respond appropriately to the patient's query. It is true that the patient will not realistically look like he or she used to, but this does not instill hope.

A patient in the emergent/resuscitative phase of a burn injury has had blood work and arterial blood gases drawn. Upon analysis of the patient's laboratory studies, the nurse will expect the results to indicate what? A) Hyperkalemia, hyponatremia, elevated hematocrit, and metabolic acidosis B) Hypokalemia, hypernatremia, decreased hematocrit, and metabolic acidosis C) Hyperkalemia, hypernatremia, decreased hematocrit, and metabolic alkalosis D) Hypokalemia, hyponatremia, elevated hematocrit, and metabolic alkalosis

Ans: A Feedback: Fluid and electrolyte changes in the emergent/resuscitative phase of a burn injury include hyperkalemia related to the release of potassium into the extracellular fluid, hyponatremia from large amounts of sodium lost in trapped edema fluid, hemoconcentration that leads to an increased hematocrit, and loss of bicarbonate ions that results in metabolic acidosis.

A patient who is in the acute phase of recovery from a burn injury has yet to experience adequate pain control. What pain management strategy is most likely to meet this patient's needs? A) A patient-controlled analgesia (PCA) system B) Oral opioids supplemented by NSAIDs C) Distraction and relaxation techniques supplemented by NSAIDs D) A combination of benzodiazepines and topical anesthetics

Ans: A Feedback: The goal of treatment is to provide a long-acting analgesic that will provide even coverage for this long-term discomfort. It is helpful to use escalating doses when initiating the medication to reach the level of pain control that is acceptable to the patient. The use of patient-controlled analgesia (PCA) gives control to the patient and achieves this goal. Patients cannot normally achieve adequate pain control without the use of opioids, and parenteral administration is usually required.

A patient is in the acute phase of a burn injury. One of the nursing diagnoses in the plan of care is Ineffective Coping Related to Trauma of Burn Injury. What interventions appropriately address this diagnosis? Select all that apply. A) Promote truthful communication. B) Avoid asking the patient to make decisions. C) Teach the patient coping strategies. D) Administer benzodiazepines as ordered. E) Provide positive reinforcement.

Ans: A, C, E Feedback: The nurse can assist the patient to develop effective coping strategies by setting specific expectations for behavior, promoting truthful communication to build trust, helping the patient practice appropriate strategies, and giving positive reinforcement when appropriate. The patient may benefit from being able to make decisions regarding his or her care. Benzodiazepines may be needed for short-term management of anxiety, but they are not used to enhance coping.

A home care nurse is performing a visit to a patient's home to perform wound care following the patient's hospital treatment for severe burns. While interacting with the patient, the nurse should assess for evidence of what complication? A) Psychosis B) Post-traumatic stress disorder C) Delirium D) Vascular dementia

Ans: B Feedback: Post-traumatic stress disorder (PTSD) is the most common psychiatric disorder in burn survivors, with a prevalence that may be as high as 45%. As a result, it is important for the nurse to assess for this complication of burn injuries. Psychosis, delirium, and dementia are not among the noted psychiatric and psychosocial complications of burns.

A nurse who provides care on a burn unit is preparing to apply a patient's ordered topical antibiotic ointment. What action should the nurse perform when administering this medication? A) Apply the new ointment without disturbing the existing layer of ointment. B) Apply the ointment using a sterile tongue depressor. C) Apply a layer of ointment approximately 1/16 inch thick. D) Gently irrigate the wound bed after applying the antibiotic ointment.

Ans: C Feedback: After removing the old ointment from the wound bed, the nurse should apply a layer of ointment 1/16-inch thick using clean gloves. The wound would not be irrigated after application of new ointment.

The patient tells the nurse she had a history of abdominal pain, so she had a surgery to make an opening into the common bile duct to remove stones. The nurse knows that this surgery is called a A) colectomy B) cholecystectomy C) choledocholithotomy D) choledochojejunostomy

C) A choledocholithotomy is an opening into the common bile duct for the removal of stones. A colectomy is the removal of the colon. The cholecystectomy is the removal of the gallbladder. The choledochojejunostomy is an opening between the common bile duct and the jejunum.

A patient has a tracheostomy tube after reconstructive surgery for invasive head and neck cancer. What is most important for the nurse to assess before performing tracheostomy cannula care? A. Level of consciousness B. Quality of breath sounds C. Presence of the gag reflex D. Tracheostomy cuff pressure

B Before performing tracheostomy care, the nurse will auscultate lung sounds to determine the presence of secretions. To prevent aspiration, secretions must be cleared either by coughing or by suctioning before performing tracheostomy cannula care.

Physiological Integrity 3. Which assessment data collected by the nurse who is admitting a patient with chest pain suggest that the pain is caused by an acute myocardial infarction (AMI)? a. The pain increases with deep breathing. b. The pain has lasted longer than 30 minutes. c. The pain is relieved after the patient takes nitroglycerin. d. The pain is reproducible when the patient raises the arms.

B Chest pain that lasts for 20 minutes or more is characteristic of AMI. Changes in pain that occur with raising the arms or with deep breathing are more typical of musculoskeletal pain or pericarditis. Stable angina is usually relieved when the patient takes nitroglycerin. DIF: Cognitive Level: Apply (application) REF: 748 TOP: Nursing Process: Assessment MSC:

A 61-year-old patient with suspected bowel obstruction had a nasogastric tube inserted at 4:00 AM. The nurse shares in the morning report that the day shift staff should check the tube for patency at what times? A) 7:00 AM, 10:00 AM, and 1:00 PM B) 8:00 AM, 12:00 PM, and 4:00 PM C) 9:00 AM and 3:00 PM D) 9:00 AM, 12:00 PM, and 3:00 PM

B) A nasogastric tube should be checked for patency routinely at 4-hour intervals. Thus if the tube were inserted at 4:00 AM, it would be due to be checked at 8:00 AM, 12:00 PM, and 4:00 PM.

Psychosocial Integrity 22. When evaluating the effectiveness of preoperative teaching with a patient scheduled for coronary artery bypass graft (CABG) surgery using the internal mammary artery, the nurse determines that additional teaching is needed when the patient says which of the following? a. "They will circulate my blood with a machine during the surgery." b. "I will have small incisions in my leg where they will remove the vein." c. "They will use an artery near my heart to go around the area that is blocked." d. "I will need to take an aspirin every day after the surgery to keep the graft open."

B When the internal mammary artery is used there is no need to have a saphenous vein removed from the leg. The other statements by the patient are accurate and indicate that the teaching has been effective. DIF: Cognitive Level: Apply (application) REF: 752 TOP: Nursing Process: Evaluation MSC:

The results of a patients recent endoscopy indicate the presence of peptic ulcer disease (PUD). Which of the following teaching points should the nurse provide to the pt in light of his new diagnosis? A) "You'll need to drink at least two to three glasses of milk daily." B) "It would likely be beneficial for you to eliminate drinking alcohol." C) "Many people find that a minced or pureed diet eases their sxs of PUD." D) "Your medications should allow you to maintain your present diet while minimizing symptoms."

B) "It would likely be beneficial for you to eliminate drinking alcohol." Although there is no specific recommended dietary modification for PUD, most patients find it necessary to make some sort of dietary modifications to minimize symptoms. Milk may exacerbate PUD and alcohol is best avoided because it can delay healing.

The nurse is planning care for a 68-year-old patient with an abdominal mass and suspected bowel obstruction. Which factor in the patient's history increases the patient's risk for colorectal cancer? A) Osteoarthritis B) History of colorectal polyps C) History of lactose intolerance D) Use of herbs as dietary supplements

B) A history of colorectal polyps places this patient at risk for colorectal cancer. This tissue can degenerate over time and become malignant. Osteoarthritis, lactose intolerance, and the use of herbs do not pose additional risk to the patient.

Because the incidence of Lyme disease is very high in Wisconsin, the public health nurse is planning to provide community education to increase the number of people who seek health care promptly after a tick bite. What information should the nurse provide when teaching people who are at risk for a tick bite? A) The best therapy for the acute illness is an IV antibiotic. B) Check for an enlarging reddened area with a clear center. C) Surveillance is necessary during the summer months only. D) Antibiotics will prevent Lyme disease if taken for 10 days.

B) Following a tick bite, the expanding "bull's eye rash" is the most characteristic symptom that usually occurs in 3 to 30 days. There may also be flu-like symptoms and migrating joint and muscle pain. Active lesions are treated with oral antibiotics for 2 to 3 weeks, and doxycycline is effective in preventing Lyme disease when given within 3 days after the bite of a deer tick. IV therapy is used with neurologic or cardiac complications. Although ticks are most prevalent during summer months, residents of high-risk areas should check for ticks whenever they are outdoors.

The pt who is admitted with a diagnosis of diverticulitis and a history of irritable bowel disease and gastroesophageal reflux disease (GERD) has received a dose of Mylanta 30 ml PO. The nurse would evaluate its effectiveness by questioning the patient as to whether which of the following sxs has been resolved? A) Diarrhea B) Heartburn C) Constipation D) Lower abdominal pain

B) Heartburn (Mylanta is an antacid that contains both aluminum and magnesium. It is indicated for the relief of GI discomfort, such as with heartburn associated with GERD.)

The patient with sudden pain in the left upper quadrant radiating to the back and vomiting was diagnosed with acute pancreatitis. What intervention(s) should the nurse expect to include in the patient's plan of care? A) Immediately start enteral feeding to prevent malnutrition. B) Insert an NG and maintain NPO status to allow pancreas to rest. C) Initiate early prophylactic antibiotic therapy to prevent infection. D) Administer acetaminophen (Tylenol) every 4 hours for pain relief.

B) Initial treatment with acute pancreatitis will include an NG tube if there is vomiting and being NPO to decrease pancreatic enzyme stimulation and allow the pancreas to rest and heal. Fluid will be administered to treat or prevent shock. The pain will be treated with IV morphine because of the NPO status. Enteral feedings will only be used for the patient with severe acute pancreatitis in whom oral intake is not resumed. Antibiotic therapy is only needed with acute necrotizing pancreatitis and signs of infection.

When reinforcing health teaching about the management of osteoarthritis (OA), the nurse determines that the patient needs additional instruction after making which statement? A) "I should take the Naprosyn as prescribed to help control the pain." B) "I should try to stay standing all day to keep my joints from becoming stiff." C) "I can use a cane if I find it helpful in relieving the pressure on my back and hip." D) "A warm shower in the morning will help relieve the stiffness I have when I get up."

B) It is important to maintain a balance between rest and activity to prevent overstressing the joints with OA. Naproxen (Naprosyn) may be used for moderate to severe OA pain. Using a cane and warm shower to help relieve pain and morning stiffness are helpful

The nurse should administer a prn dose of magnesium hydroxide (MOM) after noting which of the following while reviewing a patient's medical record? A) Abdominal pain and bloating B) No bowel movement for 3 days C) A decrease in appetite by 50% over 24 hours D) Muscle tremors and other signs of hypomagnesemia

B) MOM is an osmotic laxative that produces a soft, semisolid stool usually within 15 minutes to 3 hours. This medication would benefit the patient who has not had a bowel movement for 3 days. MOM would not be given for abdominal pain and bloating, decreased appetite, or signs of hypomagnesemia.

The nurse is admitting a patient who is scheduled for knee arthroscopy related to osteoarthritis (OA). Which finding should the nurse expect to be present on examination of the patient's knees? A) Ulnar drift B) Pain with joint movement C) Reddened, swollen affected joints D) Stiffness that increases with movement

B) OA is characterized predominantly by joint pain upon movement and is a classic feature of the disease. Ulnar drift occurs with rheumatoid arthritis (RA) not osteoarthritis. Not all joints are reddened or swollen. Only Heberden's and Bouchard's nodes may be. Stiffness decreases with movement.

The patient with right upper quadrant abdominal pain has an abdominal ultrasound that reveals cholelithiasis. What should the nurse expect to do for this patient? A) Prevent all oral intake. B) Control abdominal pain. C) Provide enteral feedings. D) Avoid dietary cholesterol.

B) Patients with cholelithiasis can have severe pain, so controlling pain is important until the problem can be treated. NPO status may be needed if the patient will have surgery but will not be used for all patients with cholelithiasis. Enteral feedings should not be needed, and avoiding dietary cholesterol is not used to treat cholelithiasis.

A colectomy is scheduled for a 38-year-old woman with ulcerative colitis. The nurse should plan to include what prescribed measure in the preoperative preparation of this patient? A) Instruction on irrigating a colostomy B) Administration of a cleansing enema C) A high-fiber diet the day before surgery D) Administration of IV antibiotics for bowel preparation

B) Preoperative preparation for bowel surgery typically includes bowel cleansing with antibiotics, such as oral neomycin and cleansing enemas, including Fleet enemas. Instructions to irrigate the colostomy will be done postoperatively. Oral antibiotics are given preoperatively, and an IV antibiotic may be used in the OR. A clear liquid diet will be used the day before surgery with the bowel cleansing.

The patient had a car accident and was "scared to death." The patient is now reporting constipation. What affecting the gastrointestinal (GI) tract does the nurse know could be contributing to the constipation? A) The patient is too nervous to eat or drink, so there is no stool. B) The sympathetic nervous system was activated, so the GI tract was slowed. C) The parasympathetic nervous system is now functioning to slow the GI tract. D) The circulation in the GI system has been increased, so less waste is removed.

B) The constipation is most likely related to the sympathetic nervous system activation from the stress related to the accident. SNS activation can decrease peristalsis. Even without oral intake for a short time, stool will be formed. The parasympathetic system stimulates peristalsis. The circulation to the GI system is decreased with stress.

The nurse admits a 73-yr-old male patient with dementia for treatment of uncontrolled hypertension. The nurse will closely monitor for hypokalemia if the patient receives which medication? A. Clonidine (Catapres) B. Bumetanide (Bumex) C.Amiloride (Midamor) D. Spironolactone (Aldactone)

B. Bumetanide is a loop diuretic. Hypokalemia is a common adverse effect of this medication. Amiloride is a potassium-sparing diuretic. Spironolactone is an aldosterone-receptor blocker. Hyperkalemia is an adverse effect of both amiloride and spironolactone. Clonidine is a central-acting α-adrenergic antagonist and does not cause electrolyte abnormalities.

The nurse is caring for a patient admitted with a history of hypertension. The patient's medication history includes hydrochlorothiazide daily for the past 10 years. Which parameter would indicate the optimal intended effect of this drug therapy? A. Weight loss of 2 lb B. BP 128/86 mm Hg Correct C. Absence of ankle edema D. Output of 600 mL per 8 hours

B. Hydrochlorothiazide may be used alone as monotherapy to manage hypertension or in combination with other medications if not effective alone. After the first few weeks of therapy, the diuretic effect diminishes, but the antihypertensive effect remains. Because the patient has been taking this medication for 10 years, the most direct measurement of its intended effect would be the blood pressure.

When teaching a patient about dietary management of stage 1 hypertension, which instruction is most appropriate? A. Increase water intake. B. Restrict sodium intake. C. Increase protein intake. D. Use calcium supplements.

B. The patient should decrease intake of sodium. This will help to control hypertension, which can be aggravated by excessive salt intake, which in turn leads to fluid retention. Protein intake does not affect hypertension. Calcium supplements are not recommended to lower blood pressure.

Physiological Integrity 14. A patient with ST-segment elevation in three contiguous electrocardiographic (ECG) leads is admitted to the emergency department (ED) and diagnosed as having an ST-segment-elevation myocardial infarction (STEMI). Which question should the nurse ask to determine whether the patient is a candidate for thrombolytic therapy? a. "Do you have any allergies?" b. "Do you take aspirin on a daily basis?" c. "What time did your chest pain begin?" d. "Can you rate your chest pain using a 0 to 10 scale?"

C Thrombolytic therapy should be started within 6 hours of the onset of the myocardial infarction (MI), so the time at which the chest pain started is a major determinant of the appropriateness of this treatment. The other information will also be needed, but it will not be a factor in the decision about thrombolytic therapy. DIF: Cognitive Level: Apply (application) REF: 743 TOP: Nursing Process: Assessment MSC:

The patient developed gout while hospitalized for a heart attack. When doing discharge teaching for this patient who takes aspirin for its antiplatelet effect, what should the nurse include about preventing future attacks of gout? A Limit fluid intake. B Administration of probenecid (Benemid) C Administration of allopurinol (Zyloprim) D Administration of nonsteroidal antiinflammatory drugs (NSAIDs)

C Administration of allopurinol (Zyloprim)

The UAP is taking orthostatic vital signs. In the supine position, the blood pressure (BP) is 130/80 mm Hg, and the heart rate (HR) is 80 beats/min. In the sitting position, the BP is 140/80, and the HR is 90 beats/min. Which action should the nurse instruct the UAP to take next? A. Repeat BP and HR in this position. B. Record the BP and HR measurements. C. Take BP and HR with patient standing. D. Return the patient to the supine position

C. The vital signs taken do not reflect orthostatic changes, so the UAP will continue with the measurements while the patient is standing. There is no need to repeat or delay the readings. The patient does not need to return to the supine positon. When assessing for orthostatic changes, the UAP will take the BP and pulse in the supine position, then place the patient in a sitting position for 1 to 2 minutes and repeat the readings, and then reposition to the standing position for 1 to 2 minutes and repeat the readings. Results consistent with orthostatic changes would have a decrease of 20 mm Hg or more in systolic BP, a decrease of 10 mm Hg or more in diastolic BP, and/or an increase in HR of greater than or equal to 20 beats/min with position changes.

The nurse is admitting a patient who complains of a new onset of lower back pain. To differentiate between the pain of a lumbar herniated disc and lower back pain from other causes, what would be the best question for the nurse to ask the patient? A. "Is the pain worse in the morning or in the evening?" B. "Is the pain sharp or stabbing or burning or aching?" C. "Does the pain radiate down the buttock or into the leg?" D. "Is the pain totally relieved by analgesics, such as acetaminophen (Tylenol)?"

C. "Does the pain radiate down the buttock or into the leg?" Lower back pain associated with a herniated lumbar disc is accompanied by radiation along the sciatic nerve and can be commonly described as traveling through the buttock, to the posterior thigh, or down the leg. This is because the herniated disc causes compression on spinal nerves as they exit the spinal column. Time of occurrence, type of pain, and pain relief questions do not elicit differentiating data.

The patient sustained a full-thickness burn encompassing the entire right arm. What is the best indicator an escharotomy achieved its desired effect? A. Patient rates the pain at less than 4. B. Blood pressure remains above 120/80 mm Hg. C. Right fingers blanch with a 2-second refill. D. Patient maintains full range of motion for the right arm.

C. Circulation to the extremities can be severely impaired by deep circumferential burns and subsequent edema that impairs the blood supply. An escharotomy (electrocautery incision through the full-thickness eschar) is performed to restore circulation. Normal refill is less than 2 seconds.

In caring for the patient with angina, the patient said, "I walked to the bathroom. While I was having a bowel movement, I started having the worst chest pain ever, like before I was admitted. I called for a nurse, but the pain is gone now." What further assessment data should the nurse obtain from the patient? A. "What precipitated the pain?" B. "Has the pain changed this time?" C. "In what areas did you feel this pain?" D. "Rate the pain on a scale from 0 to 10, with 0 being no pain and 10 being the worst pain you can imagine."

C. "In what areas did you feel this pain?" Using PQRST, the assessment data not volunteered by the patient is the radiation of pain, the area the patient felt the pain, and if it radiated. The precipitating event was going to the bathroom and having a bowel movement. The quality of the pain was "like before I was admitted," although a more specific description may be helpful. Severity of the pain was the "worst chest pain ever," although an actual number may be needed. Timing is supplied by the patient describing when the pain occurred and that he had previously had this pain.

The nurse receives report from the licensed practical nurse about care provided to patients on the orthopedic surgical unit. It is most important for the nurse to follow up on which statement? A. "The patient who had a spinal fusion 12 hours ago has hypoactive bowel sounds and is not passing flatus." B. "The patient who had cervical spine surgery 2 days ago wants to wear her soft cervical collar when out of bed." C. "The patient who had spinal surgery 3 hours ago is complaining of a headache and has clear drainage on the dressing." D. "The patient who had a laminectomy 24 hours ago is using patient-controlled analgesia with morphine for pain management."

C. "The patient who had spinal surgery 3 hours ago is complaining of a headache and has clear drainage on the dressing." After spinal surgery there is potential for cerebrospinal fluid (CSF) leakage. Severe headache or leakage of CSF (clear or slightly yellow) on the dressing should be reported immediately. The drainage is CSF if a dipstick test is positive for glucose. Patients after spinal surgery may experience paralytic ileus and interference with bowel function for several days. Postoperatively most patients require opioids such as morphine IV for 24 to 48 hours. Patient-controlled analgesia is the preferred method for pain management during this time. After cervical spine surgery patients often wear a soft or hard cervical collar to immobilize the neck.

The nurse is reinforcing general health teaching with a 64-year-old patient with osteoarthritis (OA) of the hip. Which of the following points would the nurse include in this review of the disorder (select all that apply)? A. OA cannot be successfully treated with any current therapy options. B. OA is an inflammatory disease of the joints that may present symptoms at any age. C.Joint degeneration with pain and disability occurs in the majority of people by the age of 60. D. OA is more common with aging, but usually it remains confined to a few joints and does not cause crippling. E.OA can be prevented from progressing when well controlled with a regimen of exercise, diet, and medication.

Correct answer: D. OA is more common with aging, but usually it remains confined to a few joints and does not cause crippling. E.OA can be prevented from progressing when well controlled with a regimen of exercise, diet, and medication. Rationale: OA occurs with greater frequency with increasing age, but it usually remains confined to a few joints and can be managed with a combination of exercise, diet, and medication. OA can lead to significant disability.

Teach the patient with ankylosing spondylitis the importance of a. regular exercise and maintaining proper posture b. continuing with physical activity during flare-ups c. avoiding extremes in environmental temperatures d. applying cool compresses for relief of local symptoms

Correct answer: a Rationale: Patients with ankylosing spondylitis (AS) should exercise after pain and stiffness are managed. Postural control is important to minimize spinal deformity. The exercise regimen should include back, neck, and chest stretches. The nurse should educate the patient with AS about regular exercise and attention to posture, local moist-heat applications, and knowledgeable use of drugs. The nurse should discourage excessive physical exertion during periods of active flare-up of the disease. Proper positioning at rest is essential. The mattress should be firm, and the patient should sleep on the back with a flat pillow, avoiding positions that encourage flexion deformity. Postural training emphasizes avoiding spinal flexion (e.g., leaning over a desk), heavy lifting, and prolonged walking, standing, or sitting.

When administering medications to the patient with gout, the nurse would recognize which of the following as a treatment for chronic disease? a. Colchicine b. Febuxostat c. Sulfasalazine d. Cyclosporine

Correct answer: b Rationale: Febuxostat (Uloric), a selective inhibitor of xanthine oxidase, is given for long-term management of hyperuricemia in persons with chronic gout. Acute gouty arthritis is treated with colchicine and nonsteroidal antiinflammatory drugs (NSAIDs).

In teaching a patient with chronic fatigue syndrome (CFS) about this disorder, the nurse understands that a. palpating tender points is an indicator of CFS severity b. many symptoms are similar to fibromyalgia syndrome c. definitive treatment includes low-dose hydrocortisone d. CFS is characterized by progressive memory impairment

Correct answer: b Rationale: Fibromyalgia syndrome (FS) and chronic fatigue syndrome (CFS) have several commonalities. Both occur in previously healthy, young, and middle-aged women; the cause of both includes an infectious trigger, dysfunction of the hypothalamic-pituitary-adrenal axis or an alteration in central nervous system; and common clinical manifestations are malaise and fatigue, cognitive dysfunction, headaches, sleep disturbances, depression, anxiety, fever, and generalized musculoskeletal pain. Both diseases have symptoms that fluctuate over time, and both disorders have no definitive laboratory tests or joint and muscle examinations. They remain diagnoses of exclusion. Treatment for both disorders is symptomatic and may include antidepressant drugs. Other measures are heat, massage, regular stretching, biofeedback, stress management, and relaxation training.

In teaching a patient with SLE about the disorder, the nurse knows that the pathophysiology of SLE includes a. circulating immune complexes formed from IgG autoantibodies reacting with IgG b. an autoimmune T-cell reaction that results in destruction of the deep dermal skin layer c. immunologic dysfunction leading to chronic inflammation in the cartilage and muscles d. the production of a variety of autoantibodies directed against components of the cell nucleus

Correct answer: d Rationale: Systemic lupus erythematosus (SLE) is characterized by the production of many autoantibodies against nucleic acids (e.g., single-and double-stranded DNA), erythrocytes, coagulation proteins, lymphocytes, platelets, and many other self-proteins. Autoimmune reactions characteristically are directed against constituents of the cell nucleus (e.g., antinuclear antibodies [ANAs]), particularly DNA. Circulating immune complexes containing antibody against DNA are deposited in the basement membranes of capillaries in the kidneys, heart, skin, brain, and joints. Complement is activated, and inflammation occurs. The overaggressive antibody response is also related to activation of B and T cells. The specific manifestations of SLE depend on which cell types or organs are involved. SLE is a type III hypersensitivity response.

In assessing the joints of a patient with rheumatoid arthritis, the nurse understands that the joints are damaged by (select all that apply) a. bony ankylosis following inflammation of the joints b. the deterioration of cartilage by proteolytic enzymes c. the development of Heberden's nodes in the joint capsule d.. increased cartilage and bony growth at the joint margins e. invasion of pannus into the joint causing a loss of cartilage

Correct answers: a, e Rationale: Bony ankylosis is the union of the bones of a joint by proliferation of bone cells, resulting in complete immobility. Bony ankylosis occurs with advanced rheumatoid arthritis. Joint changes from chronic inflammation begin when the hypertrophied synovial membrane invades the surrounding cartilage, ligaments, tendons, and joint capsule. Pannus (i.e., highly vascular granulation tissue) forms within the joint. It eventually covers and erodes the entire surface of the articular cartilage. The production of inflammatory cytokines at the pannus-cartilage junction further contributes to cartilage destruction. The pannus scars and shortens supporting structures such as tendons and ligaments, ultimately causing joint laxity, subluxation, and contracture.

When caring for a patient with systemic sclerosis, the nurse knows it is important to instruct the patient related to (select all that apply) a. avoiding the consumption of high-purine foods b. strategies for good dental hygiene and mouth care c. protecting the extremities from hot and cold temperatures d. maintaining joint function and preserving muscle strength e. performing mouth excursion (yawning) exercises on a daily basis

Correct answers: b, c, d, e Rationale: Systemic sclerosis (SS), or scleroderma, is a disorder of connective tissue characterized by fibrotic, degenerative, and occasionally inflammatory changes in the skin, blood vessels, synovium, skeletal muscle, and internal organs. The nurse should include the following in the teaching plan for a patient with SS: daily oral hygiene (neglect may increase tooth and gingival problems); protection of hands and feet from cold exposure and possible burns or cuts (wounds heal slowly); avoidance of emotional stress and cold ambient temperatures (they aggravate Raynaud's phenomenon); isometric exercises for arthropathy (no joint movement occurs); use of assistive devices as appropriate and organization of activities to preserve strength and reduce disability; and mouth excursion (i.e., yawning with an open mouth) (helps maintain temporomandibular joint function).

15. A patient was exposed to human immunodeficiency virus (HIV) 2 weeks ago through sharing needles with other substance users. What symptoms will the nurse teach the patient to report that would indicate the patient has developed an acute HIV infection? A. Cough, diarrhea, headaches, blurred vision, muscle fatigue B. Night sweats, fatigue, fever, and persistent generalized lymphadenopathy C. Oropharyngeal candidiasis or thrush, vaginal candidal infection, or oral or genital herpes D. Flu-like symptoms such as fever, sore throat, swollen lymph glands, nausea, or diarrhea

D Clinical manifestations of an acute infection with HIV include flu-like symptoms between 2 to 4 weeks after exposure. Early chronic HIV infection clinical manifestations are either asymptomatic or include fatigue, headache, low-grade fever, night sweats, and persistent generalized lymphadenopathy. Intermediate chronic HIV infection clinical manifestations include candidal infections, shingles, oral or genital herpes, bacterial infections, Kaposi sarcoma, or oral hairy leukoplakia. Late chronic HIV infection or acquired immunodeficiency syndrome (AIDS) includes opportunistic diseases (infections and cancer).

14. The nurse is providing postoperative care for a patient with human immunodeficiency virus (HIV) infection after an appendectomy. What type of precautions should the nurse observe to prevent the transmission of this disease? A. Droplet precautions B. Contact precautions C. Airborne precautions D. Standard precautions

D Standard precautions are indicated for prevention of transmission of HIV to the health care worker. HIV is not transmitted by casual contact or respiratory droplets. HIV may be transmitted through sexual intercourse with an infected partner; exposure to HIV-infected blood or blood products; and perinatal transmission during pregnancy, at delivery, or though breastfeeding.

The nurse teaches a patient with hypertension and osteoarthritis about actions to prevent and control epistaxis. Which statement, if made by the patient, indicates further teaching is required? A. "I should avoid using ibuprofen for pain and discomfort." B. "It is important for me to take my blood pressure medication every day." C. "I will sit down and pinch the tip of my nose for at least 10 to 15 minutes." D. "If I get a nosebleed, I will lie down flat and raise my feet above my heart."

D A simple measure to control epistaxis (or a nosebleed) is for the patient to remain quiet in a sitting position. Another measure is to apply direct pressure by pinching the entire soft lower portion of the nose for 10 to 15 minutes. Aspirin and nonsteroidal antiinflammatory drugs such as ibuprofen increase the bleeding time and should be avoided. Elevated blood pressure makes epistaxis more difficult to control. The patient should continue with antihypertensive medications as prescribed.

Physiological Integrity 32. A patient admitted to the coronary care unit (CCU) with an ST-segment-elevation myocardial infarction (STEMI) is restless and anxious. The blood pressure is 86/40 and heart rate is 123. Based on this information, which nursing diagnosis is a priority for the patient? a. Acute pain related to myocardial infarction b. Anxiety related to perceived threat of death c. Stress overload related to acute change in health d. Decreased cardiac output related to cardiogenic shock

D All the nursing diagnoses may be appropriate for this patient, but the hypotension and tachycardia indicate decreased cardiac output and shock from the damaged myocardium. This will result in decreased perfusion to all vital organs (e.g., brain, kidney, heart) and is a priority. DIF: Cognitive Level: Apply (application) REF: 746-747 OBJ: Special Questions: Prioritization TOP: Nursing Process: Diagnosis MSC:

A 67-yr-old woman with hypertension is admitted to the emergency department with a blood pressure of 234/148 mm Hg and was started on nitroprusside (Nitropress). After one hour of treatment, the mean arterial blood pressure (MAP) is 55 mm Hg. Which nursing action is a priority? A. Start an infusion of 0.9% normal saline at 100 mL/hr. B. Maintain the current administration rate of the nitroprusside. C. Request insertion of an arterial line for accurate blood pressure monitoring. D. Stop the nitroprusside infusion and assess the patient for potential complications.

D. Nitroprusside is a potent vasodilator medication. A blood pressure of 234/118 mm Hg would have a calculated MAP of 177 mm Hg. Subtracting 25% (or 44 mm Hg) = 133 mm Hg. The initial treatment goal is to decrease MAP by no more than 25% within minutes to 1 hour. For this patient, the goal MAP would be approximately 133 mm Hg. Minimal MAP required to perfuse organs is around 60 to 65 mm Hg. Lowering the blood pressure too rapidly may decrease cerebral, coronary, or renal perfusion and could precipitate a stroke, myocardial infarction, or renal failure. The priority is to stop the nitroprusside infusion and then use fluids only if necessary to support restoration of MAP.

The patient is being dismissed from the hospital after ACS and will be attending rehabilitation. What information does the patient need to be taught about the early recovery phase of rehabilitation? A. Therapeutic lifestyle changes should become lifelong habits. B. Physical activity is always started in the hospital and continued at home. C. Attention will focus on management of chest pain, anxiety, dysrhythmias, and other complications. D. Activity level is gradually increased under cardiac rehabilitation team supervision and with ECG monitoring.

D. Activity level is gradually increased under cardiac rehabilitation team supervision and with ECG monitoring. In the early recovery phase after the patient is dismissed from the hospital, the activity level is gradually increased under supervision and with ECG monitoring. The late recovery phase includes therapeutic lifestyle changes that become lifelong habits. In the first phase of recovery, activity is dependent on the severity of the angina or MI, and attention is focused on the management of chest pain, anxiety, dysrhythmias, and other complications. With early recovery phase, the cardiac rehabilitation team may suggest that physical activity be initiated at home, but this is not always done.

Multiple patients arrive in the emergency department from a house fire. Which patient is a priority? A. Patient with erythremic, dry burns over the arms and a history of taking prednisone B. Patient with moist blisters over the chest and who reports pain as 10 C. Patient with dry, black skin on one hand and a history of diabetes mellitus D. Patient with multiple reddened skin areas on the chest and with high-pitched respiratory sounds

D. Airway injury is a priority, and stridor results from a narrowing of the airway caused by edema. A history of prednisone use or diabetes is a concern for long-term infection risk, but the airway is always first.

The nurse is reinforcing health teaching about osteoporosis with a 72-year-old patient admitted to the hospital. In reviewing this disorder, what should the nurse explain to the patient? A. With a family history of osteoporosis, there is no way to prevent or slow bone resorption. B. Continuous, low-dose corticosteroid treatment is effective in stopping the course of osteoporosis. C. Estrogen therapy must be maintained to prevent rapid progression of the osteoporosis. D. Even with a family history of osteoporosis, the calcium loss from bones can be slowed by increased calcium intake and exercise.

D. Even with a family history of osteoporosis, the calcium loss from bones can be slowed by increased calcium intake and exercise. The rate of progression of osteoporosis can be slowed if the patient takes calcium supplements and/or foods high in calcium and engages in regular weight-bearing exercise. Corticosteroids interfere with bone metabolism. Estrogen therapy is no longer used to prevent osteoporosis because of the associated increased risk of heart disease and breast and uterine cancer.

A 67-year-old patient hospitalized with osteomyelitis has an order for bed rest with bathroom privileges with the affected foot elevated on two pillows. The nurse would place highest priority on which intervention? A. Ambulate the patient to the bathroom every 2 hours. B. Ask the patient about preferred activities to relieve boredom. C. Allow the patient to dangle legs at the bedside every 2 to 4 hours. D. Perform frequent position changes and range-of-motion exercises.

D. Perform frequent position changes and range-of-motion exercises. The patient is at risk for atelectasis of the lungs and for contractures because of prescribed bed rest. For this reason, the nurse should place the priority on changing the patient's position frequently to promote lung expansion and performing range-of-motion (ROM) exercises to prevent contractures. Assisting the patient to the bathroom will keep the patient safe as the patient is in pain, but it may not be needed every 2 hours. Providing activities to relieve boredom will assist the patient to cope with the bed rest, and dangling the legs every 2 to 4 hours may be too painful. The priority is position changes and ROM exercises.

B

The nurse determines that a patient is experiencing common adverse effects from the inhaled corticosteroid beclomethasone after what occurs? Hypertension and pulmonary edema Oropharyngeal candidiasis and hoarseness Elevation of blood glucose and calcium levels Adrenocortical dysfunction and hyperglycemia

B

The nurse determines that the patient understood medication instructions about the use of a spacer device when taking inhaled medications after hearing the patient state what as the primary benefit? "I will pay less for medication because it will last longer." "More of the medication will get down into my lungs to help my breathing." "Now I will not need to breathe in as deeply when taking the inhaler medications." "This device will make it so much easier and faster to take my inhaled medications."

C

The nurse evaluates that a patient is experiencing the expected beneficial effects of ipratropium after noting which assessment finding? Decreased respiratory rate Increased respiratory rate Increased peak flow readings Decreased sputum production

B

Which test result identifies that a patient with asthma is responding to treatment? An increase in CO2 levels A decreased exhaled nitric oxide A decrease in white blood cell count An increase in serum bicarbonate levels

C

Which position is most appropriate for the nurse to place a patient experiencing an asthma exacerbation? Supine Lithotomy High Fowler's Reverse Trendelenburg

The nurse is conducting discharge teaching for a patient with metastatic lung cancer who was admitted with a bowel impaction. Which instructions would be most helpful to prevent further episodes of constipation? a. Maintain a high intake of fluid and fiber in the diet. b. Reduce intake of medications causing constipation. c. Eat several small meals per day to maintain bowel motility. d. Sit upright during meals to increase bowel motility by gravity.

a. Maintain a high intake of fluid and fiber in the diet. Increased fluid intake and a high-fiber diet reduce the incidence of constipation caused by immobility, medications, and other factors. Fluid and fiber provide bulk that in turn increases peristalsis and bowel motility. Analgesics taken for lung cancer probably cannot be reduced. Other medications may decrease constipation, but it is best to avoid laxatives. Eating several small meals per day and position do not facilitate bowel motility. Defecation is easiest when the person sits on the commode with the knees higher than the hips.

After change-of-shift report, which patient should the nurse assess first? a. Patient with myasthenia gravis who is reporting increased muscle weakness b. Patient with a bilateral headache described as "like a band around my head" c. Patient with seizures who is scheduled to receive a dose of phenytoin (Dilantin) d. Patient with Parkinson's disease who has developed cogwheel rigidity of the arms

a. Patient with myasthenia gravis who is reporting increased muscle weakness Because increased muscle weakness may indicate the onset of a myasthenic crisis, the nurse should assess this patient first.

7. Which treatments in CF would the nurse expect to implement in the management plan of patients with CF (select all that apply)? a. Sperm banking b. IV corticosteroids on a chronic basis c. Airway clearance techniques (e.g., Acapella) d. GoLYTELY given PRN for severe constipation e. Inhaled tobramycin to combat Pseudomonas infection

a. Sperm banking c. Airway clearance techniques (e.g., Acapella) d. GoLYTELY given PRN for severe constipation e. Inhaled tobramycin to combat Pseudomonas infection

A patient is admitted with first- and second-degree burns covering the face, neck, entire right upper extremity, and the right anterior trunk area. Using the rule of nines, the nurse would calculate the extent of these burns as being A. 9%. B. 18%. C. 22.5%. D. 36%.

c. Using the rule of nines, the face and neck together encompass 4.5% of the body area; the right upper arm encompasses 9% of the body area; and the entire anterior trunk encompasses 18% of the body area. Since the patient has burns on only the right side of the anterior trunk, the nurse would assess that burn as encompassing half of the 18%, or 9%. Therefore adding the three areas together, the nurse would correctly calculate the extent of this patient's burns to cover approximately 22.5% of the total body surface area.

7. The client is complaining of joint stiffness, especially in the morning. Which diagnostic tests would the nurse expect the HCP to order to R/O OA? a. Full body MRI scan b. Serum studies for synovial fluid amount c. X-ray of the affected joints d. Serum erythrocyte sedimentation rate (ESR)

c. X-ray of the affected joints

A nurse is caring for a client with a new donor site that was harvested to treat a burn. The nurse should position the client to: a. allow ventilation of the site b. make the site dependent c. avoid pressure on the site d. keep the site fully covered

c. a universal concern in the care of donor sites for burn care is to keep the site away from sources of pressure.

The adult client was burned as a result of an explosion. The burn initially affected the client's entire face (anterior half of the head) and the upper half of the anterior torso, and there were circumferential burns to the lower half of both arms. The client's clothes caught on fire, and the client ran, causing subsequent burn injuries to the posterior surface of the head and the upper half of the posterior torso. Using the rule of nines, what would be the extent of the burn injury? a. 18% b. 24% c. 36% d. 48%

c. anterior head = 4.5%, upper half of anterior torso = 9%, lower half of both arms is 9%, posterior head 4.5%, upper half of posterior torso 9%, total 36%

The nurse is caring for a client following an autograft and grafting to a burn would on the right knee. Which of the following would the nurse anticipate to be prescribed for the client? a. out of bed b. brp c. Immobilization of the affected leg d. placing the affected leg in a dependent position

c. autografts placed over joints or on the lower extremities after surgery often are elevated and immobilized for 3-7 days. this period allows the autograft time to adhere to the wound bed.

The condition of a client with extensive third degree burns begins to deteriorate. The nurse is aware that which type of shock may occur as a result of inadequate circulating blood volume that occurs with a burn injury? a. cardiogenic b. distributive c. hypovolemic d. septic

c. burns and the resulting low circulating fluid volume can cause hypovolemic shock.

5. The major advantage of a Venturi mask is that it can a. deliver up to 80% O2. b. provide continuous 100% humidity. c. deliver a precise concentration of O2. d. be used while a patient eats and sleeps.

c. deliver a precise concentration of O2.

The nurse is caring for a postoperative patient with a colostomy. The nurse is preparing to administer a dose of famotidine (Pepcid) when the patient asks why the medication was ordered since the patient does not have a history of heartburn or gastroesophageal reflux disease (GERD). What response by the nurse would be the most appropriate? a. "This will prevent air from accumulating in the stomach, causing gas pains." b. "This will prevent the heartburn that occurs as a side effect of general anesthesia." c. "The stress of surgery is likely to cause stomach bleeding if you do not receive it." d. "This will reduce the amount of HCl in the stomach until the nasogastric tube is removed and you can eat a regular diet again."

d. "This will reduce the amount of HCl in the stomach until the nasogastric tube is removed and you can eat a regular diet again." Famotidine is an H2-receptor antagonist that inhibits gastric HCl secretion and thus minimizes damage to gastric mucosa while the patient is not eating a regular diet after surgery. Famotidine does not prevent air from accumulating in the stomach or stop the stomach from bleeding. Heartburn is not a side effect of general anesthesia.

The nurse is caring for a patient with partial- and full-thickness burns to 65% of the body. When planning nutritional interventions for this patient, the nurse should implement which of the following dietary choices? A. Full liquids only B. Whatever the patient requests C. High-protein and low-sodium foods D. High-calorie and high-protein foods

d. A hypermetabolic state occurs proportional to the size of the burn area. Massive catabolism can occur and is characterized by protein breakdown and increases gluconeogenesis. Caloric needs are often in the 5000-kcal range. Failure to supply adequate calories and protein leads to malnutrition and delayed healing.

17. Which psychosocial problem would be priority for a client diagnosed with RA? a. Alteration in comfort b. Ineffective coping c. Anxiety d. Altered body image

d. Altered body image

A patient is admitted to the burn center with burns over his head, neck, chest, back, and left arm and hand after an explosion and fire in his garage. On admission to the unit, you auscultate wheezes throughout the lung fields. On reassessment, you notice that the wheezes are gone and the breath sounds are greatly diminished. Which of the following actions is the most appropriate next step? a. place the pt in high fowler's position b. encourage the pt to cough and auscultate the lungs again c. document the results and continue to monitor the pt's progress d. anticipate the need for endotracheal intubation and notify the physician

d. Anticipate the need for endotracheal intubation, and notify the physician. Inhalation injury results in exposure of respiratory tract to intense heat or flames with inhalation of noxious chemicals, smoke, or carbon monoxide. You should anticipate the need for intubation.

21. The client with early-stage RA is being discharged from the outpatient clinic. Which discharge instructions should the nurse teach regarding the use of NSAIDs? a. Take an over-the-counter medication for the stomach b. Drink a full glass of water with each pill c. If a dose is missed, double the medication at the next dosing time d. Avoid taking the NSAID on an empty stomach

d. Avoid taking the NSAID on an empty stomach

The nurse is preparing to administer a scheduled dose of docusate sodium (Colace) when the patient reports an episode of loose stool and does not want to take the medication. What is the appropriate action by the nurse? a. Write an incident report about this untoward event. b. Attempt to have the family convince the patient to take the ordered dose. c. Withhold the medication at this time and try to administer it later in the day. d. Chart the dose as not given on the medical record and explain in the nursing progress notes.

d. Chart the dose as not given on the medical record and explain in the nursing progress notes. Whenever a patient refuses medication, the dose should be charted as not given with an explanation of the reason documented in the nursing progress notes. In this instance, the refusal indicates good judgment by the patient, and the patient should not be encouraged to take it today.

The nurse would question the use of which cathartic agent in a patient with renal insufficiency? a. Bisacodyl (Dulcolax) b. Lubiprostone (Amitiza) c. Cascara sagrada (Senekot) d. Magnesium hydroxide (Milk of Magnesia)

d. Magnesium hydroxide (Milk of Magnesia) Milk of Magnesia may cause hypermagnesemia in patients with renal insufficiency. The nurse should question this order with the health care provider. Bisacodyl, lubiprostone, and cascara sagrada are safe to use in patients with renal insufficiency as long as the patient is not currently dehydrated.

The nurse asks a 68-year-old patient scheduled for colectomy to sign the operative permit as directed in the physician's preoperative orders. The patient states that the physician has not really explained very well what is involved in the surgical procedure. What is the most appropriate action by the nurse? a. Ask family members whether they have discussed the surgical procedure with the physician. b. Have the patient sign the form and state the physician will visit to explain the procedure before surgery. c. Explain the planned surgical procedure as well as possible and have the patient sign the consent form. d. Delay the patient's signature on the consent and notify the physician about the conversation with the patient.

d. Delay the patient's signature on the consent and notify the physician about the conversation with the patient. The patient should not be asked to sign a consent form unless the procedure has been explained to the satisfaction of the patient. The nurse should notify the physician, who has the responsibility for obtaining consent.

2. The client has been diagnosed with OA for the last 7 years and has tried multiple medical treatments and alternative treatments but still has significant joint pain. Which psychosocial client problem would the nurse identify? a. Severe pain b. Body-image disturbance c. Knowledge deficit d. Depression

d. Depression

Which medication taken by a patient with *restless legs syndrome* should the nurse discuss with the patient? a. Ibuprofen c. Acetaminophen b. Multivitamin d. Diphenhydramine

d. Diphenhydramine *Antihistamines* can *aggravate restless legs syndrome*.

When teaching the patient about the diet for diverticular disease, which foods should the nurse recommend? a. White bread, cheese, and green beans b. Fresh tomatoes, pears, and corn flakes c. Oranges, baked potatoes, and raw carrots d. Dried beans, All Bran (100%) cereal, and raspberries

d. Dried beans, All Bran (100%) cereal, and raspberries A high fiber diet is recommended for diverticular disease. Dried beans, All Bran (100%) cereal, and raspberries all have higher amounts of fiber than white bread, cheese, green beans, fresh tomatoes, pears, corn flakes, oranges, baked potatoes, and raw carrots.

A patient has been treated for second- and third-degree burns over 30% of his body and is now ready for discharge. You provide discharge instructions related to wound care. Which statement indicates that the patient understands the instructions? a. I can expect occasional periods of low grade fever and can take Tylenol every 4 hours b. I must wear my jobst elastic garment all day and can only remove it when I'm going to bed. c. I will need to take sponge baths at home to avoid exposing the wounds to unsterile bath water. d. If any healed areas break open, I should cover them with a sterile dressing and then immediately report it."

d. If any healed areas break open, I should cover them with a sterile dressing and then immediately report it."

Which nursing diagnosis is of *highest priority* for a patient with *Parkinson's disease* who is *unable to move the facial muscles*? a. Activity intolerance b. Self-care deficit: toileting c. Ineffective self-health management d. Imbalanced nutrition: less than body requirements

d. Imbalanced nutrition: less than body requirements The data about the patient indicate that poor nutrition will be a concern because of decreased swallowing.

The nurse is preparing to administer a dose of bisacodyl (Dulcolax). In explaining the medication to the patient, the nurse would explain that it acts in what way? a. Increases bulk in the stool b. Lubricates the intestinal tract to soften feces c. Increases fluid retention in the intestinal tract d. Increases peristalsis by stimulating nerves in the colon wall

d. Increases peristalsis by stimulating nerves in the colon wall Bisacodyl is a stimulant laxative that aids in producing a bowel movement by irritating the colon wall and stimulating enteric nerves. It is available in oral and suppository forms. Fiber and bulk forming drugs increase bulk in the stool; water and stool softeners soften feces, and saline and osmotic solutions cause fluid retention in the intestinal tract.

8. The client diagnosed with OA is prescribed a NSAID. Which instruction should the nurse teach the client? a. Take the medication on an empty stomach b. Make sure the client tapers the medication when discontinuing c. Apply the medication topically over the affected joints d. Notify the HCP if vomiting blood

d. Notify the HCP if vomiting blood

4. The nurse's responsibility for a patient with a suspected disc herniation who is experiencing acute pain and muscle spasms is a.encouraging total bed rest for several days. b.teaching the principles of back strengthening exercises. c.stressing the importance of straight-leg raises to decrease pain. d.promoting the use of cold and hot compresses and pain medication.

d.promoting the use of cold and hot compresses and pain medication. If the acute muscle spasms and accompanying pain are not severe and debilitating, the patient may be treated on an outpatient basis with nonsteroidal antiinflammatory drugs (NSAIDs; e.g., acetaminophen) and muscle relaxants (e.g., cyclobenzaprine [Flexeril]). Massage and back manipulation, acupuncture, and the application of cold and hot compresses may help some patients. Severe pain may necessitate a brief course of opioid analgesics. A brief period (1 to 2 days) of rest at home may be necessary for some people; most patients do better with a continuation of their regular activities. Prolonged bed rest should be avoided. All patients during this time should refrain from activities that aggravate the pain, including lifting, bending, twisting, and prolonged sitting.


Conjuntos de estudio relacionados

PETER CHEN EXAM 2 (CH 5,6,7,8) - 1

View Set

Chapter 11: Childhood and Neurodevelopmental Disorders

View Set

8.3 Beyond the Mississippi: Louisiana Purchase and the Expedition of Lewis and Clark

View Set

Chapter 1: What is Plant Biology?

View Set